NURS 104 Exam 1

Pataasin ang iyong marka sa homework at exams ngayon gamit ang Quizwiz!

A 198-lb patient is to receive a dobutamine infusion at 5 mcg/kg/minute. The label on the infusion bag states: dobutamine 250 mg in 250 mL normal saline. When setting the infusion pump, the nurse will set the infusion rate at how many mL per hour?

27 In order to administer the dobutamine at the prescribed rate of 5 mcg/kg/minute from a concentration of 250 mg in 250 mL, the nurse will need to infuse 27 mL/hour.

New-onset atrial fibrillation can be serious for which of the following reasons? a. It increases the risk of stroke and pulmonary embolism from atrial clots. b. It increases the patient's risk of deep venous thrombosis. c. It may increase cardiac output to dangerous levels. d. It indicates that the patient is about to have an MI.

a. It increases the risk of stroke and pulmonary embolism from atrial clots. In atrial fibrillation the atria do not contract normally; they quiver. This increases the chance of the blood clotting in the atria because of a lack of complete emptying of the atria. These clots can break free and cause embolic strokes and pulmonary emboli. Atrial fibrillation does not indicate impending myocardial infarction or an increased risk of deep venous thrombosis. Atrial fibrillation decreases cardiac output from the loss of atrial kick.

Which of the following describe(s) S1, the first heart sound? (Select all that apply.) a. It is associated with closure of the mitral and tricuspid valves. b. It is a high-pitched sound. c. It can be heard most clearly with the diaphragm of the stethoscope. d. The best listening point is in the aortic area. e. The "split" sound can best be detected in the tricuspid area.

a. It is associated with closure of the mitral and tricuspid valves. b. It is a high-pitched sound. c. It can be heard most clearly with the diaphragm of the stethoscope. e. The "split" sound can best be detected in the tricuspid area. S1 is the sound associated with mitral and tricuspid valve closure and is heard most clearly in the mitral and tricuspid areas. S sounds are high pitched and heard best with the diaphragm of the stethoscope.

A patient who is receiving positive pressure ventilation is scheduled for a spontaneous breathing trial (SBT). Which finding by the nurse is most important to discuss with the health care provider before starting the SBT? a. New ST segment elevation is noted on the cardiac monitor. b. Enteral feedings are being given through an orogastric tube. c. Scattered rhonchi are heard when auscultating breath sounds. d. HYDROmorphone (Dilaudid) is being used to treat postoperative pain.

a. New ST segment elevation is noted on the cardiac monitor. Myocardial ischemia is a contraindication for ventilator weaning. The ST segment elevation is an indication that weaning should be postponed until further investigation and/or treatment for myocardial ischemia can be done. The other information will also be shared with the health care provider, but ventilator weaning can proceed when opioids are used for pain management, abnormal lung sounds are present, or enteral feedings are being used.

During change-of-shift report, the nurse is told that a patient has been admitted with dehydration and hypotension after having vomiting and diarrhea for 4 days. Which finding is most important for the nurse to report to the health care provider? a. New onset of confusion b. Heart rate 112 beats/minute c. Decreased bowel sounds d. Pale, cool, and dry extremities

a. New onset of confusion The changes in mental status are indicative that the patient is in the progressive stage of shock and that rapid intervention is needed to prevent further deterioration. The other information is consistent with compensatory shock.

The nurse is caring for a patient immediately after repair of an abdominal aortic aneurysm. On assessment, the patient has absent popliteal, posterior tibial, and dorsalis pedis pulses. The legs are cool and mottled. Which action should the nurse take first? a. Notify the surgeon and anesthesiologist. b. Wrap both the legs in a warming blanket. c. Document the findings and recheck in 15 minutes. d. Compare findings to the preoperative assessment of the pulses.

a. Notify the surgeon and anesthesiologist. Lower extremity pulses may be absent for a short time after surgery because of vasospasm and hypothermia. Decreased or absent pulses together with a cool and mottled extremity may indicate embolization or graft occlusion. These findings should be reported to the physician immediately because this is an emergency situation. Because pulses are marked prior to surgery, the nurse would know whether pulses were present prior to surgery before notifying the health care providers about the absent pulses. Because the patient's symptoms may indicate graft occlusion or multiple emboli and a possible need to return to surgery, it is not appropriate to wait 15 minutes before taking action. A warming blanket will not improve the circulation to the patient's legs.

An 80-year-old patient with a history of an abdominal aortic aneurysm arrives at the emergency department (ED) with severe back pain and absent pedal pulses. Which actions should the nurse take first? a. Obtain the blood pressure. b. Obtain blood for laboratory testing. c. Assess for the presence of an abdominal bruit. d. Determine any family history of kidney disease.

a. Obtain the blood pressure. Because the patient appears to be experiencing aortic dissection, the nurse's first action should be to determine the hemodynamic status by assessing blood pressure. The other actions also may be done, but they will not provide information that will determine what interventions are needed immediately for this patient.

To determine whether there is a delay in impulse conduction through the atria, the nurse will measure the duration of the patient's a. P wave. b. Q wave. c. P-R interval. d. QRS complex.

a. P wave. The P wave represents the depolarization of the atria. The P-R interval represents depolarization of the atria, atrioventricular (AV) node, bundle of His, bundle branches, and the Purkinje fibers. The QRS represents ventricular depolarization. The Q wave is the first negative deflection following the P wave and should be narrow and short.

A patient is being monitored by continuous ECG after placement of a transvenous pacemaker. "Loss of capture" is seen on the ECG. Which nursing intervention may correct this situation? a. Position the patient on the left side. b. Decrease the milliamperes as ordered. c. Increase the rate as ordered. d. Monitor the patient in a different lead.

a. Position the patient on the left side. "Loss of capture" most often can be attributed either to displacement of the pacing electrode or to an increase in threshold as a result of drugs, metabolic disorders, electrolyte imbalances, or fibrosis or myocardial ischemia at the site of electrode placement. In many cases, increasing the output (mA) may elicit capture. For transvenous leads, repositioning the patient to the left side may improve lead contact and restore capture.

The physician anticipates the CVC dwelling time to be 10 to 20 days. The nurse anticipates that the CVC will be placed in the a. SC vein. b. IJ vein. c. EJ vein. d. femoral vein.

a. SC vein. If the anticipated central venous catheter (CVC) dwelling time is prolonged more than 5 days, the subcutaneous (SC) site is preferred. The SC position has the lowest infection rate and produces the least patient discomfort from the catheter. The internal jugular (IJ) vein is the most frequently used access site for CVC insertion. Compared with the other thoracic veins, it is the easiest to canalize. If the IJ vein is not available, the external jugular (EJ) vein may be accessed, although blood flow is significantly higher in the IJ vein, making it the preferred site. This may be the reason why catheter-related infections are higher in the IJ than the SC position for indwelling catheters left in place for more than 4 days. The femoral vein is considered the easiest cannulation site because there are no curves in the insertion route. Because there is a higher rate of nosocomial infection with femoral catheters, this site is not recommended.

The nurse hears a murmur between the S1 and S2 heart sounds at the patient's left fifth intercostal space and midclavicular line. How will the nurse record this information? a. Systolic murmur heard at mitral area b. Systolic murmur heard at Erb's point c. Diastolic murmur heard at aortic area d. Diastolic murmur heard at the point of maximal impulse

a. Systolic murmur heard at mitral area The S1 signifies the onset of ventricular systole. S2 signifies the onset of diastole. A murmur occurring between these two sounds is a systolic murmur. The mitral area is the intersection of the left fifth intercostal space and the midclavicular line. The other responses describe murmurs heard at different landmarks on the chest and/or during the diastolic phase of the cardiac cycle.

A patient who is 2 days post-femoral-popliteal bypass graft to the right leg is being cared for on the vascular unit. Which action by a licensed practical/vocational nurse (LPN/LVN) caring for the patient requires the registered nurse (RN) to intervene? a. The LPN/LVN has the patient sit in a chair for 90 minutes. b. The LPN/LVN assists the patient to walk 40 feet in the hallway. c. The LPN/LVN gives the ordered aspirin 160 mg after breakfast. d. The LPN/LVN places the patient in a Fowler's position for meals.

a. The LPN/LVN has the patient sit in a chair for 90 minutes. The patient should avoid sitting for long periods because of the increased stress on the suture line caused by leg edema and because of the risk for venous thromboembolism (VTE). The other actions by the LPN/LVN are appropriate.

A patient with respiratory failure has arterial pressure-based cardiac output (APCO) monitoring and is receiving mechanical ventilation with peak end-expiratory pressure (PEEP) of 12 cm H O. Which information indicates that a change in the ventilator settings may be required? a. The arterial pressure is 90/46. b. The heart rate is 58 beats/minute. c. The stroke volume is increased. d. The stroke volume variation is 12%.

a. The arterial pressure is 90/46. The hypotension suggests that the high intrathoracic pressure caused by the PEEP may be decreasing venous return and (potentially) cardiac output. The other assessment data would not be a direct result of PEEP and mechanical ventilation.

Which intervention by a new nurse who is caring for a patient who has just had an implantable cardioverterdefibrillator (ICD) inserted indicates a need for more education about care of patients with ICDs? a. The nurse assists the patient to do active range of motion exercises for all extremities. b. The nurse assists the patient to fill out the application for obtaining a Medic Alert ID. c. The nurse gives amiodarone (Cordarone) to the patient without first consulting with the health care provider. d. The nurse teaches the patient that sexual activity usually can be resumed once the surgical incision is healed.

a. The nurse assists the patient to do active range of motion exercises for all extremities. The patient should avoid moving the arm on the ICD insertion site until healing has occurred in order to prevent displacement of the ICD leads. The other actions by the new nurse are appropriate for this patient.

The nurse is caring for a 78-year-old patient with aortic stenosis. Which assessment data obtained by the nurse would be most important to report to the health care provider? a. The patient complains of chest pressure when ambulating. b. A loud systolic murmur is heard along the right sternal border. c. A thrill is palpated at the second intercostal space, right sternal border. d. The point of maximum impulse (PMI) is at the left midclavicular line.

a. The patient complains of chest pressure when ambulating. Chest pressure (or pain) occurring with aortic stenosis is caused by cardiac ischemia, and reporting this information would be a priority. A systolic murmur and thrill are expected in a patient with aortic stenosis. A PMI at the left midclavicular line is normal.

The nurse is caring for a 64-year-old patient admitted with mitral valve regurgitation. Which information obtained by the nurse when assessing the patient should be communicated to the health care provider immediately? a. The patient has bilateral crackles. b. The patient has bilateral, 4+ peripheral edema. c. The patient has a loud systolic murmur across the precordium. d. The patient has a palpable thrill felt over the left anterior chest.

a. The patient has bilateral crackles. Crackles that are audible throughout the lungs indicate that the patient is experiencing severe left ventricular failure with pulmonary congestion and needs immediate interventions such as diuretics. A systolic murmur and palpable thrill would be expected in a patient with mitral regurgitation. Although 4+ peripheral edema indicates a need for a change in therapy, it does not need to be addressed urgently.

Which assessment finding obtained by the nurse when caring for a patient with a right radial arterial line indicates a need for the nurse to take immediate action? a. The right hand is cooler than the left hand. b. The mean arterial pressure (MAP) is 77 mm Hg. c. The system is delivering 3 mL of flush solution per hour. d. The flush bag and tubing were last changed 3 days previously.

a. The right hand is cooler than the left hand. The change in temperature of the left hand suggests that blood flow to the left hand is impaired. The flush system needs to be changed every 96 hours. A mean arterial pressure (MAP) of 75 mm Hg is normal. Flush systems for hemodynamic monitoring are set up to deliver 3 to 6 mL/hour of flush solution.

Which preventive actions by the nurse will help limit the development of systemic inflammatory response syndrome (SIRS) in patients admitted to the hospital (select all that apply)? a. Use aseptic technique when caring for invasive lines or devices. b. Ambulate postoperative patients as soon as possible after surgery. c. Remove indwelling urinary catheters as soon as possible after surgery. d. Advocate for parenteral nutrition for patients who cannot take oral feedings. e. Administer prescribed antibiotics within 1 hour for patients with possible sepsis.

a. Use aseptic technique when caring for invasive lines or devices. b. Ambulate postoperative patients as soon as possible after surgery. c. Remove indwelling urinary catheters as soon as possible after surgery. e. Administer prescribed antibiotics within 1 hour for patients with possible sepsis. Because sepsis is the most frequent etiology for SIRS, measures to avoid infection such as removing indwelling urinary catheters as soon as possible, use of aseptic technique, and early ambulation should be included in the plan of care. Adequate nutrition is important in preventing SIRS. Enteral, rather than parenteral, nutrition is preferred when patients are unable to take oral feedings because enteral nutrition helps maintain the integrity of the intestine, thus decreasing infection risk. Antibiotics should be administered within 1 hour after being prescribed to decrease the risk of sepsis progressing to SIRS.

Which lead is best to monitor a patient? a. Varies based on the patient's clinical condition and recent clinical history b. Lead MCL1 c. Lead V1 d. Lead II

a. Varies based on the patient's clinical condition and recent clinical history The selection of an electrocardiographic monitoring lead is not a decision to be made casually or according to habit. The monitoring lead should be chosen with consideration of the patient's clinical condition and recent clinical history. If the monitored heart has a normal electrical axis, lead II displays a waveform that is predominantly upright, with a positive P wave and positive QRS waveform. P waves are usually easy to identify in lead II, and it is recommended for monitoring of atrial dysrhythmias. However, it is difficult to identify right bundle branch block (RBBB) and left bundle branch block (LBBB). Lead V1 is the optimal lead to select if the critical care nurse needs to analyze ventricular ectopy. V1 provides information to facilitate differentiation between RBBB versus LBBB pattern or distinguish between ventricular tachycardia and supraventricular tachycardia with aberrant conduction, determine whether premature ventricular contractions originate in the right or left ventricle, and clarify when ST segment changes are caused by the RBBB and when they are the result of ischemia. Lead V1 is excellent for this purpose. MCL1 is an uncommon lead choice today. It is used only if monitoring with a three-lead system such as on a transport monitor.

A sudden increase in left atrial pressure, acute pulmonary edema, and low cardiac output, caused by the ventricle contracting during systole, are all characteristics of a. acute mitral regurgitation. b. aortic insufficiency. c. chronic mitral regurgitation. d. pericardial friction rub.

a. acute mitral regurgitation. Acute mitral regurgitation occurs when the ventricle contracts during systole and a jet of blood is sent in a retrograde manner to the left atrium, causing a sudden increase in left atrial pressure, acute pulmonary edema, and low CO and leading to cardiogenic shock. Chronic mitral regurgitation is auscultated in the mitral area and occurs during systole. It is high pitched and blowing, although the pitch and intensity vary, depending on the degree of regurgitation. As mitral regurgitation progresses, the murmur radiates more widely. Aortic insufficiency is an incompetent aortic valve. If the valve cusps do not maintain this seal, the sound of blood flowing back into the left ventricle during diastole is heard as a decrescendo, high-pitched, blowing murmur. A pericardial friction rub is a sound that can occur within 2 to 7 days after a myocardial infarction. The friction rub results from pericardial inflammation (pericarditis). Classically, a pericardial friction rub is a grating or scratching sound that is both systolic and diastolic, corresponding to cardiac motion within the pericardial sac.

A vasoconstrictor used to treat shock is a. adrenaline. b. Nipride. c. Dobutrex. d. adenosine.

a. adrenaline. Adrenaline is a vasoconstrictor, Nipride is a vasodilator, Dobutrex is an inotrope, and adenosine is an antidysrhythmic.

The main cause of cardiogenic shock is a. an inability of the heart to pump blood forward. b. hypovolemia, resulting in decreased stroke volume. c. disruption of the conduction system when re-entry phenomenon occurs. d. an inability of the heart to respond to inotropic agents.

a. an inability of the heart to pump blood forward. Cardiogenic shock is the result of failure of the heart to effectively pump blood forward. It can occur with dysfunction of the right or the left ventricle or both. The lack of adequate pumping function leads to decreased tissue perfusion and circulatory failure.

The current standard of care for PCI typically includes dual antiplatelet therapy with oral a. aspirin and Clopidogrel (Plavix). b. aspirin and Abciximab (ReoPro) c. aspirin and Eptifibatide (Integrilin) d. aspirin and Tirofiban (Aggrastat)

a. aspirin and Clopidogrel (Plavix). Because platelet activation is a complex process involving multiple pathways, combination therapy with two or more agents has proven most effective. The current standard of care for percutaneous coronary interventiontypically includes dual antiplatelet therapy with aspirin and a thienopyridine. These oral agents are administered before the procedure and continued at discharge. Abciximab, eptifibatide, and tirofiban are all intravenous antiplatelet agents.

To auscultate for S3 or S4 gallops in the mitral area, the nurse listens with the a. bell of the stethoscope with the patient in the left lateral position. b. diaphragm of the stethoscope with the patient in a supine position. c. bell of the stethoscope with the patient sitting and leaning forward. d. diaphragm of the stethoscope with the patient lying flat on the left side.

a. bell of the stethoscope with the patient in the left lateral position. Gallop rhythms generate low-pitched sounds and are most easily heard with the bell of the stethoscope. Sounds associated with the mitral valve are accentuated by turning the patient to the left side, which brings the heart closer to the chest wall. The diaphragm of the stethoscope is best to use for the higher-pitched sounds such as S1 and S2.

Medical management of hypertrophic cardiomyopathy includes a. beta-blockers. b. positive inotropes. c. an intensive exercise regimen. d. aortic valve replacement.

a. beta-blockers. Pharmacologic management includes beta-blockers to decrease left ventricular workload, medications to control and prevent atrial and ventricular dysrhythmias, anticoagulation if atrial fibrillation or left ventricular thrombi are present, and finally drugs to manage heart failure.

The Allen test is used before radial arterial line placement to assess a. collateral circulation to the hand. b. patency of the radial artery. c. neurologic function of the hand. d. pain sensation at the insertion point.

a. collateral circulation to the hand. The Allen test involves occluding the radial or ulnar artery after blanching the hand. If the hand turns pink, then the non-occluded artery provides enough circulation to the hand. If the hand remains blanched, then no collateral circulation exists, and that wrist should not be used for arterial line placement.

Clinical manifestations of right-sided heart failure include a. elevated central venous pressure and sacral edema. b. pulmonary congestion and jugular venous distention. c. hypertension and chest pain. d. liver tenderness and pulmonary edema.

a. elevated central venous pressure and sacral edema. The common manifestations of right ventricular failure are the following: jugular venous distention, elevated central venous pressure, weakness, peripheral or sacral edema, hepatomegaly (enlarged liver), jaundice, and liver tenderness. Gastrointestinal symptoms include poor appetite, anorexia, nausea, and an uncomfortable feeling of fullness.

The patient is 12 hours postoperative for a CABG. The patient's vital signs include: T 103° F, HR 112, RR 22, BP 134/78 mm Hg, and O2 sat 94% on 3L/NC. The nurse suspects that the patient has developed a. infection and notifies the physician immediately. b. infection, which is common postoperatively, and monitors the patient's condition. c. cardiac tamponade and notifies the physician immediately. d. delirium caused by the elevated temperature.

a. infection and notifies the physician immediately. Postoperative fever is fairly common after cardiopulmonary bypass. However, persistent temperature elevation to greater than 101° F (38.3° C) must be investigated. Sternal wound infections and infective endocarditis are the most devastating infectious complications, but leg wound infections, pneumonia, and urinary tract infections also can occur. A potentially lethal complication, cardiac tamponade may occur after surgery if blood accumulates in the mediastinal space, impairing the heart's ability to pump. Signs of tamponade include elevated and equalized filling pressures (e.g., central venous pressure, pulmonary artery diastolic pressure, pulmonary artery occlusion pressure), decreased cardiac output, decreased blood pressure, jugular venous distention, pulsus paradoxus, muffled heart sounds, sudden cessation of chest tube drainage, and a widened cardiac silhouette on radiographs. The risk of delirium is increased in cardiac surgery patients, especially elderly patients, and is associated with increased mortality rates and reduced quality of life and cognitive function. Nursing staff can play a critical role in the prevention and recognition of delirium.

The nursing measure that can best enhance large volumes of fluid replacement in hypovolemic shock is a. insertion of a large-diameter peripheral intravenous catheter. b. positioning the patient in the Trendelenburg position. c. forcing at least 240 mL of fluid each hour. d. administering intravenous lines under pressure.

a. insertion of a large-diameter peripheral intravenous catheter. Measures to facilitate the administration of volume replacement include insertion of large-bore peripheral intravenous catheters; rapid administration of prescribed fluids; and positioning the patient with the legs elevated, trunk flat, and head and shoulders above the chest.

An assessment finding of pulsus alternans may indicate evidence of a. left-sided heart failure. b. jugular venous distention. c. pulmonary embolism. d. myocardial ischemia.

a. left-sided heart failure. Pulsus alternans describes a regular pattern of pulse amplitude changes that alternate between stronger and weaker beats. This finding is suggestive of end-stage left ventricular heart failure.

The nurse suspects cardiac tamponade in a patient who has acute pericarditis. To assess for the presence of pulsus paradoxus, the nurse should a. note when Korotkoff sounds are auscultated during both inspiration and expiration. b. subtract the diastolic blood pressure (DBP) from the systolic blood pressure (SBP). c. check the electrocardiogram (ECG) for variations in rate during the respiratory cycle. d. listen for a pericardial friction rub that persists when the patient is instructed to stop breathing.

a. note when Korotkoff sounds are auscultated during both inspiration and expiration. Pulsus paradoxus exists when there is a gap of greater than 10 mm Hg between when Korotkoff sounds can be heard during only expiration and when they can be heard throughout the respiratory cycle. The other methods described would not be useful in determining the presence of pulsus paradoxus.

While caring for a patient with aortic stenosis, the nurse identifies a nursing diagnosis of acute pain related to decreased coronary blood flow. A priority nursing intervention for this patient would be to a. promote rest to decrease myocardial oxygen demand. b. teach the patient about the need for anticoagulant therapy. c. teach the patient to use sublingual nitroglycerin for chest pain. d. raise the head of the bed 60 degrees to decrease venous return.

a. promote rest to decrease myocardial oxygen demand. Rest is recommended to balance myocardial oxygen supply and demand and to decrease chest pain. The patient with aortic stenosis requires higher preload to maintain cardiac output, so nitroglycerin and measures to decrease venous return are contraindicated. Anticoagulation is not recommended unless the patient has atrial fibrillation.

In caring for a postoperative cardiovascular patient, the nurse knows that the most frequent cause of a decreased cardiac output is a. reduced preload. b. increased afterload. c. increased contractility. d. bradycardia.

a. reduced preload. In most patients, reduced preload is the cause of low postoperative cardiac output. To enhance preload, volume may be administered in the form of crystalloid, colloid, or packed red blood cells.

One differentiating factor between stable angina and unstable angina is that stable angina a. responds predictably well to nitrates. b. is not precipitated by activity. c. has a low correlation to CAD. d. is a result of coronary artery spasm.

a. responds predictably well to nitrates. Pain control is achieved by rest and by sublingual nitroglycerin within 5 minutes. Stable angina is the result of fixed lesions (blockages) of more than 75% of the coronary artery lumen. Ischemia and chest pain occur when myocardial demand from exertion exceeds the fixed blood oxygen supply.

The abdominojugular reflux test determines the presence of a. right ventricular failure. b. hypoxemia. c. liver failure. d. pitting edema.

a. right ventricular failure. The abdominojugular reflux sign can assist with the diagnosis of right ventricular failure. A positive abdominojugular reflux sign is an increase in the jugular venous pressure (CVP equivalent) of greater than 3 cm sustained for at least 15 seconds.

The nurse is caring for a patient who was admitted to the coronary care unit following an acute myocardial infarction (AMI) and percutaneous coronary intervention the previous day. Teaching for this patient would include a. when cardiac rehabilitation will begin. b. the typical emotional responses to AMI. c. information regarding discharge medications. d. the pathophysiology of coronary artery disease.

a. when cardiac rehabilitation will begin. Early after an AMI, the patient will want to know when resumption of usual activities can be expected. At this time, the patient's anxiety level or denial will interfere with good understanding of complex information such as the pathophysiology of coronary artery disease (CAD). Teaching about discharge medications should be done closer to discharge. The nurse should support the patient by decreasing anxiety rather than discussing the typical emotional responses to myocardial infarction (MI).

When evaluating the effectiveness of preoperative teaching with a patient scheduled for coronary artery bypass graft (CABG) surgery using the internal mammary artery, the nurse determines that additional teaching is needed when the patient says which of the following? a. "They will circulate my blood with a machine during the surgery." b. "I will have small incisions in my leg where they will remove the vein." c. "They will use an artery near my heart to go around the area that is blocked." d. "I will need to take an aspirin every day after the surgery to keep the graft open."

b. "I will have small incisions in my leg where they will remove the vein." When the internal mammary artery is used there is no need to have a saphenous vein removed from the leg. The other statements by the patient are accurate and indicate that the teaching has been effective.

After the nurse teaches the patient about the use of carvedilol (Coreg) in preventing anginal episodes, which statement by a patient indicates that the teaching has been effective? a. "Carvedilol will help my heart muscle work harder." b. "It is important not to suddenly stop taking the carvedilol." c. "I can expect to feel short of breath when taking carvedilol." d. "Carvedilol will increase the blood flow to my heart muscle."

b. "It is important not to suddenly stop taking the carvedilol." Patients who have been taking b-adrenergic blockers can develop intense and frequent angina if the medication is suddenly discontinued. Carvedilol (Coreg) decreases myocardial contractility. Shortness of breath that occurs when taking b-adrenergic blockers for angina may be due to bronchospasm and should be reported to the health care provider. Carvedilol works by decreasing myocardial oxygen demand, not by increasing blood flow to the coronary arteries.

When developing a teaching plan for a 76-year-old patient newly diagnosed with peripheral artery disease (PAD), which instructions should the nurse include? a. "Exercise only if you do not experience any pain." b. "It is very important that you stop smoking cigarettes." c. "Try to keep your legs elevated whenever you are sitting." d. "Put elastic compression stockings on early in the morning."

b. "It is very important that you stop smoking cigarettes." Smoking cessation is essential for slowing the progression of PAD to critical limb ischemia and reducing the risk of myocardial infarction and death. Circulation to the legs will decrease if the legs are elevated. Patients with PAD are taught to exercise to the point of feeling pain, rest, and then resume walking. Support hose are not used for patients with PAD.

A patient who is recovering from an acute myocardial infarction (AMI) asks the nurse about when sexual intercourse can be resumed. Which response by the nurse is best? a. "Most patients are able to enjoy intercourse without any complications." b. "Sexual activity uses about as much energy as climbing two flights of stairs." c. "The doctor will provide sexual guidelines when your heart is strong enough." d. "Holding and cuddling are good ways to maintain intimacy after a heart attack."

b. "Sexual activity uses about as much energy as climbing two flights of stairs." Sexual activity places about as much physical stress on the cardiovascular system as most moderate-energy activities such as climbing two flights of stairs. The other responses do not directly address the patient's question or may not be accurate for this patient.

The nurse assesses the dorsalis pedis and posterior tibial pulses as weak and thready. Indicate the correct documentation for the pulse volume that the nurse would use. a. 0 b. 1+ c. 2+ d. 3+

b. 1+ Pulse volumes are 0, not palpable; 1+, faintly palpable (weak and thready); 2+, palpable (normal pulse); and 3+, bounding (hyperdynamic pulse).

Which of the following doses of dopamine (Intropin) results in stimulation of beta receptors and increased myocardial contractility? a. 1 mcg/kg/min b. 5 mcg/kg/min c. 15 mcg/kg/min d. 20 mcg/kg/min

b. 5 mcg/kg/min At low dosages of 1 to 2 mcg/kg/min, dopamine stimulates dopaminergic receptors, causing renal and mesenteric vasodilation. Moderate dosages result in stimulation of beta receptors to increase myocardial contractility and improve cardiac output. At dosages greater than 10 mg/kg/min, dopamine predominantly stimulates alpha receptors, resulting in vasoconstriction that often negates both the beta-adrenergic and dopaminergic effects.

When planning care for a patient hospitalized with a streptococcal infective endocarditis (IE), which intervention is a priority for the nurse to include? a. Monitor labs for streptococcal antibodies. b. Arrange for placement of a long-term IV catheter. c. Teach the importance of completing all oral antibiotics. d. Encourage the patient to begin regular aerobic exercise.

b. Arrange for placement of a long-term IV catheter. Treatment for IE involves 4 to 6 weeks of IV antibiotic therapy in order to eradicate the bacteria, which will require a long-term IV catheter such as a peripherally inserted central catheter (PICC) line. Rest periods and limiting physical activity to a moderate level are recommended during the treatment for IE. Oral antibiotics are not effective in eradicating the infective bacteria that cause IE. Blood cultures, rather than antibody levels, are used to monitor the effectiveness of antibiotic therapy.

When titrating IV nitroglycerin (Tridil) for a patient with a myocardial infarction (MI), which action will the nurse take to evaluate the effectiveness of the medication? a. Monitor heart rate. b. Ask about chest pain. c. Check blood pressure. d. Observe for dysrhythmias.

b. Ask about chest pain. The goal of IV nitroglycerin administration in MI is relief of chest pain by improving the balance between myocardial oxygen supply and demand. The nurse also will monitor heart rate and blood pressure (BP) and observe for dysrhythmias, but these parameters will not indicate whether the medication is effective.

While family members are visiting, a patient has a respiratory arrest and is being resuscitated. Which action by the nurse is best? a. Tell the family members that watching the resuscitation will be very stressful. b. Ask family members if they wish to remain in the room during the resuscitation. c. Take the family members quickly out of the patient room and remain with them. d. Assign a staff member to wait with family members just outside the patient room.

b. Ask family members if they wish to remain in the room during the resuscitation. Research indicates that family members want the option of remaining in the room during procedures such as cardiopulmonary resuscitation (CPR) and that this decreases anxiety and facilitates grieving. The other options may be appropriate if the family decides not to remain with the patient.

A 19-year-old student comes to the student health center at the end of the semester complaining that, "My heart is skipping beats." An electrocardiogram (ECG) shows occasional premature ventricular contractions (PVCs). What action should the nurse take next? a. Start supplemental O at 2 to 3 L/min via nasal cannula. b. Ask the patient about current stress level and caffeine use. c. Ask the patient about any history of coronary artery disease. d. Have the patient taken to the hospital emergency department (ED).

b. Ask the patient about current stress level and caffeine use. In a patient with a normal heart, occasional PVCs are a benign finding. The timing of the PVCs suggests stress or caffeine as possible etiologic factors. It is unlikely that the patient has coronary artery disease, and this should not be the first question the nurse asks. The patient is hemodynamically stable, so there is no indication that the patient needs to be seen in the ED or that oxygen needs to be administered.

A 68-year-old patient has been in the intensive care unit for 4 days and has a nursing diagnosis of disturbed sensory perception related to sleep deprivation. Which action should the nurse include in the plan of care? a. Administer prescribed sedatives or opioids at bedtime to promote sleep. b. Cluster nursing activities so that the patient has uninterrupted rest periods. c. Silence the alarms on the cardiac monitors to allow 30- to 40-minute naps. d. Eliminate assessments between 0100 and 0600 to allow uninterrupted sleep.

b. Cluster nursing activities so that the patient has uninterrupted rest periods. Clustering nursing activities and providing uninterrupted rest periods will minimize sleep-cycle disruption. Sedative and opioid medications tend to decrease the amount of rapid eye movement (REM) sleep and can contribute to sleep disturbance and disturbed sensory perception. Silencing the alarms on the cardiac monitors would be unsafe in a critically ill patient, as would discontinuing assessments during the night.

A patient is admitted to the intensive care unit after she develops disseminated intravascular coagulation (DIC) after a vaginal delivery. DIC is known to occur in patients with retained placental fragments. What is the result of DIC? a. Hypersensitive response to an antigen, resulting in anaphylaxis b. Depletion of clotting factors and excessive fibrinolysis, resulting in simultaneous microvascular clotting and hemorrhage c. Vasodilatation, resulting in hypotension d. Septic shock, resulting in vasodilation and decreased perfusion

b. Depletion of clotting factors and excessive fibrinolysis, resulting in simultaneous microvascular clotting and hemorrhage DIC results simultaneously in microvascular clotting and hemorrhage in organ systems, leading to thrombosis and fibrinolysis in life-threatening proportions. Clotting factor derangement leads to further inflammation and further thrombosis. Microvascular damage leads to further organ injury. Cell injury and damage to the endothelium activate the intrinsic or extrinsic coagulation pathways.

When assessing a newly admitted patient, the nurse notes a murmur along the left sternal border. To document more information about the murmur, which action will the nurse take next? a. Find the point of maximal impulse. b. Determine the timing of the murmur. c. Compare the apical and radial pulse rates. d. Palpate the quality of the peripheral pulses.

b. Determine the timing of the murmur. Murmurs are caused by turbulent blood flow, such as occurs when blood flows through a damaged valve. Relevant information includes the position in which the murmur is heard best (e.g., sitting and leaning forward), the timing of the murmur in relation to the cardiac cycle (e.g., systole, diastole), and where on the thorax the murmur is heard best. The other information is also important in the cardiac assessment but will not provide information that is relevant to the murmur.

To improve the physical activity level for a mildly obese 71-year-old patient, which action should the nurse plan to take? a. Stress that weight loss is a major benefit of increased exercise. b. Determine what kind of physical activities the patient usually enjoys. c. Tell the patient that older adults should exercise for no more than 20 minutes at a time. d. Teach the patient to include a short warm-up period at the beginning of physical activity

b. Determine what kind of physical activities the patient usually enjoys. Because patients are more likely to continue physical activities that they already enjoy, the nurse will plan to ask the patient about preferred activities. The goal for older adults is 30 minutes of moderate activity on most days. Older adults should plan for a longer warm-up period. Benefits of exercises, such as improved activity tolerance, should be emphasized rather than aiming for significant weight loss in older mildly obese adults.

Which of the following mechanisms is responsible for the augmentation of coronary arterial blood flow and increased myocardial oxygen supply seen with the intra-aortic balloon pump? a. The vacuum created in the aorta as a result of balloon deflation b. Diastolic inflation with retrograde perfusion c. Forward flow to the peripheral circulation d. Inflation during systole to augment blood pressure

b. Diastolic inflation with retrograde perfusion The blood volume in the aorta below the level of the balloon is propelled forward toward the peripheral vascular system, which may enhance renal perfusion. Subsequently, the deflation of the balloon just before the opening of the aortic valve creates a potential space or vacuum in the aorta, toward which blood flows unimpeded during ventricular ejection. This decreased resistance to left ventricular ejection, or decreased afterload, facilitates ventricular emptying and reduces myocardial oxygen demands.

A patient with a potassium level of 2.8 mEq/L is given 60 mEq over a 12-hour period, and a repeat potassium level is obtained after the bolus. The current potassium level is 2.9 mEq/L. Which of the following should now be considered? a. Stopping the patient's Aldactone b. Drawing a serum magnesium level c. Rechecking the potassium level d. Monitoring the patient's urinary output

b. Drawing a serum magnesium level A total serum magnesium concentration below 1.5 mEq/L defineshypomagnesemia. It is commonly associated with other electrolyte imbalances, most notably alterations in potassium, calcium, and phosphorus. Low serum magnesium levels can result from many causes.

Which action should the nurse perform when preparing a patient with supraventricular tachycardia for cardioversion who is alert and has a blood pressure of 110/66 mm Hg? a. Turn the synchronizer switch to the "off" position. b. Give a sedative before cardioversion is implemented. c. Set the defibrillator/cardioverter energy to 360 joules. d. Provide assisted ventilations with a bag-valve-mask device.

b. Give a sedative before cardioversion is implemented. When a patient has a nonemergency cardioversion, sedation is used just before the procedure. The synchronizer switch is turned "on" for cardioversion. The initial level of joules for cardioversion is low (e.g., 50). Assisted ventilations are not indicated for this patient.

The following interventions are ordered by the health care provider for a patient who has respiratory distress and syncope after eating strawberries. Which will the nurse complete first? a. Start a normal saline infusion. b. Give epinephrine (Adrenalin). c. Start continuous ECG monitoring. d. Give diphenhydramine (Benadryl).

b. Give epinephrine (Adrenalin). Epinephrine rapidly causes peripheral vasoconstriction, dilates the bronchi, and blocks the effects of histamine and reverses the vasodilation, bronchoconstriction, and histamine release that cause the symptoms of anaphylaxis. The other interventions are also appropriate but would not be the first ones completed.

Which of the following criteria are representative of the patient in normal sinus rhythm? a. Heart rate, 64 beats/min; rhythm regular; PR interval, 0.10 second; QRS, 0.04 second b. Heart rate, 88 beats/min; rhythm regular; PR interval, 0.18 second; QRS, 0.06 second c. Heart rate, 54 beats/min; rhythm regular; PR interval, 0.16 second; QRS, 0.08 second d. Heart rate, 92 beats/min; rhythm irregular; PR interval, 0.16 second; QRS, 0.04 second

b. Heart rate, 88 beats/min; rhythm regular; PR interval, 0.18 second; QRS, 0.06 second The parameters for normal sinus rhythm are heart rate, 60 to 100 beats/min; rhythm, regular; PR interval, 0.12 to 0.20 second; and QRS, 0.06 to 0.10 second.

Nursing interventions after angioplasty would include which of the following? (Select all that apply.) a. Elevating the head of the bed to 45 degrees b. Hydration as a renal protection measure c. Assessing pedal pulses on the involved limb every 15 minutes for the first 2 hours after the procedure d. Monitoring the vascular hemostatic device for signs of bleeding e. Educating the patient on the necessity of staying supine for 1 to 2 hours after the procedure

b. Hydration as a renal protection measure c. Assessing pedal pulses on the involved limb every 15 minutes for the first 2 hours after the procedure d. Monitoring the vascular hemostatic device for signs of bleeding The head of the bed must not be elevated more than 30 degrees, and the patient should be instructed to keep the affected leg straight. Bed rest is 6 to 8 hours in duration unless a vascular hemostatic device is used. The nurse observes the patient for bleeding or swelling at the puncture site and frequently assesses adequacy of circulation to the involved extremity.

A patient has ST segment changes that support an acute inferior wall myocardial infarction. Which lead would be best for monitoring the patient? a. I b. II c. V2 d. V6

b. II Leads II, III, and AVF reflect the inferior area of the heart and the ST segment changes. Lead II will best capture any electrocardiographic (ECG) changes that indicate further damage to the myocardium. The other leads do not reflect the inferior part of the myocardial wall and will not provide data about further ischemic changes in that area.

What is the initial intervention in a patient with sinus tachycardia with the following vital signs: HR, 136 beats/min; BP, 102/60 mm Hg; RR, 24 breaths/min; T, 99.2° F; SpO2, 94% on oxygen 2 L/min by nasal cannula? a. Stat adenosine to decrease heart rate b. Identification and correction of the cause of the increased heart rate c. Sublingual nitroglycerine 0.4 mg d. Lidocaine 75 mg IV push

b. Identification and correction of the cause of the increased heart rate Sinus tachycardia can be caused by a wide variety of factors, such as exercise, emotion, pain, fever, hemorrhage, shock, heart failure, and thyrotoxicosis. Illegal stimulant drugs such as cocaine, "ecstasy," and amphetamines can raise the resting heart rate significantly. Many medications used in critical care can also cause sinus tachycardia; common culprits are aminophylline, dopamine, hydralazine, atropine, and catecholamines such as epinephrine. This patient has a stable heart rate and SpO ; therefore, there is time to identify the cause of the sinus tachycardia. Lidocaine is indicated for ventricular dysrhythmias. Nitroglycerine is not indicated because the patient is not having chest pain at this time. Adenosine is usually not indicated unless the heart rate is greater than 150 beats/min

Following surgery for an abdominal aortic aneurysm, a patient's central venous pressure (CVP) monitor indicates low pressures. Which action is a priority for the nurse to take? a. Administer IV diuretic medications. b. Increase the IV fluid infusion per protocol. c. Document the CVP and continue to monitor. d. Elevate the head of the patient's bed to 45 degrees.

b. Increase the IV fluid infusion per protocol. A low CVP indicates hypovolemia and a need for an increase in the infusion rate. Diuretic administration will contribute to hypovolemia and elevation of the head may decrease cerebral perfusion. Documentation and continued monitoring is an inadequate response to the low CVP.

The nurse is assessing a patient with myocarditis before administering the scheduled dose of digoxin (Lanoxin). Which finding is most important for the nurse to communicate to the health care provider? a. Leukocytosis b. Irregular pulse c. Generalized myalgia d. Complaint of fatigue

b. Irregular pulse Myocarditis predisposes the heart to digoxin-associated dysrhythmias and toxicity. The other findings are common symptoms of myocarditis and there is no urgent need to report these.

A patient returns from the cardiac catheterization laboratory after angioplasty and stent placement (ECG changes had indicated an inferior wall myocardial infarction in progress). Which lead would best monitor this patient? a. Varies based on the patient's clinical condition and recent clinical history b. Lead MCL1 c. Lead V1 d. Lead II

b. Lead MCL1 If the monitored heart has a normal electrical axis, lead II displays a waveform that is predominantly upright, with a positive P wave and positive QRS waveform. P waves are usually easy to identify in lead II, and it is recommended for monitoring of atrial dysrhythmias. However, it is difficult to identify right bundle branch block (RBBB) and left bundle branch block (LBBB). The selection of an electrocardiographic monitoring lead is not a decision to be made casually or according to habit. The monitoring lead should be chosen with consideration of the patient's clinical condition and recent clinical history. Lead V1 is the optimal lead to select if the critical care nurse needs to analyze ventricular ectopy. V1 provides information to facilitate differentiation between RBBB versus LBBB pattern or distinguish between ventricular tachycardia and supraventricular tachycardia with aberrant conduction; determine whether premature ventricular contractions originate in the right or left ventricle, and clarify when ST segment changes are caused by the RBBB and when they are the result of ischemia. Lead V1 is excellent for this purpose. MCL1 is an uncommon lead choice today. It is used only if monitoring with a three-lead system such as on a transport monitor.

A patient suddenly develops a wide QRS complex tachycardia. The patient's heart rate is 220 beats/min and regular; blood pressure is 96/40 mm Hg; and respiratory rate is 22 breaths/min, and the patient is awake without complaint except for palpitations. Which of the following interventions would be best to try first? a. Adenosine 6 mg rapid IV push b. Lidocaine 1 mg/kg IV push c. Verapamil 0.5 mg IV push d. Digoxin 0.25 mg IV push

b. Lidocaine 1 mg/kg IV push Because ventricular tachycardia is more dangerous, treatment with lidocaine would be the first intervention. If this rhythm is ventricular tachycardia, lidocaine is the drug of choice. Adenosine and digoxin would have no effect on ventricular tachycardia, and verapamil could cause intractable hypotension. If the rhythm is a supraventricular contraction with aberrant conduction, then lidocaine will do no harm.

Which of the following expresses the correct order when working with an invasive pressure monitor? a. Level the transducer, locate the phlebostatic axis, zero the transducer, and take the reading. b. Locate the phlebostatic axis, level the transducer, zero the transducer, and take the reading. c. Take the reading, level the transducer, locate the phlebostatic axis, and zero the transducer. d. Locate the phlebostatic axis, zero the transducer, level the transducer, and take the reading.

b. Locate the phlebostatic axis, level the transducer, zero the transducer, and take the reading. The correct order is locate the phlebostatic axis, level the transducer, zero the transducer, and take the reading. The transducer cannot be zeroed before it is leveled. Readings cannot be taken before the transducer is zeroed, and leveling the transducer cannot occur until the phlebostatic axis has been identified.

Which admission order written by the health care provider for a patient admitted with infective endocarditis (IE) and a fever would be a priority for the nurse to implement? a. Administer ceftriaxone (Rocephin) 1 g IV. b. Order blood cultures drawn from two sites. c. Give acetaminophen (Tylenol) PRN for fever. d. Arrange for a transesophageal echocardiogram.

b. Order blood cultures drawn from two sites. Treatment of the IE with antibiotics should be started as quickly as possible, but it is essential to obtain blood cultures before initiating antibiotic therapy to obtain accurate sensitivity results. The echocardiogram and acetaminophen administration also should be implemented rapidly, but the blood cultures (and then administration of the antibiotic) have the highest priority.

Which assessment finding by the nurse caring for a patient who has had coronary artery bypass grafting using a right radial artery graft is most important to communicate to the health care provider? a. Complaints of incisional chest pain b. Pallor and weakness of the right hand c. Fine crackles heard at both lung bases d. Redness on both sides of the sternal incision

b. Pallor and weakness of the right hand The changes in the right hand indicate compromised blood flow, which requires immediate evaluation and actions such as prescribed calcium channel blockers or surgery. The other changes are expected and/or require nursing interventions.

After receiving the following information about four patients during change-of-shift report, which patient should the nurse assess first? a. Patient with acute pericarditis who has a pericardial friction rub b. Patient who has just returned to the unit after balloon valvuloplasty c. Patient who has hypertrophic cardiomyopathy and a heart rate of 116 d. Patient with a mitral valve replacement who has an anticoagulant scheduled

b. Patient who has just returned to the unit after balloon valvuloplasty The patient who has just arrived after balloon valvuloplasty will need assessment for complications such as bleeding and hypotension. The information about the other patients is consistent with their diagnoses and does not indicate any complications or need for urgent assessment or intervention.

After receiving report on the following patients, which patient should the nurse assess first? a. Patient with rheumatic fever who has sharp chest pain with a deep breath b. Patient with acute aortic regurgitation whose blood pressure is 86/54 mm Hg c. Patient with infective endocarditis who has a murmur and splinter hemorrhages d. Patient with dilated cardiomyopathy who has bilateral crackles at the lung bases

b. Patient with acute aortic regurgitation whose blood pressure is 86/54 mm Hg Hypotension in patients with acute aortic regurgitation may indicate cardiogenic shock. The nurse should immediately assess this patient for other findings such as dyspnea or chest pain. The findings in the other patients are typical of their diagnoses and do not indicate a need for urgent assessment and intervention.

Which patient at the cardiovascular clinic requires the most immediate action by the nurse? a. Patient with type 2 diabetes whose current blood glucose level is 145 mg/dL b. Patient with stable angina whose chest pain has recently increased in frequency c. Patient with familial hypercholesterolemia and a total cholesterol of 465mg/dL d. Patient with chronic hypertension whose blood pressure today is 172/98 mmHg

b. Patient with stable angina whose chest pain has recently increased in frequency The history of more frequent chest pain suggests that the patient may have unstable angina, which is part of the acute coronary syndrome spectrum. This will require rapid implementation of actions such as cardiac catheterization and possible percutaneous coronary intervention. The data about the other patients suggest that their conditions are stable.

After change-of-shift report in the progressive care unit, who should the nurse care for first? a. Patient who had an inferior myocardial infarction 2 days ago and has crackles in the lung bases b. Patient with suspected urosepsis who has new orders for urine and blood cultures and antibiotics c. Patient who had a T5 spinal cord injury 1 week ago and currently has a heart rate of 54 beats/minute d. Patient admitted with anaphylaxis 3 hours ago who now has clear lung sounds and a blood pressure of 108/58 mm Hg

b. Patient with suspected urosepsis who has new orders for urine and blood cultures and antibiotics Antibiotics should be administered within the first hour for patients who have sepsis or suspected sepsis in order to prevent progression to systemic inflammatory response syndrome (SIRS) and septic shock. The data on the other patients indicate that they are more stable. Crackles heard only at the lung bases do not require immediate intervention in a patient who has had a myocardial infarction. Mild bradycardia does not usually require atropine in patients who have a spinal cord injury. The findings for the patient admitted with anaphylaxis indicate resolution of bronchospasm and hypotension.

A transesophageal echocardiogram (TEE) is ordered for a patient with possible endocarditis. Which action included in the standard TEE orders will the nurse need to accomplish first? a. Start an IV line. b. Place the patient on NPO status. c. Administer O per nasal cannula. d. Give lorazepam (Ativan) 1 mg IV.

b. Place the patient on NPO status. The patient will need to be NPO for 6 hours preceding the TEE, so the nurse should place the patient on NPO status as soon as the order is received. The other actions also will need to be accomplished but not until just before or during the procedure.

The nurse and unlicensed assistive personnel (UAP) on the telemetry unit are caring for four patients. Which nursing action can be delegated to the UAP? a. Teaching a patient scheduled for exercise electrocardiography about the procedure b. Placing electrodes in the correct position for a patient who is to receive ECG monitoring c. Checking the catheter insertion site for a patient who is recovering from a coronary angiogram d. Monitoring a patient who has just returned to the unit after a transesophageal echocardiogram

b. Placing electrodes in the correct position for a patient who is to receive ECG monitoring UAP can be educated in standardized lead placement for ECG monitoring. Assessment of patients who have had procedures where airway maintenance (transesophageal echocardiography) or bleeding (coronary angiogram) is a concern must be done by the registered nurse (RN). Patient teaching requires RN level education and scope of practice.

The patient's admitting 12-lead ECG shows peaked P waves. Which of the following admitting diagnoses could be responsible for this finding? a. Mitral stenosis b. Pulmonary edema c. Ischemia d. Pericarditis

b. Pulmonary edema Tall, peaked P waves occur in right atrial hypertrophy and are referred to as P pulmonale because this condition is often the result of chronic pulmonary disease. Ischemia occurs when the delivery of oxygen to the tissues is insufficient to meet metabolic demand. Cardiac ischemia in an unstable form occurs because of a sudden decrease in supply, such as when the artery is blocked by a thrombus or when coronary artery spasm occurs. If the pulmonary edema is caused by heart failure, sometimes described as hydrostatic pulmonary edema, the fluid may be in a "bat-wing" distribution, with the white areas concentrated in the hilar region (origin of the major pulmonary vessels). However, as the heart failure progresses, the quantity of fluid in the alveolar spaces increases, and the white, fluffy appearance is seen throughout the lung. Pericarditis is inflammation of the sac around the heart.

Which of the following AV blocks can be described as a gradually lengthening PR interval until ultimately the final P wave in the group fails to conduct? a. First-degree AV block b. Second-degree AV block, type I c. Second-degree AV block, type II d. Third-degree AV block

b. Second-degree AV block, type I In Mobitz type I block, the atrioventricular (AV) conduction times progressively lengthen until a P wave is not conducted. This typically occurs in a pattern ofgrouped beats and is observed on the electrocardiogram (ECG) by a gradually lengthening PR interval until ultimately the final P wave in the group fails to conduct. When all atrial impulses are conducted to the ventricles but the PR interval is greater than 0.20 second, a condition known as first-degree AV blockexists. Mobitz type II block is always anatomically located below the AV node in the bundle of His in the bundle branches or even in the Purkinje fibers. This results in an all-or-nothing situation with respect to AV conduction. Sinus P waves are or are not conducted. When conduction does occur, all PR intervals are the same. Because of the anatomic location of the block, on the surface, ECG the PR interval is constant and the QRS complexes are wide. Third-degree, or complete, AV block is a condition in which no atrial impulses can conduct from the atria to the ventricles. This is also described by the term complete heart block.

The nurse is caring for a patient who has septic shock. Which assessment finding is most important for the nurse to report to the health care provider? a. Blood pressure (BP) 92/56 mm Hg b. Skin cool and clammy c. Oxygen saturation 92% d. Heart rate 118 beats/minute

b. Skin cool and clammy Because patients in the early stage of septic shock have warm and dry skin, the patient's cool and clammy skin indicates that shock is progressing. The other information will also be reported, but does not indicate deterioration of the patient's status.

Which medication is not recommended in the treatment of shock-related lactic acidosis? a. Glucose b. Sodium bicarbonate c. Vasoconstrictor d. Large quantity of crystalloids fluids

b. Sodium bicarbonate Sodium bicarbonate is not recommended in the treatment of shock-related lactic acidosis. Glucose control to a target level of 140 to 180 mg/dL is recommended for all critically ill patients. Vasoconstrictor agents are used to increase afterload by increasing the systemic vascular resistance and improving the patient's blood pressure level. Crystalloids are balanced electrolyte solutions that may be hypotonic, isotonic, or hypertonic. Examples of crystalloid solutions used in shock situations are normal saline and lactated Ringer solution.

A patient whose heart monitor shows sinus tachycardia, rate 132, is apneic and has no palpable pulses. What is the first action that the nurse should take? a. Perform synchronized cardioversion. b. Start cardiopulmonary resuscitation (CPR). c. Administer atropine per agency dysrhythmia protocol. d. Provide supplemental oxygen via non-rebreather mask.

b. Start cardiopulmonary resuscitation (CPR). The patient's clinical manifestations indicate pulseless electrical activity and the nurse should immediately start CPR. The other actions would not be of benefit to this patient.

Which hemodynamic parameter is most appropriate for the nurse to monitor to determine the effectiveness of medications given to a patient to reduce left ventricular afterload? a. Mean arterial pressure (MAP) b. Systemic vascular resistance (SVR) c. Pulmonary vascular resistance (PVR) d. Pulmonary artery wedge pressure (PAWP)

b. Systemic vascular resistance (SVR) Systemic vascular resistance reflects the resistance to ventricular ejection, or afterload. The other parameters will be monitored, but do not reflect afterload as directly.

When developing a community health program to decrease the incidence of rheumatic fever, which action would be most important for the community health nurse to include? a. Vaccinate high-risk groups in the community with streptococcal vaccine. b. Teach community members to seek treatment for streptococcal pharyngitis. c. Teach about the importance of monitoring temperature when sore throats occur. d. Teach about prophylactic antibiotics to those with a family history of rheumatic fever

b. Teach community members to seek treatment for streptococcal pharyngitis. The incidence of rheumatic fever is decreased by treatment of streptococcal infections with antibiotics. Family history is not a risk factor for rheumatic fever. There is no immunization that is effective in decreasing the incidence of rheumatic fever. Teaching about monitoring temperature will not decrease the incidence of rheumatic fever.

The nurse educator is evaluating the performance of a new registered nurse (RN) who is providing care to a patient who is receiving mechanical ventilation with 15 cm H2O of peak end-expiratory pressure (PEEP). Which action indicates that the new RN is safe? a. The RN plans to suction the patient every 1 to 2 hours. b. The RN uses a closed-suction technique to suction the patient. c. The RN tapes connection between the ventilator tubing and the ET. d. The RN changes the ventilator circuit tubing routinely every 48 hours.

b. The RN uses a closed-suction technique to suction the patient. The closed-suction technique is used when patients require high levels of PEEP (>10 cm H2O) to prevent the loss of PEEP that occurs when disconnecting the patient from the ventilator. Suctioning should not be scheduled routinely, but it should be done only when patient assessment data indicate the need for suctioning. Taping connections between the ET and the ventilator tubing would restrict the ability of the tubing to swivel in response to patient repositioning. Ventilator tubing changes increase the risk for ventilator-associated pneumonia (VAP) and are not indicated routinely.

Which of the following patients would be a candidate for fibrinolytic therapy? (Select all that apply.) a. The patient chest pain started 8 hours ago. She has a diagnosis of NSTEMI. b. The patient chest pain started 3 hours ago, and her ECG shows a new left bundle branch block. c. The patient presents to the emergency department with chest pain of 30 minutes' duration. She has a history of cerebrovascular accident 1 month ago. d. The patient has a history of unstable angina. He has been experiencing chest pain with sudden onset. e. The patient chest pain started 1 hour ago, and his ECG shows ST elevation.

b. The patient chest pain started 3 hours ago, and her ECG shows a new left bundle branch block. e. The patient chest pain started 1 hour ago, and his ECG shows ST elevation. Eligibility criteria for administering fibrinolytics include chest pain of less than 12 hours' duration and persistent ST elevation. Exclusion criteria include recent surgery, cerebrovascular accident, and trauma.

When admitting a patient for a cardiac catheterization and coronary angiogram, which information about the patient is most important for the nurse to communicate to the health care provider? a. The patient's pedal pulses are +1. b. The patient is allergic to shellfish. c. The patient had a heart attack a year ago. d. The patient has not eaten anything today

b. The patient is allergic to shellfish. The contrast dye used for the procedure is iodine based, so patients who have shellfish allergies will require treatment with medications such as corticosteroids and antihistamines before the angiogram. The other information is also communicated to the health care provider but will not require a change in the usual precardiac catheterization orders or medications.

Which of the following statements regarding beta-blockers is correct? a. They increase heart rate and are contraindicated in tachydysrhythmias. b. They result in bronchospasm and should not be used in patients with COPD. c. They increase cardiac output and help with left ventricular failure. d. They are helpful in increasing atrioventricular node conduction and are used in heart blocks.

b. They result in bronchospasm and should not be used in patients with COPD. Knowledge of the effects of adrenergic-receptor stimulation allows for anticipation of not only the therapeutic responses brought about by beta-blockade but also the potential adverse effects of these agents. For example, bronchospasm can be precipitated by noncardioselective beta-blockers in a patient with chronic obstructive pulmonary disease secondary to blocking the effects of beta receptors in the lungs.

Which of the following cardiac enzymes is a highly specific biomarker for myocardial damage? a. CK-MB b. Troponin I c. Troponin T d. LDH

b. Troponin I Because cTnI is found only in cardiac muscle, it is a highly specific biomarker for myocardial damage, considerably more specific than CK-MB. As a consequence, patients with a positive cTnI result and a negative CK-MB result usually rule in an acute myocardial infarction (MI). A negative cTnI result that remains negative many hours after an episode of chest pain is a strong indicator that the patient is not experiencing an acute MI. Even with a negative cTnI result, symptoms of chest pain still indicate that the patient should have a comprehensive cardiac evaluation to determine if there is underlying CAD present that may later lead to complications.

Which finding is the best indicator that the fluid resuscitation for a patient with hypovolemic shock has been effective? a. Hemoglobin is within normal limits. b. Urine output is 60 mL over the last hour. c. Central venous pressure (CVP) is normal. d. Mean arterial pressure (MAP) is 72 mm Hg.

b. Urine output is 60 mL over the last hour. Assessment of end organ perfusion, such as an adequate urine output, is the best indicator that fluid resuscitation has been successful. The hemoglobin level, CVP, and MAP are useful in determining the effects of fluid administration, but they are not as useful as data indicating good organ perfusion.

The nurse identifies the nursing diagnosis of decreased cardiac output related to valvular insufficiency for the patient with infective endocarditis (IE) based on which assessment finding(s)? a. Fever, chills, and diaphoresis b. Urine output less than 30 mL/hr c. Petechiae on the inside of the mouth and conjunctiva d. Increase in heart rate of 15 beats/minute with walking

b. Urine output less than 30 mL/hr Decreased renal perfusion caused by inadequate cardiac output will lead to decreased urine output. Petechiae, fever, chills, and diaphoresis are symptoms of IE, but are not caused by decreased cardiac output. An increase in pulse rate of 15 beats/minute is normal with exercise.

The nurse notes that a patient's cardiac monitor shows that every other beat is earlier than expected, has no visible P wave, and has a QRS complex that is wide and bizarre in shape. How will the nurse document the rhythm? a. Ventricular couplets b. Ventricular bigeminy c. Ventricular R-on-T phenomenon d. Multifocal premature ventricular contractions

b. Ventricular bigeminy Ventricular bigeminy describes a rhythm in which every other QRS complex is wide and bizarre looking. Pairs of wide QRS complexes are described as ventricular couplets. There is no indication that the premature ventricular contractions (PVCs) are multifocal or that the R-on-T phenomenon is occurring.

Which of the following is most often found in ventricular dysrhythmias? a. Retrograde P waves b. Wide QRS complexes c. No P waves d. An inverted T wave

b. Wide QRS complexes Ventricular dysrhythmias result from an ectopic focus in any portion of the ventricular myocardium. The usual conduction pathway through the ventricles is not used, and the wave of depolarization must spread from cell to cell. As a result, the QRS complex is prolonged and is always greater than 0.12 second. It is the width of the QRS, not the height, that is important in the diagnosis of ventricular ectopy.

During the assessment of a 25-year-old patient with infective endocarditis (IE), the nurse would expect to find a. substernal chest pressure. b. a new regurgitant murmur. c. a pruritic rash on the chest. d. involuntary muscle movement.

b. a new regurgitant murmur. New regurgitant murmurs occur in IE because vegetations on the valves prevent valve closure. Substernal chest discomfort, rashes, and involuntary muscle movement are clinical manifestations of other cardiac disorders such as angina and rheumatic fever.

The patients at highest risk for neurogenic shock are those who have had a. a stroke. b. a spinal cord injury. c. Guillain-Barré syndrome. d. a craniotomy.

b. a spinal cord injury. The most common cause is spinal cord injury (SCI). Neurogenic shock may mistakenly be referred to as spinal shock. The latter condition refers to loss of neurologic activity below the level of SCI, but it does not necessarily involve ineffective tissue perfusion.

An 82-year-old patient is admitted into the critical care unit with a diagnosis of left-sided heart failure related to mitral stenosis. Physical assessment findings reveal tachycardia with an S and a 3/6 systolic murmur. The nurse knows that the presence of an S heart sound is a. normal for a person this age. b. a ventricular gallop. c. a systolic sound. d. heard best with the diaphragm of the stethoscope.

b. a ventricular gallop. The abnormal heart sounds are labeled the third heart sound (S3) and the fourth heart sound (S4) and are referred to as gallops when auscultated during an episode of tachycardia. Not unexpectedly, the development of an S3 heart sound is strongly associated with elevated levels of brain natriuretic peptide.

When caring for a patient who is recovering from a sudden cardiac death (SCD) event and has no evidence of an acute myocardial infarction (AMI), the nurse will anticipate teaching the patient that a. sudden cardiac death events rarely reoccur. b. additional diagnostic testing will be required. c. long-term anticoagulation therapy will be needed. d. limited physical activity after discharge will be needed to prevent future events.

b. additional diagnostic testing will be required. Diagnostic testing (e.g., stress test, Holter monitor, electrophysiologic studies, cardiac catheterization) is used to determine the possible cause of the SCD and treatment options. SCD is likely to recur. Anticoagulation therapy will not have any effect on the incidence of SCD, and SCD can occur even when the patient is resting.

The value of SVO monitoring is to determine a. oxygen saturation at the capillary level. b. an imbalance between oxygen supply and metabolic tissue demand. c. the diffusion of gases at the alveolar capillary membrane. d. the predicted cardiac output for acute pulmonary edema.

b. an imbalance between oxygen supply and metabolic tissue demand. Three of these factors (CO, Hgb, and SaO ) contribute to the supply of oxygen to the tissues. Tissue metabolism (VO ) determines oxygen consumption or the quantity of oxygen extracted at tissue level that creates the demand for oxygen.

The health care provider prescribes an infusion of heparin (Hep-Lock) and daily partial thromboplastin time (PTT) testing for a patient with venous thromboembolism (VTE). The nurse will plan to a. decrease the infusion when the PTT value is 65 seconds. b. avoid giving any IM medications to prevent localized bleeding. c. monitor posterior tibial and dorsalis pedis pulses with the Doppler. d. have vitamin K available in case reversal of the heparin is needed.

b. avoid giving any IM medications to prevent localized bleeding. IM injections are avoided in patients receiving anticoagulation. A PTT of 65 seconds is within the therapeutic range. Vitamin K is used to reverse warfarin. Pulse quality is not affected by VTE.

Angiotensin-converting enzyme (ACE) inhibitors such as captopril (Capoten) and enalapril (Vasotec) decrease peripheral vascular resistance and decrease blood pressure by the mechanism of a. direct arterial vasodilation. b. blocking the conversion of angiotensin I to angiotensin II. c. increased fluid excretion at the loop of Henle in the renal tubules. d. peripheral vasoconstriction and central vasodilation.

b. blocking the conversion of angiotensin I to angiotensin II. Angiotensin-converting enzyme (ACE) inhibitors produce vasodilation by blocking the conversion of angiotensin I to angiotensin II. Because angiotensin is a potent vasoconstrictor, limiting its production decreases peripheral vascular resistance. In contrast to the direct vasodilators and nifedipine, ACE inhibitors do not cause reflex tachycardia or induce sodium and water retention.

The nurse will suspect that the patient with stable angina is experiencing a side effect of the prescribed metoprolol (Lopressor) if the a. patient is restless and agitated. b. blood pressure is 90/54 mm Hg. c. patient complains about feeling anxious. d. cardiac monitor shows a heart rate of 61 beats/minute.

b. blood pressure is 90/54 mm Hg. Patients taking b-adrenergic blockers should be monitored for hypotension and bradycardia. Because this class of medication inhibits the sympathetic nervous system, restlessness, agitation, hypertension, and anxiety will not be side effects.

When auscultating over the patient's abdominal aorta, the nurse hears a humming sound. The nurse documents this finding as a a. thrill. b. bruit. c. murmur. d. normal finding.

b. bruit. A bruit is the sound created by turbulent blood flow in an artery. Thrills are palpable vibrations felt when there is turbulent blood flow through the heart or in a blood vessel. A murmur is the sound caused by turbulent blood flow through the heart. Auscultating a bruit in an artery is not normal and indicates pathology.

An essential aspect of teaching that may prevent recurrence of heart failure is a. notifying the physician if a 2-lb weight gain occurs in 24 hours. b. compliance with diuretic therapy. c. taking nitroglycerin if chest pain occurs. d. assessment of an apical pulse.

b. compliance with diuretic therapy. Primary topics of education include (1) the importance of a daily weight, (2) fluid restrictions, and (3) written information about the multiple medications used to control the symptoms of heart failure. Reduction or cessation of diuretics usually results in sodium and water retention, which may precipitate heart failure.

Diltiazem (Cardizem) is ordered for a patient with newly diagnosed Prinzmetal's (variant) angina. When teaching the patient, the nurse will include the information that diltiazem will a. reduce heart palpitations. b. decrease spasm of the coronary arteries. c. increase the force of the heart contractions. d. help prevent plaque from forming in the coronary arteries.

b. decrease spasm of the coronary arteries. Prinzmetal's angina is caused by coronary artery spasm. Calcium channel blockers (e.g., diltiazem, amlodipine [Norvasc]) are a first-line therapy for this type of angina. Lipid-lowering drugs help reduce atherosclerosis (i.e., plaque formation), and b-adrenergic blockers decrease sympathetic stimulation of the heart (i.e., palpitations). Medications or activities that increase myocardial contractility will increase the incidence of angina by increasing oxygen demand.

The physiologic effect of left ventricular afterload reduction is a. decreased left atrial tension. b. decreased systemic vascular resistance. c. increased filling pressures. d. decreased cardiac output.

b. decreased systemic vascular resistance. Afterload is defined as the pressure the ventricle generates to overcome the resistance to ejection created by the arteries and arterioles. After a decrease in afterload, wall tension is lowered. The technical name for afterload is systemic vascular resistance (SVR). Resistance to ejection from the right side of the heart is estimated by calculating the pulmonary vascular resistance (PVR). The PVR value is normally one-sixth of the SVR.

A patient with heart failure may be at risk for hypomagnesemia as a result of a. pump failure. b. diuretic use. c. fluid overload. d. hemodilution.

b. diuretic use. Hypomagnesemia can be caused by diuresis. Diuretic use with heart failure often contributes to low serum magnesium levels.

A patient has been on the medical floor for 1 week after a vaginal hysterectomy. A urinary catheter was inserted. Complete blood cell count results have revealed escalating white blood cell counts. The patient is transferred to the critical care unit when her condition deteriorates. Septic shock is diagnosed. The medical management of the patient's condition is aimed toward a. limiting fluids to minimize the possibility of congestive heart failure. b. finding and eradicating the cause of infection. c. discontinuing invasive monitoring as a possible cause of sepsis. d. administering vasodilator substances to increase blood flow to vital organs.

b. finding and eradicating the cause of infection. Effective treatment of severe sepsis and septic shock depends on timely recognition. The diagnosis of severe sepsis is based on the identification of three conditions: known or suspected infection, two or more of the clinical indications of the systemic inflammatory response, and evidence of at least one organ dysfunction. Clinical indications of systemic inflammatory response and sepsis were included in the original American College of Chest Physicians/Society of Critical Care Medicine consensus definitions.

Shock syndrome can best be described as a a. physiologic state resulting in hypotension and tachycardia. b. generalized systemic response to inadequate tissue perfusion. c. degenerative condition leading to death. d. condition occurring with hypovolemia that results in irreversible hypotension.

b. generalized systemic response to inadequate tissue perfusion. Shock is a complex pathophysiologic process that often results in multiple organ dysfunction syndrome and death. All types of shock eventually result in ineffective tissue perfusion and the development of acute circulatory failure.

A registered nurse (RN) is observing a student nurse who is doing a physical assessment on a patient. The RN will need to intervene immediately if the student nurse a. presses on the skin over the tibia for 10 seconds to check for edema. b. palpates both carotid arteries simultaneously to compare pulse quality. c. documents a murmur heard along the right sternal border as a pulmonic murmur. d. places the patient in the left lateral position to check for the point of maximal impulse.

b. palpates both carotid arteries simultaneously to compare pulse quality. The carotid pulses should never be palpated at the same time to avoid vagal stimulation, dysrhythmias, and decreased cerebral blood flow. The other assessment techniques also need to be corrected. However, they are not dangerous to the patient.

When an intra-aortic balloon is in place, it is essential for the nurse to frequently assess a. for a pulse deficit. b. peripheral pulses distal to the catheter insertion site. c. bilateral blood pressures. d. coronary artery perfusion.

b. peripheral pulses distal to the catheter insertion site. One complication of intra-aortic balloon support is lower extremity ischemia resulting from occlusion of the femoral artery by the catheter itself or by emboli caused by thrombus formation on the balloon. Although ischemic complications have decreased with sheathless insertion techniques and the introduction of smaller balloon catheters, evaluation of peripheral circulation remains an important nursing assessment. The presence and quality of peripheral pulses distal to the catheter insertion site are assessed frequently along with color, temperature, and capillary refill of the involved extremity. Signs of diminished perfusion must be reported immediately.

The intensive care unit (ICU) nurse educator will determine that teaching about arterial pressure monitoring for a new staff nurse has been effective when the nurse a. balances and calibrates the monitoring equipment every 2 hours. b. positions the zero-reference stopcock line level with the phlebostatic axis. c. ensures that the patient is supine with the head of the bed flat for all readings. d. rechecks the location of the phlebostatic axis when changing the patient's position.

b. positions the zero-reference stopcock line level with the phlebostatic axis. For accurate measurement of pressures, the zero-reference level should be at the phlebostatic axis. There is no need to rebalance and recalibrate monitoring equipment hourly. Accurate hemodynamic readings are possible with the patient's head raised to 45 degrees or in the prone position. The anatomic position of the phlebostatic axis does not change when patients are repositioned.

The central venous oxygen saturation (ScvO ) is decreasing in a patient who has severe pancreatitis. To determine the possible cause of the decreased ScvO , the nurse assesses the patient's a. lipase. b. temperature. c. urinary output. d. body mass index.

b. temperature. Elevated temperature increases metabolic demands and oxygen use by tissues, resulting in a drop in oxygen saturation of central venous blood. Information about the patient's body mass index, urinary output, and lipase will not help in determining the cause of the patient's drop in ScvO .

A patient has a 6-cm thoracic aortic aneurysm that was discovered during a routine chest x-ray. When obtaining an admission history from the patient, it will be most important for the nurse to ask about a. low back pain. b. trouble swallowing. c. abdominal tenderness. d. changes in bowel habits.

b. trouble swallowing. Difficulty swallowing may occur with a thoracic aneurysm because of pressure on the esophagus. The other symptoms will be important to assess for in patients with abdominal aortic aneurysms.

When evaluating the discharge teaching for a patient with chronic peripheral artery disease (PAD), the nurse determines a need for further instruction when the patient says, "I will a. have to buy some loose clothes that do not bind across my legs or waist." b. use a heating pad on my feet at night to increase the circulation and warmth in my feet." c. change my position every hour and avoid long periods of sitting with my legs crossed." d. walk to the point of pain, rest, and walk again until the pain returns for at least 30 minutes 3 times a week."

b. use a heating pad on my feet at night to increase the circulation and warmth in my feet." Because the patient has impaired circulation and sensation to the feet, the use of a heating pad could lead to burns. The other patient statements are correct and indicate that teaching has been successful.

The major key to the clinical significance of atrial flutter is the a. atrial rate. b. ventricular response rate. c. PR interval. d. QRS duration.

b. ventricular response rate. The major factor underlying atrial flutter symptoms is the ventricular response rate. If the atrial rate is 300 and the atrioventricular (AV) conduction ratio is 4:1, the ventricular response rate is 75 beats/min and should be well tolerated. If, on the other hand, the atrial rate is 300 beats/min but the AV conduction ratio is 2:1, the corresponding ventricular rate of 150 beats/min may cause angina, acute heart failure, or other signs of cardiac decompensation

The most frequent dysrhythmia seen initially with sudden cardiac death is a. premature ventricular contractions. b. ventricular tachycardia. c. ventricular fibrillation. d. asystole.

b. ventricular tachycardia. When the onset of symptoms is rapid, the most likely mechanism of death is ventricular tachycardia, which degenerates into ventricular fibrillation.

After the nurse has finished teaching a patient about the use of sublingual nitroglycerin (Nitrostat), which patient statement indicates that the teaching has been effective? a. "I can expect some nausea as a side effect of nitroglycerin." b. "I should only take the nitroglycerin if I start to have chest pain." c. "I will call an ambulance if I still have pain after taking 3 nitroglycerin 5 minutes apart." d. "Nitroglycerin helps prevent a clot from forming and blocking blood flow to my heart."

c. "I will call an ambulance if I still have pain after taking 3 nitroglycerin 5 minutes apart." The emergency medical services (EMS) system should be activated when chest pain or other symptoms are not completely relieved after 3 sublingual nitroglycerin tablets taken 5 minutes apart. Nitroglycerin can be taken to prevent chest pain or other symptoms from developing (e.g., before intercourse). Gastric upset (e.g., nausea) is not an expected side effect of nitroglycerin. Nitroglycerin does not impact the underlying pathophysiology of coronary artery atherosclerosis.

In preparation for discharge, the nurse teaches a patient with chronic stable angina how to use the prescribed short-acting and long-acting nitrates. Which patient statement indicates that the teaching has been effective? a. "I will check my pulse rate before I take any nitroglycerin tablets." b. "I will put the nitroglycerin patch on as soon as I get any chest pain." c. "I will stop what I am doing and sit down before I put the nitroglycerin under my tongue." d. "I will be sure to remove the nitroglycerin patch before taking any sublingual nitroglycerin."

c. "I will stop what I am doing and sit down before I put the nitroglycerin under my tongue." The patient should sit down before taking the nitroglycerin to decrease cardiac workload and prevent orthostatic hypotension. Transdermal nitrates are used prophylactically rather than to treat acute pain and can be used concurrently with sublingual nitroglycerin. Although the nurse should check blood pressure before giving nitroglycerin, patients do not need to check the pulse rate before taking nitrates.

A patient with a venous thromboembolism (VTE) is started on enoxaparin (Lovenox) and warfarin (Coumadin). The patient asks the nurse why two medications are necessary. Which response by the nurse is most appropriate? a. "Taking two blood thinners reduces the risk for another clot to form." b. "Lovenox will start to dissolve the clot, and Coumadin will prevent any more clots from forming." c. "Lovenox will work right away, but Coumadin takes several days to have an effect on preventing clots." d. "Because of the risk for a blood clot in the lungs, it is important for you to take more than one blood thinner."

c. "Lovenox will work right away, but Coumadin takes several days to have an effect on preventing clots." Low molecular weight heparin (LMWH) is used because of the immediate effect on coagulation and discontinued once the international normalized ratio (INR) value indicates that the warfarin has reached a therapeutic level. LMWH has no thrombolytic properties. The use of two anticoagulants is not related to the risk for pulmonary embolism, and two are not necessary to reduce the risk for another VTE. Furthermore, anticoagulants should not be described as blood thinners.

A patient has a junctional escape rhythm on the monitor. The nurse will expect the patient to have a heart rate of _____ beats/minute. a. 15 to 20 b. 20 to 40 c. 40 to 60 d. 60 to 100

c. 40 to 60 If the sinoatrial (SA) node fails to discharge, the atrioventricular (AV) node will automatically discharge at the normal rate of 40 to 60 beats/minute. The slower rates are typical of the bundle of His and the Purkinje system and may be seen with failure of both the SA and AV node to discharge. The normal SA node rate is 60 to 100 beats/minute.

After receiving report, which patient admitted to the emergency department should the nurse assess first? a. 67-year-old who has a gangrenous left foot ulcer with a weak pedal pulse b. 58-year-old who is taking anticoagulants for atrial fibrillation and has black stools c. 50-year-old who is complaining of sudden "sharp" and "worst ever" upper back pain d. 39-year-old who has right calf tenderness, redness, and swelling after a long plane ride

c. 50-year-old who is complaining of sudden "sharp" and "worst ever" upper back pain The patient's presentation is consistent with dissecting thoracic aneurysm, which will require rapid intervention. The other patients do not need urgent interventions.

One theory suggests that organ dysfunction in MODS occurs in a sequential or progressive pattern. Place the following organs in the order in which they are affected: 1. Bone marrow 2. Cardiac 3. Gut 4. Kidneys 5. Liver 6. Lungs a. 6, 5, 2, 1, 3, 4 b. 5, 4, 6, 1, 2, 3 c. 6, 5, 3, 4, 2, 1 d. 6, 3, 4, 5, 2, 1

c. 6, 5, 3, 4, 2, 1 Organ dysfunction may occur in a sequential or progressive pattern. Organ dysfunction may begin in the lungs, the most commonly affected major organ, and progress to the liver, gut, and kidneys. Cardiac and bone marrow dysfunction may follow. Neurologic and autonomic system impairment may occur and propagate the progression of organ failure, which is associated with illness severity and mortality. Organs may fail simultaneously; for example, kidney dysfunction may occur concurrently with hepatic dysfunction. After the initial insult and resuscitation, patients develop persistent hypermetabolism, a metabolic consequence of sustained systemic inflammation and physiologic stress followed closely by pulmonary dysfunction, manifested as acute respiratory distress syndrome.

You are teaching a patient with endocarditis about his diagnosis. Which statement below is correct? a. Endocarditis is a viral infection that is easily treated with antibiotics. b. The risk of this diagnosis is occlusion of the coronary arteries. c. A long course of antibiotics is needed to treat this disorder. d. Complications are rare after antibiotics have been started.

c. A long course of antibiotics is needed to treat this disorder. Treatment requires prolonged IV therapy with adequate doses of antimicrobial agents tailored to the specific infective endocarditis microbe and patient circumstances. Antibiotic treatment is prolonged, administered in high doses intravenously, and may involve combination therapy. Best outcomes are achieved if therapy is initiated before hemodynamic compromise.

The nurse notes thick, white secretions in the endotracheal tube (ET) of a patient who is receiving mechanical ventilation. Which intervention will be mosteffective in addressing this problem? a. Increase suctioning to every hour. b. Reposition the patient every 1 to 2 hours. c. Add additional water to the patient's enteral feedings. d. Instill 5 mL of sterile saline into the ET before suctioning.

c. Add additional water to the patient's enteral feedings. Because the patient's secretions are thick, better hydration is indicated. Suctioning every hour without any specific evidence for the need will increase the incidence of mucosal trauma and would not address the etiology of the ineffective airway clearance. Instillation of saline does not liquefy secretions and may decrease the SpO . Repositioning the patient is appropriate but will not decrease the thickness of secretions.

A 19-year-old patient with massive trauma and possible spinal cord injury is admitted to the emergency department (ED). Which assessment finding by the nurse will help confirm a diagnosis of neurogenic shock? a. Inspiratory crackles. b. Cool, clammy extremities. c. Apical heart rate 45 beats/min. d. Temperature 101.2° F (38.4° C).

c. Apical heart rate 45 beats/min. Neurogenic shock is characterized by hypotension and bradycardia. The other findings would be more consistent with other types of shock.

Which action is a priority for the nurse to take when the low pressure alarm sounds for a patient who has an arterial line in the left radial artery? a. Fast flush the arterial line. b. Check the left hand for pallor. c. Assess for cardiac dysrhythmias. d. Rezero the monitoring equipment.

c. Assess for cardiac dysrhythmias. The low pressure alarm indicates a drop in the patient's blood pressure, which may be caused by cardiac dysrhythmias. There is no indication to rezero the equipment. Pallor of the left hand would be caused by occlusion of the radial artery by the arterial catheter, not by low pressure. There is no indication of a need for flushing the line.

Which action by the nurse will determine if the therapies ordered for a patient with chronic constrictive pericarditis are effective? a. Assess for the presence of a paradoxical pulse. b. Monitor for changes in the patient's sedimentation rate. c. Assess for the presence of jugular venous distention (JVD). d. Check the electrocardiogram (ECG) for ST segment changes.

c. Assess for the presence of jugular venous distention (JVD). Because the most common finding on physical examination for a patient with chronic constrictive pericarditis is jugular venous distention, a decrease in JVD indicates improvement. Paradoxical pulse, ST-segment ECG changes, and changes in sedimentation rates occur with acute pericarditis but are not expected in chronic constrictive pericarditis.

A patient is recovering from a myocardial infarction (MI) and develops chest pain on day 3 that increases when taking a deep breath and is relieved by leaning forward. Which action should the nurse take next? a. Assess the feet for pedal edema. b. Palpate the radial pulses bilaterally. c. Auscultate for a pericardial friction rub. d. Check the heart monitor for dysrhythmias.

c. Auscultate for a pericardial friction rub. The patient's symptoms are consistent with the development of pericarditis, a possible complication of MI. The other assessments listed are not consistent with the description of the patient's symptoms.

The nurse obtains the following data when assessing a patient who experienced an ST-segment-elevation myocardial infarction (STEMI) 2 days previously. Which information is most important to report to the health care provider? a. The troponin level is elevated. b. The patient denies ever having a heart attack. c. Bilateral crackles are auscultated in the mid-lower lobes. d. The patient has occasional premature atrial contractions (PACs).

c. Bilateral crackles are auscultated in the mid-lower lobes. The crackles indicate that the patient may be developing heart failure, a possible complication of myocardial infarction (MI). The health care provider may need to order medications such as diuretics or angiotensin-converting enzyme (ACE) inhibitors for the patient. Elevation in troponin level at this time is expected. PACs are not life-threatening dysrhythmias. Denial is a common response in the immediate period after the MI.

Which of the following diagnostic tests is most effective for measuring overall heart size? a. Twelve-lead electrocardiography b. Echocardiography c. Chest radiography d. Vectorcardiography

c. Chest radiography Chest radiography is the oldest noninvasive method for visualizing images of the heart, and it remains a frequently used and valuable diagnostic tool. Information about cardiac anatomy and physiology can be obtained with ease and safety at a relatively low cost. Radiographs of the chest are used to estimate the cardiothoracic ratio and measure overall heart size.

The nurse should anticipate which of the following as the most common complication after an MI? a. Pulmonary edema b. Cardiogenic shock c. Dysrhythmias d. Deep vein thrombosis

c. Dysrhythmias Many patients experience complications occurring either early or late in the postinfarction course. These complications may result from electrical dysfunction or from a cardiac contractility problem. Pumping complications can cause heart failure, pulmonary edema, and cardiogenic shock. The presence of a new murmur in a patient with an acute myocardial infarction warrants special attention because it may indicate rupture of the papillary muscle. The murmur can be indicative of severe damage and impending complications such as heart failure and pulmonary edema.

A patient has been newly diagnosed with stable angina. He tells the nurse he knows a lot about his diagnosis already because his father had the same diagnosis 15 years ago. The nurse asks him to state what he already knows about angina. Which of the following responses by the patient indicates the need for additional education? a. He should stop smoking. b. He can no longer drink colas or coffee. c. He can no longer get a strong back massage. d. He should take stool softeners to prevent straining.

c. He can no longer get a strong back massage. Longer term education of the patient and the family can begin. Points to cover include (1) risk factor modification, (2) signs and symptoms of angina, (3) when to call the physician, (4) medications, and (5) dealing with emotions and stress. It is essential to teach avoidance of the Valsalva maneuver, which is defined as forced expiration against a closed glottis. This can be explained to the patient as "bearing down" during defecation or breath holding when repositioning in bed. Relaxation therapy and techniques including back rubs are encouraged when appropriate.

Following an acute myocardial infarction (AMI), a patient ambulates in the hospital hallway. When the nurse is evaluating the patient's response to the activity, which assessment data would indicate that the exercise level should be decreased? a. Blood pressure (BP) changes from 118/60 to 126/68 mmHg. b. Oxygen saturation drops from 99% to 95%. c. Heart rate increases from 66 to 92 beats/minute. d. Respiratory rate goes from 14 to 20 breaths/minute.

c. Heart rate increases from 66 to 92 beats/minute. A change in heart rate of more than 20 beats over the resting heart rate indicates that the patient should stop and rest. The increases in BP and respiratory rate, and the slight decrease in oxygen saturation, are normal responses to exercise.

Which nursing intervention will be most effective when assisting the patient with coronary artery disease (CAD) to make appropriate dietary changes? a. Give the patient a list of low-sodium, low-cholesterol foods that should be included in the diet. b. Emphasize the increased risk for heart problems unless the patient makes the dietary changes. c. Help the patient modify favorite high-fat recipes by using monosaturated oils when possible. d. Inform the patient that a diet containing no saturated fat and minimal salt will be necessary.

c. Help the patient modify favorite high-fat recipes by using monosaturated oils when possible. Lifestyle changes are more likely to be successful when consideration is given to the patient's values and preferences. The highest percentage of calories from fat should come from monosaturated fats. Although low-sodium and lowcholesterol foods are appropriate, providing the patient with a list alone is not likely to be successful in making dietary changes. Completely removing saturated fat from the diet is not a realistic expectation. Up to 7% of calories in the therapeutic lifestyle changes (TLC) diet can come from saturated fat. Telling the patient about the increased risk without assisting further with strategies for dietary change is unlikely to be successful.

An older patient with cardiogenic shock is cool and clammy and hemodynamic monitoring indicates a high systemic vascular resistance (SVR). Which intervention should the nurse anticipate doing next? a. Increase the rate for the dopamine (Intropin) infusion. b. Decrease the rate for the nitroglycerin (Tridil) infusion. c. Increase the rate for the sodium nitroprusside (Nipride) infusion. d. Decrease the rate for the 5% dextrose in normal saline (D5/.9 NS) infusion.

c. Increase the rate for the sodium nitroprusside (Nipride) infusion. Nitroprusside is an arterial vasodilator and will decrease the SVR and afterload, which will improve cardiac output. Changes in the D /.9 NS and nitroglycerin infusions will not directly decrease SVR. Increasing the dopamine will tend to increase SVR.

Clinical manifestations of ischemic hepatitis show up 1 to 2 days after the insult. Which symptom below is indicative of hepatic insufficiency? a. Elevated serum creatinine b. Decreased bilirubin c. Jaundice d. Decreased serum transaminase

c. Jaundice Clinical manifestations of hepatic insufficiency are evident 1 to 2 days after the insult. Jaundice and transient elevations in serum transaminase and bilirubin levels occur. Hyperbilirubinemia results from hepatocyte anoxic injury and an increased production of bilirubin from hemoglobin catabolism.

Which assessment finding obtained by the nurse when assessing a patient with acute pericarditis should be reported immediately to the health care provider? a. Pulsus paradoxus 8 mm Hg b. Blood pressure (BP) of 168/94 c. Jugular venous distention (JVD) to jaw level d. Level 6 (0 to 10 scale) chest pain with a deep breath

c. Jugular venous distention (JVD) to jaw level The JVD indicates that the patient may have developed cardiac tamponade and may need rapid intervention to maintain adequate cardiac output. Hypertension would not be associated with complications of pericarditis, and the BP is not high enough to indicate that there is any immediate need to call the health care provider. A pulsus paradoxus of 8 mm Hg is normal. Level 6/10 chest pain should be treated but is not unusual with pericarditis.

A 21-year-old woman is scheduled for percutaneous transluminal balloon valvuloplasty to treat mitral stenosis. Which information should the nurse include when explaining the advantages of valvuloplasty over valve replacement to the patient? a. Biologic valves will require immunosuppressive drugs after surgery. b. Mechanical mitral valves need to be replaced sooner than biologic valves. c. Lifelong anticoagulant therapy will be needed after mechanical valve replacement. d. Ongoing cardiac care by a health care provider is not necessary after valvuloplasty.

c. Lifelong anticoagulant therapy will be needed after mechanical valve replacement. Long-term anticoagulation therapy is needed after mechanical valve replacement, and this would restrict decisions about career and childbearing in this patient. Mechanical valves are durable and last longer than biologic valves. All valve repair procedures are palliative, not curative, and require lifelong health care. Biologic valves do not activate the immune system, and immunosuppressive therapy is not needed.

After an MI, a patient presents with an increasing frequency of PVCs. The patient's heart rate is 110 beats/min, and ECG indicates a sinus rhythm with up to five unifocal PVCs per minute. Which of the following should be done? The patient is alert and responsive and denies any chest pain or dyspnea. a. Administer lidocaine 100 mg bolus IV push stat. b. Administer Cardizem 20 mg IV push stat. c. Notify the physician and monitor the patient closely. d. Nothing; PVCs are expected in this patient.

c. Notify the physician and monitor the patient closely. Although premature ventricular contractions (PVCs) are frequently present after myocardial infarction, they are not always benign. In individuals with underlying heart disease, PVCs or episodes of self-terminating ventricular tachycardia (VT) are potentially malignant. Nonsustained VT is defined as three or more consecutive premature ventricular beats at a rate faster than 110 beats/min lasting less than 30 seconds. The patient does not appear symptomatic from the PVCs at this time; therefore, lidocaine is not indicated. Cardizem is not prescribed for ventricular ectopy.

A patient becomes unresponsive. The patient's heart rate is 32 beats/min, idioventricular rhythm; blood pressure is 60/32 mm Hg; SpO is 90%; and respiratory rate is 14 breaths/min. Which of the following interventions would the nurse do first? a. Notify the physician and hang normal saline wide open. b. Notify the physician and obtain the defibrillator. c. Notify the physician and obtain a temporary pacemaker. d. Notify the physician and obtain a 12-lead ECG.

c. Notify the physician and obtain a temporary pacemaker. If the sinus node and the atrioventricular (AV) junction fail, the ventricles depolarize at their own intrinsic rate of 20 to 40 times per minute. This is called adinioventricular rhythm and is naturally protective mechanism. Rather than trying to abolish the ventricular beats, the aim of treatment is to increase the effective heart rate (HR) and re-establish dominance of a higher pacing site such as the sinus node or the AV junction. Usually, a temporary pacemaker is used to increase the HR until the underlying problems that caused failure of the other pacing sites can be resolved.

Which nursing action can the registered nurse (RN) delegate to experienced unlicensed assistive personnel (UAP) working as a telemetry technician on the cardiac care unit? a. Decide whether a patient's heart rate of 116 requires urgent treatment. b. Monitor a patient's level of consciousness during synchronized cardioversion. c. Observe cardiac rhythms for multiple patients who have telemetry monitoring. d. Select the best lead for monitoring a patient admitted with acute coronary syndrome.

c. Observe cardiac rhythms for multiple patients who have telemetry monitoring. UAP serving as telemetry technicians can monitor cardiac rhythms for individuals or groups of patients. Nursing actions such as assessment and choice of the most appropriate lead based on ST segment elevation location require RN-level education and scope of practice.

The nurse will plan discharge teaching about the need for prophylactic antibiotics when having dental procedures for which patient? a. Patient admitted with a large acute myocardial infarction. b. Patient being discharged after an exacerbation of heart failure. c. Patient who had a mitral valve replacement with a mechanical valve. d. Patient being treated for rheumatic fever after a streptococcal infection

c. Patient who had a mitral valve replacement with a mechanical valve. Current American Heart Association guidelines recommend the use of prophylactic antibiotics before dental procedures for patients with prosthetic valves to prevent infective endocarditis (IE). The other patients are not at risk for IE.

Assessment of a patient with pericarditis may reveal which of the following signs and symptoms? a. Ventricular gallop and substernal chest pain b. Narrowed pulse pressure and shortness of breath c. Pericardial friction rub and pain d. Pericardial tamponade and widened pulse pressure

c. Pericardial friction rub and pain Pain is the most common symptom of pericarditis, and a pericardial friction rub is the most common initial sign. A friction rub is best auscultated with a stethoscope at the sternal border and is described as a grating, scraping, or leathery scratching. Pericarditis frequently produces a pericardial effusion.

Which of the following assessment findings is most specific for acute onset of pulmonary edema? a. Pulmonary crackles b. Peripheral edema c. Pink, frothy sputum d. Elevated central venous pressure

c. Pink, frothy sputum Patients experiencing heart failure and pulmonary edema are extremely breathless and anxious and have a sensation of suffocation. They expectorate pink, frothy sputum and feel as if they are drowning. They may sit bolt upright, gasp for breath, or thrash about. The respiratory rate is elevated, and accessory muscles of ventilation are used, with nasal flaring and bulging neck muscles. Respirations are characterized by loud inspiratory and expiratory gurgling sounds.

A patient who has been involved in a motor vehicle crash arrives in the emergency department (ED) with cool, clammy skin; tachycardia; and hypotension. Which intervention ordered by the health care provider should the nurse implement first? a. Insert two large-bore IV catheters. b. Initiate continuous electrocardiogram (ECG) monitoring. c. Provide oxygen at 100% per non-rebreather mask. d. Draw blood to type and crossmatch for transfusions.

c. Provide oxygen at 100% per non-rebreather mask. The first priority in the initial management of shock is maintenance of the airway and ventilation. ECG monitoring, insertion of IV catheters, and obtaining blood for transfusions should also be rapidly accomplished but only after actions to maximize oxygen delivery have been implemented.

When caring for a patient with pulmonary hypertension, which parameter ismost appropriate for the nurse to monitor to evaluate the effectiveness of the treatment? a. Central venous pressure (CVP) b. Systemic vascular resistance (SVR) c. Pulmonary vascular resistance (PVR) d. Pulmonary artery wedge pressure (PAWP)

c. Pulmonary vascular resistance (PVR) PVR is a major contributor to pulmonary hypertension, and a decrease would indicate that pulmonary hypertension was improving. The other parameters also may be monitored but do not directly assess for pulmonary hypertension.

Which of the ECG findings would be positive for an inferior wall MI? a. ST segment depression in leads I, aVL, and V2 to V4 b. Q waves in leads V1 to V2 c. Q waves in leads II, III, and aVF d. T-wave inversion in leads V4 to V6, I, and aVL

c. Q waves in leads II, III, and aVF Abnormal Q waves develop in leads overlying the affected area. An inferior wall infarction is seen with changes in leads II, III, and aVF. Leads I and aVF are selected to detect a sudden change in ventricular axis. If ST segment monitoring is required, the lead is selected according to the area of ischemia. If the ischemic area is not known, leads V3 and III are recommended to detect ST segment ischemia.

A 52-year-old patient presents to the emergency department with reports of substernal chest pain. A history is taken; serum creatine kinase (CK) and lactate dehydrogenase (LDH) isoenzymes and serum lipid studies are ordered, as is a 12-lead ECG. Which of the following results is most significant in diagnosing an MI during the first 12 hours of chest pain? a. ECG—inverted T waves b. Serum enzymes—elevated LDH4 c. Serum enzymes—elevated CK-MB d. Patient history—substernal chest pain

c. Serum enzymes—elevated CK-MB The creatine kinase (CK) muscle/brain (MB) biomarker (CK-MB) is released as a result of myocardial damage, and serum levels rise 4 to 8 hours after myocardial infarction (MI), peak at 15 to 24 hours, and remain elevated for 2 to 3 days. Serial samples are drawn routinely at 6- or 8-hour intervals, and three samples are usually sufficient to support or rule out the diagnosis of MI.

A patient admitted to the coronary care unit (CCU) with an ST-segment-elevation myocardial infarction (STEMI) is restless and anxious. The blood pressure is 86/40 and heart rate is 123. Based on this information, which nursing diagnosis is a priority for the patient? a. Acute pain related to myocardial infarction b. Anxiety related to perceived threat of death c. Stress overload related to acute change in health d. Decreased cardiac output related to cardiogenic shock

c. Stress overload related to acute change in health All the nursing diagnoses may be appropriate for this patient, but the hypotension and tachycardia indicate decreased cardiac output and shock from the damaged myocardium. This will result in decreased perfusion to all vital organs (e.g., brain, kidney, heart) and is a priority.

To accurately measure the heart rate of a patient in normal sinus rhythm, which technique would be the most accurate? a. The number of R waves in a 6-second strip b. The number of large boxes in a 6-second strip c. The number of small boxes between QRS complexes divided into 1500 d. The number of large boxes between consecutive R waves divided into 300

c. The number of small boxes between QRS complexes divided into 1500 Calculation of heart rate if the rhythm is regular may be done using the following methods. Method 1: number of RR intervals in 6 seconds multiplied by 10 (e.g., 8 ´ 10 = 80/min). Method 2: number of large boxes between QRS complexes divided into 300 (e.g., 300 ÷ 4 = 75/min). Method 3: number of small boxes between QRS complexes divided into 1500 (e.g., 1500 ÷ 18 = 84/min).

Which action by a nurse who is giving fondaparinux (Arixtra) to a patient with a lower leg venous thromboembolism (VTE) indicates that more education about the drug is needed? a. The nurse avoids rubbing the injection site after giving the drug. b. The nurse injects the drug into the abdominal subcutaneous tissue. c. The nurse ejects the air bubble in the syringe before giving the drug. d. The nurse fails to assess the partial thromboplastin time (PTT) before giving the drug.

c. The nurse ejects the air bubble in the syringe before giving the drug. The air bubble is not ejected before giving fondaparinux to avoid loss of medication. The other actions by the nurse are appropriate.

Which information obtained by the nurse who is admitting the patient for magnetic resonance imaging (MRI) will be most important to report to the health care provider before the MRI? a. The patient has an allergy to shellfish. b. The patient has a history of atherosclerosis. c. The patient has a permanent ventricular pacemaker. d. The patient took all the prescribed cardiac medications today.

c. The patient has a permanent ventricular pacemaker. MRI is contraindicated for patients with implanted metallic devices such as pacemakers. The other information also will be reported to the health care provider but does not impact on whether or not the patient can have an MRI.

Which finding about a patient who is receiving vasopressin (Pitressin) to treat septic shock is most important for the nurse to communicate to the health care provider? a. The patient's urine output is 18 mL/hr. b. The patient's heart rate is 110 beats/minute. c. The patient is complaining of chest pain. d. The patient's peripheral pulses are weak.

c. The patient is complaining of chest pain. Because vasopressin is a potent vasoconstrictor, it may decrease coronary artery perfusion. The other information is consistent with the patient's diagnosis and should be reported to the health care provider but does not indicate a need for a change in therapy.

A nurse is weaning a 68-kg male patient who has chronic obstructive pulmonary disease (COPD) from mechanical ventilation. Which patient assessment finding indicates that the weaning protocol should be stopped? a. The patient's heart rate is 97 beats/min. b. The patient's oxygen saturation is 93%. c. The patient respiratory rate is 32 breaths/min. d. The patient's spontaneous tidal volume is 450 mL.

c. The patient respiratory rate is 32 breaths/min. Tachypnea is a sign that the patient's work of breathing is too high to allow weaning to proceed. The patient's heart rate is within normal limits, although the nurse should continue to monitor it. An oxygen saturation of 93% is acceptable for a patient with COPD. A spontaneous tidal volume of 450 mL is within the acceptable range.

The patient with neurogenic shock is receiving a phenylephrine (Neo-Synephrine) infusion through a right forearm IV. Which assessment finding obtained by the nurse indicates a need for immediate action? a. The patient's heart rate is 58 beats/minute. b. The patient's extremities are warm and dry. c. The patient's IV infusion site is cool and pale. d. The patient's urine output is 28 mL over the last hour.

c. The patient's IV infusion site is cool and pale. The coldness and pallor at the infusion site suggest extravasation of the phenylephrine. The nurse should discontinue the IV and, if possible, infuse the medication into a central line. An apical pulse of 58 is typical for neurogenic shock but does not indicate an immediate need for nursing intervention. A 28-mL urinary output over 1 hour would require the nurse to monitor the output over the next hour, but an immediate change in therapy is not indicated. Warm, dry skin is consistent with early neurogenic shock, but it does not indicate a need for a change in therapy or immediate action.

Which assessment finding obtained by the nurse when caring for a patient receiving mechanical ventilation indicates the need for suctioning? a. The patient's oxygen saturation is 93%. b. The patient was last suctioned 6 hours ago. c. The patient's respiratory rate is 32 breaths/minute. d. The patient has occasional audible expiratory wheezes.

c. The patient's respiratory rate is 32 breaths/minute. The increase in respiratory rate indicates that the patient may have decreased airway clearance and requires suctioning. Suctioning is done when patient assessment data indicate that it is needed, not on a scheduled basis. Occasional expiratory wheezes do not indicate poor airway clearance, and suctioning the patient may induce bronchospasm and increase wheezing. An oxygen saturation of 93% is acceptable and does not suggest that immediate suctioning is needed.

Which information will the nurse include when teaching a patient who is scheduled for a radiofrequency catheter ablation for treatment of atrial flutter? a. The procedure will prevent or minimize the risk for sudden cardiac death. b. The procedure will use cold therapy to stop the formation of the flutter waves. c. The procedure will use electrical energy to destroy areas of the conduction system. d. The procedure will stimulate the growth of new conduction pathways between the atria.

c. The procedure will use electrical energy to destroy areas of the conduction system. Radiofrequency catheter ablation therapy uses electrical energy to "burn" or ablate areas of the conduction system as definitive treatment of atrial flutter (i.e., restore normal sinus rhythm) and tachydysrhythmias. All other statements regarding the procedure are incorrect.

A patient presents with severe substernal chest pain. The patient exclaims, "This is the most severe pain I have ever felt!" The patient reports that the pain came on suddenly about 2 hours ago and that three sublingual nitroglycerin tablets have not relieved the pain. The 12-lead ECG reveals only the following abnormalities: T-wave inversion in leads I, aV3, V4, and V5; pathologic Q waves in leads II, III, and aVF; ST segment elevation in leads V1, V2, V3, and V4. Which of the following is probably accurate about this patient? a. This patient has an old lateral wall infarction. b. This patient is having an inferior wall infarction. c. This patient is having an acute anterior wall infarction. d. This patient is having a posterior wall infarction.

c. This patient is having an acute anterior wall infarction. Acute anterior wall infarctions are manifested by electrocardiographic (ECG) changes in leads V2 to V4. Inferior infarctions are manifested by ECG changes in leads II, III, and aVF. Lateral wall infarctions are manifested by ECG changes in leads V5 to V6, I, and aVL. Posterior wall infarctions are manifested by ECG changes in leads V1 to V2.

A nurse is assessing a patient who is receiving a nitroprusside (Nipride) infusion to treat cardiogenic shock. Which finding indicates that the medication is effective? a. No new heart murmurs b. Decreased troponin level c. Warm, pink, and dry skin d. Blood pressure 92/40 mm Hg

c. Warm, pink, and dry skin Warm, pink, and dry skin indicates that perfusion to tissues is improved. Since nitroprusside is a vasodilator, the blood pressure may be low even if the medication is effective. Absence of a heart murmur and a decrease in troponin level are not indicators of improvement in shock.

During the administration of the thrombolytic agent to a patient with an acute myocardial infarction (AMI), the nurse should stop the drug infusion if the patient experiences a. bleeding from the gums. b. increase in blood pressure. c. a decrease in level of consciousness. d. a nonsustained episode of ventricular tachycardia.

c. a decrease in level of consciousness. The change in level of consciousness indicates that the patient may be experiencing intracranial bleeding, a possible complication of thrombolytic therapy. Some bleeding of the gums is an expected side effect of the therapy but not an indication to stop infusion of the thrombolytic medication. A decrease in blood pressure could indicate internal bleeding. A nonsustained episode of ventricular tachycardia is a common reperfusion dysrhythmia and may indicate that the therapy is effective.

A nurse palpates the descending aorta and feels a strong, bounding pulse. The nurse reports the findings to the physician because the results suggest a. decreased cardiac output. b. increased cardiac output. c. an aneurysm. d. aortic insufficiency.

c. an aneurysm. When the patient is in the supine position, the abdominal aortic pulsation is located in the epigastric area and can be felt as a forward movement when firm fingertip pressure is applied above the umbilicus. An abnormally strong or bounding pulse suggests the presence of an aneurysm or an occlusion distal to the examination site. If it is prominent or diffuse, the pulsation may indicate an abdominal aneurysm. A diminished or absent pulse may indicate low CO, arterial stenosis, or occlusion proximal to the site of the examination.

Data concerning coronary artery disease (CAD) and specific risk factors have demonstrated a. a low correlation of modifiable risk factors to CAD. b. the onset of CAD in middle age. c. an association between development of specific risk factors and CAD. d. no decisive correlation between risk factors and CAD.

c. an association between development of specific risk factors and CAD. Research and epidemiologic data collected during the past 50 years have demonstrated a strong association between specific risk factors and the development of CAD. In general, CAD symptoms are seen in persons age 45 years and older. Primary cardiovascular risk factors are different in men and women, with women having higher rates of diabetes and hypertension compared with men.

Nursing management of the patient with angina is directed toward a. immediate administration of antiplatelet therapy. b. assessment of history of previous anginal episodes. c. assessment and documentation of chest pain episodes. d. administration of prophylactic lidocaine for ventricular ectopy.

c. assessment and documentation of chest pain episodes. Nursing interventions focus on early identification of myocardial ischemia, control of chest pain, recognition of complications, maintenance of a calm environment, and patient and family education. It is important to document the characteristics of the pain and the patient's heart rate and rhythm, blood pressure, respirations, temperature, skin color, peripheral pulses, urine output, mentation, and overall tissue perfusion.

A 23-year-old patient tells the health care provider about experiencing cold, numb fingers when running during the winter and Raynaud's phenomenon is suspected. The nurse will anticipate teaching the patient about tests for a. hyperglycemia. b. hyperlipidemia. c. autoimmune disorders. d. coronary artery disease.

c. autoimmune disorders. Secondary Raynaud's phenomenon may occur in conjunction with autoimmune diseases such as rheumatoid arthritis. Patients should be screened for autoimmune disorders. Raynaud's phenomenon is not associated with hyperlipidemia, hyperglycemia, or coronary artery disease.

The patient is on day 3 of hospitalization in the ICU. The 45-year-old patient was admitted with a diagnosis of acute MI. The patient complains of fatigue, not sleeping the past two nights, and change in appetite. The nurse is aware that these are symptoms of a. angina. b. anxiety. c. depression. d. endocarditis.

c. depression. Depression is a phenomenon that occurs across a wide spectrum of human experience. Key symptoms of depression mentioned frequently by cardiac patients are fatigue, change in appetite, and sleep disturbance.

The gastrointestinal system is a common target organ for MODS related to a. anorexia. b. limited or absent food ingestion. c. disruption of the mucosal barrier from hypoperfusion. d. a decrease in hydrochloric acid secretion.

c. disruption of the mucosal barrier from hypoperfusion. With microcirculatory failure to the gastrointestinal tract, the gut's barrier function may be lost, which leads to bacterial translocation, sustained inflammation, endogenous endotoxemia, and MODS.

When checking the patient's back, the nurse pushes her thumb into the patient's sacrum. An indentation remains. The nurse charts that the patient has a. sacral compromise. b. delayed skin turgor. c. pitting edema. d. dehydration.

c. pitting edema. Pitting edema occurs when an impression is left in the tissue when the thumb is removed. The dependent tissues within the legs and sacrum are particularly susceptible. Edema may be dependent, unilateral, or bilateral and pitting or nonpitting.

Laboratory values for DIC show abnormalities in a. liver function tests. b. tests for renal function. c. platelet counts. d. blood glucose levels

c. platelet counts. Low platelet counts and elevated D-dimer concentrations and fibrinogen degradation products are clinical indicators of DIC.

A 68-year-old patient is admitted to the critical care unit with reports of midchest pressure radiating into the jaw and shortness of breath when walking up stairs. The patient is admitted with a diagnosis of "rule out myocardial infarction." The history portion of the assessment should be guided by a. medical history. b. history of prior surgeries. c. presenting symptoms. d. a review of systems.

c. presenting symptoms. For a patient in acute distress, the history taking is shortened to just a few questions about the patient's chief complaint, precipitating events, and current medications. For a patient who is not in obvious distress, the history focuses on the following four areas: review of the patient's present illness; overview of the patient's general cardiovascular status; review of the patient's general health status, including family history of coronary artery disease (CAD), hypertension, diabetes, peripheral arterial disease, or stroke; and survey of the patient's lifestyle, including risk factors for CAD.

The nurse performing an assessment with a patient who has chronic peripheral artery disease (PAD) of the legs and an ulcer on the right second toe would expect to find a. dilated superficial veins. b. swollen, dry, scaly ankles. c. prolonged capillary refill in all the toes. d. a serosanguineous drainage from the ulcer.

c. prolonged capillary refill in all the toes. Capillary refill is prolonged in PAD because of the slower and decreased blood flow to the periphery. The other listed clinical manifestations are consistent with chronic venous disease.

A pacemaker is connected to an external temporary pulse generator. The sensitivity control a. determines heart rate to muscle activity. b. determines the milliamperes needed to achieve capture. c. regulates the ability of the pacemaker to detect the heart's intrinsic electrical activity. d. allows for selection of intervals between atria and ventricles.

c. regulates the ability of the pacemaker to detect the heart's intrinsic electrical activity. The sensitivity control regulates the ability of the pacemaker to detect the heart's intrinsic electrical activity. Sensitivity is measured in millivolts (mV) and determines the size of the intracardiac signal that the generator will recognize.

Hypovolemic shock that results from an internal shifting of fluid from the intravascular space to the extravascular space is known as a. absolute hypovolemia. b. distributive hypovolemia. c. relative hypovolemia. d. compensatory hypovolemia.

c. relative hypovolemia. Hypovolemia results in a loss of circulating fluid volume. A decrease in circulating volume leads to a decrease in venous return, which results in a decrease in end-diastolic volume or preload.

When caring for a patient with infective endocarditis of the tricuspid valve, the nurse should monitor the patient for the development of a. flank pain. b. splenomegaly. c. shortness of breath. d. mental status changes.

c. shortness of breath. Embolization from the tricuspid valve would cause symptoms of pulmonary embolus. Flank pain, changes in mental status, and splenomegaly would be associated with embolization from the left-sided valves.

A patient is admitted for palliative care for end-stage heart failure. The nurse's primary goal is a. to reverse heart failure with the use of diuretics. b. to increase activity tolerance. c. symptom management and relief of pain. d. to increase cardiac output related to alteration of contractility.

c. symptom management and relief of pain. The primary aim of palliative care is symptom management and the relief of suffering. Fundamental to all symptom management strategies for heart failure is the optimization of medications, according to current guidelines.

A patient has an implantable cardioverter defibrillator (ICD) for chronic ventricular tachydysrhythmias. If the patient's rhythm deteriorates to ventricular fibrillation, a. an external defibrillator will need to be applied. b. start CPR and call a code. c. the ICD will defibrillate at a high energy level. d. the ICD is programmed to cardiovert at a high energy level.

c. the ICD will defibrillate at a high energy level. If the dysrhythmia deteriorates into ventricular fibrillation, the implantable cardioverter defibrillator is programmed to defibrillate at a higher energy. If the dysrhythmia terminates spontaneously, the device will not discharge.

The rationale for giving the patient additional fluids after a cardiac catheterization is that a. fluids help keep the femoral vein from clotting at the puncture site. b. the patient had a nothing-by-mouth order before the procedure. c. the radiopaque contrast acts as an osmotic diuretic. d. fluids increase cardiac output.

c. the radiopaque contrast acts as an osmotic diuretic. Fluid is given for rehydration because the radiopaque contrast acts as an osmotic diuretic. Fluid is also used to prevent contrast-induced nephropathy or damage to the kidney from the contrast dye used to visualize the heart structures.

A characteristic event in junctional dysrhythmias is a. irregular rhythm. b. rapid depolarization to the ventricles. c. the spread of the impulse in two directions at once. d. a widened QRS.

c. the spread of the impulse in two directions at once. After an ectopic impulse arises in the junction, it spreads in two directions at once. One wave of depolarization spreads upward into the atria and depolarizes them, causing the recording of a P wave on the electrocardiogram. At the same time, another wave of depolarization spreads downward into the ventricles through the normal conduction pathway, producing a normal QRS complex.

The nurse has received the laboratory results for a patient who developed chest pain 4 hours ago and may be having a myocardial infarction. The mostimportant laboratory result to review will be a. myoglobin. b. low-density lipoprotein (LDL) cholesterol. c. troponins T and I. d. creatine kinase-MB (CK-MB).

c. troponins T and I. Cardiac troponins start to elevate 4 to 6 hours after myocardial injury and are highly specific to myocardium. They are the preferred diagnostic marker for myocardial infarction. Myoglobin rises in response to myocardial injury within 30 to 60 minutes. It is rapidly cleared from the body, thus limiting its use in the diagnosis of myocardial infarction. LDL cholesterol is useful in assessing cardiovascular risk but is not helpful in determining whether a patient is having an acute myocardial infarction. Creatine kinase (CK-MB) is specific to myocardial injury and infarction and increases 4 to 6 hours after the infarction occurs. It is often trended with troponin levels.

The emergency department (ED) nurse receives report that a patient involved in a motor vehicle crash is being transported to the facility with an estimated arrival in 1 minute. In preparation for the patient's arrival, the nurse will obtain a. hypothermia blanket. b. lactated Ringer's solution. c. two 14-gauge IV catheters. d. dopamine (Intropin) infusion.

c. two 14-gauge IV catheters. A patient with multiple trauma may require fluid resuscitation to prevent or treat hypovolemic shock, so the nurse will anticipate the need for 2 large bore IV lines to administer normal saline. Lactated Ringer's solution should be used cautiously and will not be ordered until the patient has been assessed for possible liver abnormalities. Vasopressor infusion is not used as the initial therapy for hypovolemic shock. Patients in shock need to be kept warm not cool.

A patient with diagnosis of CAD with chest pain is admitted into the critical care unit. The patient is suddenly awakened with severe chest pain. Three nitroglycerin sublingual tablets are administered 5 minutes apart without relief. A 12-lead ECG reveals nonspecific ST segment elevation. This patient probably has a. silent ischemia. b. stable angina. c. unstable angina. d. Prinzmetal angina.

c. unstable angina. Unstable angina usually is more intense than stable angina, may awaken the person from sleep, or may necessitate more than nitrates for pain relief. A change in the level or frequency of symptoms requires immediate medical evaluation. Severe angina that persists for more than 5 minutes, worsens in intensity, and is not relieved by one nitroglycerin tablet is a medical emergency. Stable angina is predictable and caused by similar precipitating factors each time; typically, it is exercise induced. Patients become used to the pattern of this type of angina and may describe it as "my usual chest pain." Pain control should be achieved within 5 minutes of rest and by taking sublingual nitroglycerin. Silent ischemia describes a situation in which objective evidence of ischemia is observed on an electrocardiographic monitor but the person does not complain of anginal symptoms. Variant unstable angina, or Prinzmetal angina, is caused by a dynamic obstruction from intense vasoconstriction of a coronary artery. Spasm can occur with or without atherosclerotic lesions. Variant angina commonly occurs when the individual is at rest, and it is often cyclic, occurring at the same time every day.

A patient was admitted on the night shift with a diagnosis of acute myocardial infarction. Upon auscultation, the nurse hears a harsh, holosystolic murmur along the left sternal border. The nurse notifies the physician immediately because the symptoms are indicate the patient has developed a. papillary muscle rupture. b. tricuspid stenosis. c. ventricular septal rupture. d. pericardial friction rub.

c. ventricular septal rupture. Ventricular septal rupture is a new opening in the septum between the two ventricles. It creates a harsh, holosystolic murmur that is loudest (by auscultation) along the left sternal border. Papillary muscle rupture is auscultation of a new, high-pitched, holosystolic, blowing murmur at the cardiac apex. Tricuspid stenosis is a quiet murmur that becomes louder with inspiration and is located in the epigastrium area. A pericardial friction rub is a sound that can occur within 2 to 7 days after a myocardial infarction. The friction rub results from pericardial inflammation (pericarditis). Classically, a pericardial friction rub is a grating or scratching sound that is both systolic and diastolic, corresponding to cardiac motion within the pericardial sac.

The nurse obtains a health history from a 65-year-old patient with a prosthetic mitral valve who has symptoms of infective endocarditis (IE). Which question by the nurse is most appropriate? a. "Do you have a history of a heart attack?" b. "Is there a family history of endocarditis?" c. "Have you had any recent immunizations?" d. "Have you had dental work done recently?"

d. "Have you had dental work done recently?" Dental procedures place the patient with a prosthetic mitral valve at risk for infective endocarditis (IE). Myocardial infarction (MI), immunizations, and a family history of endocarditis are not risk factors for IE.

The nurse establishes the nursing diagnosis of ineffective health maintenance related to lack of knowledge regarding long-term management of rheumatic fever when a 30-year-old recovering from rheumatic fever without carditis says which of the following? a. "I will need prophylactic antibiotic therapy for 5 years." b. "I will need to take aspirin or ibuprofen (Motrin) to relieve my joint pain." c. "I will call the doctor if I develop excessive fatigue or difficulty breathing." d. "I will be immune to further episodes of rheumatic fever after this infection."

d. "I will be immune to further episodes of rheumatic fever after this infection." Patients with a history of rheumatic fever are more susceptible to a second episode. Patients with rheumatic fever without carditis require prophylaxis until age 20 and for a minimum of 5 years. The other patient statements are correct and would not support the nursing diagnosis of ineffective health maintenance.

Which statement made by a patient with coronary artery disease after the nurse has completed teaching about therapeutic lifestyle changes (TLC) diet indicates that further teaching is needed? a. "I will switch from whole milk to 1% milk." b. "I like salmon and I will plan to eat it more often." c. "I can have a glass of wine with dinner if I want one." d. "I will miss being able to eat peanut butter sandwiches."

d. "I will miss being able to eat peanut butter sandwiches." Although only 30% of the daily calories should come from fats, most of the fat in the TLC diet should come from monosaturated fats such as are found in nuts, olive oil, and canola oil. The patient can include peanut butter sandwiches as part of the TLC diet. The other patient comments indicate a good understanding of the TLC diet.

After providing a patient with discharge instructions on the management of a new permanent pacemaker, the nurse knows that teaching has been effective when the patient states a. "I will avoid cooking with a microwave oven or being near one in use." b. "It will be 1 month before I can take a bath or return to my usual activities." c. "I will notify the airlines when I make a reservation that I have a pacemaker." d. "I won't lift the arm on the pacemaker side up very high until I see the doctor."

d. "I won't lift the arm on the pacemaker side up very high until I see the doctor." The patient is instructed to avoid lifting the arm on the pacemaker side above the shoulder to avoid displacing the pacemaker leads. The patient should notify airport security about the presence of a pacemaker before going through the metal detector, but there is no need to notify the airlines when making a reservation. Microwave oven use does not affect the pacemaker. The insertion procedure involves minor surgery that will have a short recovery period.

While working in the outpatient clinic, the nurse notes that a patient has a history of intermittent claudication. Which statement by the patient would support this information? a."When I stand too long, my feet start to swell." b. "I get short of breath when I climb a lot of stairs." c. "My fingers hurt when I go outside in cold weather." d. "My legs cramp whenever I walk more than a block."

d. "My legs cramp whenever I walk more than a block." Cramping that is precipitated by a consistent level of exercise is descriptive of intermittent claudication. Finger pain associated with cold weather is typical of Raynaud's phenomenon. Shortness of breath that occurs with exercise is not typical of intermittent claudication, which is reproducible. Swelling associated with prolonged standing is typical of venous disease.

The nurse has received change-of-shift report about the following patients on the progressive care unit. Which patient should the nurse see first? a. A patient who is in a sinus rhythm, rate 98, after having electrical cardioversion 2 hours ago b. A patient with new onset atrial fibrillation, rate 88, who has a first dose of warfarin (Coumadin) due c. A patient with second-degree atrioventricular (AV) block, type 1, rate 60, who is dizzy when ambulating d. A patient whose implantable cardioverter-defibrillator (ICD) fired two times today who has a dose of amiodarone (Cordarone) due

d. A patient whose implantable cardioverter-defibrillator (ICD) fired two times today who has a dose of amiodarone (Cordarone) due The frequent firing of the ICD indicates that the patient's ventricles are very irritable, and the priority is to assess the patient and administer the amiodarone. The other patients may be seen after the amiodarone is administered.

Which of the following interventions should be strictly followed to ensure accurate cardiac output readings? a. Use 5 mL of iced injectate only. b. Inject the fluid into the pulmonary artery port only. c. Ensure a difference of at least 5° C between injectate temperature and the patient's body temperature. d. Administer the injectate within 4 seconds.

d. Administer the injectate within 4 seconds. To ensure accurate readings, the difference between injectate temperature and body temperature must be at least 10° C, and the injectate must be delivered within 4 seconds, with minimal handling of the syringe to prevent warming of the solution. This is particularly important when iced injectate is used.

Which topic should the nurse include in patient teaching for a patient with a venous stasis ulcer on the left lower leg? a. Need to increase carbohydrate intake b. Methods of keeping the wound area dry c. Purpose of prophylactic antibiotic therapy d. Application of elastic compression stockings

d. Application of elastic compression stockings Compression of the leg is essential to healing of venous stasis ulcers. High dietary intake of protein, rather than carbohydrates, is needed. Prophylactic antibiotics are not routinely used for venous ulcers. Moist dressings are used to hasten wound healing.

A patient develops sinus bradycardia at a rate of 32 beats/minute, has a blood pressure (BP) of 80/42 mm Hg, and is complaining of feeling faint. Which actions should the nurse take next? a. Recheck the heart rhythm and BP in 5 minutes. b. Have the patient perform the Valsalva maneuver. c. Give the scheduled dose of diltiazem (Cardizem). d. Apply the transcutaneous pacemaker (TCP) pads.

d. Apply the transcutaneous pacemaker (TCP) pads. The patient is experiencing symptomatic bradycardia, and treatment with TCP is appropriate. Continued monitoring of the rhythm and BP is an inadequate response. Calcium channel blockers will further decrease the heart rate, and the diltiazem should be held. The Valsalva maneuver will further decrease the rate.

To determine the effects of therapy for a patient who is being treated for heart failure, which laboratory result will the nurse plan to review? a. Troponin b. Homocysteine (Hcy) c. Low-density lipoprotein (LDL) d. B-type natriuretic peptide (BNP)

d. B-type natriuretic peptide (BNP) Increased levels of BNP are a marker for heart failure. The other laboratory results would be used to assess for myocardial infarction (troponin) or risk for coronary artery disease (Hcy and LDL).

A patient's cardiac monitor shows sinus rhythm, rate 64. The P-R interval is 0.18 seconds at 1:00 AM, 0.22 seconds at 2:30 PM, and 0.28 seconds at 4:00PM. Which action should the nurse take next? a. Place the transcutaneous pacemaker pads on the patient. b. Administer atropine sulfate 1 mg IV per agency dysrhythmia protocol. c. Document the patient's rhythm and assess the patient's response to the rhythm. d. Call the health care provider before giving the next dose of metoprolol (Lopressor).

d. Call the health care provider before giving the next dose of metoprolol (Lopressor). The patient has progressive first-degree atrioventricular (AV) block, and the b-blocker should be held until discussing the medication with the health care provider. Documentation and assessment are appropriate but not fully adequate responses. The patient with first-degree AV block usually is asymptomatic, and a pacemaker is not indicated. Atropine is sometimes used for symptomatic bradycardia, but there is no indication that this patient is symptomatic.

Evaluation of arterial circulation to an extremity is accomplished by assessing which of the following? a. Homans sign b. Skin turgor c. Peripheral edema d. Capillary refill

d. Capillary refill Capillary refill assessment is a maneuver that uses the patient's nail beds to evaluate both arterial circulation to the extremity and overall perfusion. The severity of arterial insufficiency is directly proportional to the amount of time necessary to re-establish flow and color.

A patient is being evaluated for post-thrombotic syndrome. Which assessment will the nurse perform? a. Ask about leg pain with exercise. b. Determine the ankle-brachial index. c. Assess capillary refill in the patient's toes. d. Check for presence of lipodermatosclerosis.

d. Check for presence of lipodermatosclerosis. Clinical signs of post-thrombotic syndrome include lipodermatosclerosis. In this situation, the skin on the lower leg becomes scarred, and the leg becomes tapered like an "inverted bottle." The other assessments would be done for patients with peripheral arterial disease.

To evaluate the effectiveness of the pantoprazole (Protonix) ordered for a patient with systemic inflammatory response syndrome (SIRS), which assessment will the nurse perform? a. Auscultate bowel sounds. b. Palpate for abdominal pain. c. Ask the patient about nausea. d. Check stools for occult blood.

d. Check stools for occult blood. Proton pump inhibitors are given to decrease the risk for stress ulcers in critically ill patients. The other assessments also will be done, but these will not help in determining the effectiveness of the pantoprazole administration.

Which action could the nurse delegate to unlicensed assistive personnel (UAP) trained as electrocardiogram (ECG) technicians working on the cardiac unit? a. Select the best lead for monitoring a patient with an admission diagnosis of Dressler syndrome. b. Obtain a list of herbal medications used at home while admitting a new patient with pericarditis. c. Teach about the need to monitor the weight daily for a patient who has hypertrophic cardiomyopathy. d. Check the heart monitor for changes in rhythm while a patient who had a valve replacement ambulates.

d. Check the heart monitor for changes in rhythm while a patient who had a valve replacement ambulates. Under the supervision of registered nurses (RNs), UAP check the patient's cardiac monitor and obtain information about changes in heart rate and rhythm with exercise. Teaching and obtaining information about home medications (prescribed or complementary) and selecting the best leads for monitoring patients require more critical thinking and should be done by the RN.

A patient's CVP reading suddenly increased from 10 to 48 mm Hg. His lungs are clear except for fine rales at the bases. Immediate response should be which of the following? a. Nothing; this reading is still within normal limits. b. Place a stat call into the physician. c. Administered ordered prn Lasix. d. Check the transducer level.

d. Check the transducer level. If the transducer falls below the correct level, the reading would be falsely elevated. This rise is consistent with a transducer having fallen from the correct level on the bed to the floor. Lasix is not indicated. Central venous pressure (CVP) of 45 mm Hg, if true, is severely elevated. Not enough information has been provided to call the physician. If the CVP value is true and the patient's condition is poor, a call to the physician would be appropriate after assessment.

When caring for a patient with acute coronary syndrome who has returned to the coronary care unit after having angioplasty with stent placement, the nurse obtains the following assessment data. Which data indicate the need forimmediate action by the nurse? a. Heart rate 102 beats/min b. Pedal pulses 1+ bilaterally c. Blood pressure 103/54 mm Hg d. Chest pain level 7 on a 0 to 10 point scale

d. Chest pain level 7 on a 0 to 10 point scale The patient's chest pain indicates that restenosis of the coronary artery may be occurring and requires immediate actions, such as administration of oxygen and nitroglycerin, by the nurse. The other information indicates a need for ongoing assessments by the nurse.

Which medications are not effective in the immediate treatment of acute anaphylaxis? a. Epinephrine b. Vasopressors c. Diphenhydramine (Benadryl) IV d. Corticosteroids

d. Corticosteroids Epinephrine is the first-line treatment of choice for anaphylaxis and should be administered when initial signs and symptoms occur. Several medications are used as second-line adjunctive therapy. Inhaled b-adrenergic agents are used to treat bronchospasm unresponsive to epinephrine. Diphenhydramine (Benadryl) given 1 to 2 mg/kg (25?0-50 mg) by a slow intravenous line is used to block histamine response. Ranitidine, given in conjunction with diphenhydramine at a dose of 1 mg/kg intravenously over 10 to 15 minutes, has been found helpful. Corticosteroids are not effective in the immediate treatment of acute anaphylaxis but may be given with the goal of preventing a prolonged or delayed reaction.

Which of the following hemodynamic parameters supports the diagnosis of cardiogenic shock? a. Increased right atrial pressure b. Decreased pulmonary artery wedge pressure c. Increased cardiac output d. Decreased cardiac index

d. Decreased cardiac index Assessment of the hemodynamic parameter of patients in cardiogenic shock reveals a decreased cardiac output and a cardiac index less than 2.2 L/min/m2.

When caring for the patient with a pulmonary artery (PA) pressure catheter, the nurse observes that the PA waveform indicates that the catheter is in the wedged position. Which action should the nurse take next? a. Zero balance the transducer. b. Activate the fast flush system. c. Notify the health care provider. d. Deflate and reinflate the PA balloon.

d. Deflate and reinflate the PA balloon. When the catheter is in the wedge position, blood flow past the catheter is obstructed, placing the patient at risk for pulmonary infarction. A health care provider or advanced practice nurse should be called to reposition the catheter. The other actions will not correct the wedging of the PA catheter.

While doing the admission assessment for a thin 76-year-old patient, the nurse observes pulsation of the abdominal aorta in the epigastric area. Which action should the nurse take? a. Teach the patient about aneurysms. b. Notify the hospital rapid response team. c. Instruct the patient to remain on bed rest. d. Document the finding in the patient chart.

d. Document the finding in the patient chart. Visible pulsation of the abdominal aorta is commonly observed in the epigastric area for thin individuals. The nurse should simply document the finding in the admission assessment. Unless there are other abnormal findings (such as a bruit, pain, or hyper/hypotension) associated with the pulsation, the other actions are not necessary.

Which of the following drugs promotes bronchodilation and vasoconstriction? a. Solu-Medrol b. Gentamicin c. Atropine d. Epinephrine

d. Epinephrine Epinephrine is given in anaphylactic shock to promote bronchodilation and vasoconstriction and inhibit further release of biochemical mediators

A patient with hyperlipidemia has a new order for colesevelam (Welchol). Which nursing action is most appropriate when giving the medication? a. Have the patient take this medication with an aspirin. b. Administer the medication at the patient's usual bedtime. c. Have the patient take the colesevelam with a sip of water. d. Give the patient's other medications 2 hours after the colesevelam.

d. Give the patient's other medications 2 hours after the colesevelam. The bile acid sequestrants interfere with the absorption of many other drugs, and giving other medications at the same time should be avoided. Taking an aspirin concurrently with the colesevelam may increase the incidence of gastrointestinal side effects such as heartburn. An increased fluid intake is encouraged for patients taking the bile acid sequestrants to reduce the risk for constipation. For maximum effect, colesevelam should be administered with meals.

Which nursing intervention for a patient who had an open repair of an abdominal aortic aneurysm 2 days previously is appropriate for the nurse to delegate to unlicensed assistive personnel (UAP)? a. Monitor the quality and presence of the pedal pulses. b. Teach the patient the signs of possible wound infection. c. Check the lower extremities for strength and movement. d. Help the patient to use a pillow to splint while coughing.

d. Help the patient to use a pillow to splint while coughing. Assisting a patient who has already been taught how to cough is part of routine postoperative care and within the education and scope of practice for UAP. Patient teaching and assessment of essential postoperative functions such as circulation and movement should be done by RNs.

The blood test used to standardize PT results among clinical laboratories worldwide is known as a. aPTT. b. ACT. c. HDL. d. INR.

d. INR. The international normalized ratio was developed by the World Health Organization in 1982 to standardize prothrombin time results among clinical laboratories worldwide. High-density lipoproteins are particles of the total serum cholesterol. Activated coagulation time (ACT) is also known as the activated clotting time. The ACT is a point of care test that is performed outside of the laboratory setting in areas such as the cardiac catheterization laboratory, the operating room, or critical care units. The activated partial thromboplastin time is used to measure the effectiveness of intravenous or subcutaneos ultrafractionated heparin therapy.

When the nurse educator is evaluating the skills of a new registered nurse (RN) caring for patients experiencing shock, which action by the new RN indicates a need for more education? a. Placing the pulse oximeter on the ear for a patient with septic shock b. Keeping the head of the bed flat for a patient with hypovolemic shock c. Increasing the nitroprusside (Nipride) infusion rate for a patient with a high SVR d. Maintaining the room temperature at 66° to 68° F for a patient with neurogenic shock

d. Maintaining the room temperature at 66° to 68° F for a patient with neurogenic shock Patients with neurogenic shock may have poikilothermia. The room temperature should be kept warm to avoid hypothermia. The other actions by the new RN are appropriate.

The nurse responds to a ventilator alarm and finds the patient lying in bed holding the endotracheal tube (ET). Which action should the nurse take next? a. Activate the rapid response team. b. Provide reassurance to the patient. c. Call the health care provider to reinsert the tube. d. Manually ventilate the patient with 100% oxygen.

d. Manually ventilate the patient with 100% oxygen. The nurse should ensure maximal patient oxygenation by manually ventilating with a bag-valve-mask system. Offering reassurance to the patient, notifying the health care provider about the need to reinsert the tube, and activating the rapid response team are also appropriate after the nurse has stabilized the patient's oxygenation.

When the nurse is monitoring a patient who is undergoing exercise (stress) testing on a treadmill, which assessment finding requires the most rapid action by the nurse? a. Patient complaint of feeling tired b. Pulse change from 87 to 101 beats/minute c. Blood pressure (BP) increase from 134/68 to 150/80 mmHg d. Newly inverted T waves on the electrocardiogram

d. Newly inverted T waves on the electrocardiogram ECG changes associated with coronary ischemia (such as T-wave inversions and ST segment depression) indicate that the myocardium is not getting adequate oxygen delivery and that the exercise test should be terminated immediately. Increases in BP and heart rate (HR) are normal responses to aerobic exercise. Feeling tired is also normal as the intensity of exercise increases during the stress testing.

A patient admitted with acute dyspnea is newly diagnosed with dilated cardiomyopathy. Which information will the nurse plan to teach the patient about managing this disorder? a. A heart transplant should be scheduled as soon as possible. b. Elevating the legs above the heart will help relieve dyspnea. c. Careful compliance with diet and medications will prevent heart failure. d. Notify the doctor about any symptoms of heart failure such as shortness of breath.

d. Notify the doctor about any symptoms of heart failure such as shortness of breath. The patient should be instructed to notify the health care provider about any worsening of heart failure symptoms. Because dilated cardiomyopathy does not respond well to therapy, even patients with good compliance with therapy may have recurrent episodes of heart failure. Elevation of the legs above the heart will worsen symptoms (although this approach is appropriate for a patient with hypertrophic cardiomyopathy). The patient with terminal or end-stage cardiomyopathy may consider heart transplantation.

The health care provider has prescribed bed rest with the feet elevated for a patient admitted to the hospital with venous thromboembolism. Which action by the nurse to elevate the patient's feet is best? a. The patient is placed in the Trendelenburg position. b. Two pillows are positioned under the affected leg. c. The bed is elevated at the knee and pillows are placed under the feet. d. One pillow is placed under the thighs and two pillows are placed under the lower legs.

d. One pillow is placed under the thighs and two pillows are placed under the lower legs. The purpose of elevating the feet is to enhance venous flow from the feet to the right atrium, which is best accomplished by placing two pillows under the feet and one under the thighs. Placing the patient in the Trendelenburg position will lower the head below heart level, which is not indicated for this patient. Placing pillows under the calf or elevating the bed at the knee may cause blood stasis at the calf level.

A 66-year-old patient is admitted to the critical care unit with a diagnosis of acute inferior MI. A 12-lead electrocardiogram (ECG) is done to validate the area of infarction. Which finding on the ECG is most conclusive for infarction? a. Inverted T waves b. Tall, peaked T waves c. ST segment depression d. Pathologic Q waves

d. Pathologic Q waves The changes in repolarization are seen by the presence of new Q waves. These new, pathologic Q waves are deeper and wider than tiny Q waves found on the normal 12-lead ECG.

After change-of-shift report, which patient should the progressive care nurse assess first? a. Patient who was extubated in the morning and has a temperature of 101.4° F (38.6° C) b. Patient with bilevel positive airway pressure (BiPAP) for sleep apnea whose respiratory rate is 16 c. Patient with arterial pressure monitoring who is 2 hours post-percutaneous coronary intervention who needs to void d. Patient who is receiving IV heparin for a venous thromboembolism and has a partial thromboplastin time (PTT) of 98 sec

d. Patient who is receiving IV heparin for a venous thromboembolism and has a partial thromboplastin time (PTT) of 98 sec The findings for this patient indicate high risk for bleeding from an elevated (nontherapeutic) PTT. The nurse needs to adjust the rate of the infusion (dose) per the health care provider's parameters. The patient with BiPAP for sleep apnea has a normal respiratory rate. The patient recovering from the percutaneous coronary intervention will need to be assisted with voiding and this task could be delegated to unlicensed assistive personnel. The patient with a fever may be developing ventilator-associated pneumonia, but addressing the bleeding risk is a higher priority.

After change-of-shift report on a ventilator weaning unit, which patient should the nurse assess first? a. Patient who failed a spontaneous breathing trial and has been placed in a rest mode on the ventilator b. Patient who is intubated and has continuous partial pressure end-tidal CO (PETCO ) monitoring c. Patient with a central venous oxygen saturation (ScvO ) of 69% while on bilevel positive airway pressure (BiPAP) d. Patient who was successfully weaned and extubated 4 hours ago and now has no urine output for the last 6 hours

d. Patient who was successfully weaned and extubated 4 hours ago and now has no urine output for the last 6 hours The decreased urine output may indicate acute kidney injury or that the patient's cardiac output and perfusion of vital organs have decreased. Any of these causes would require rapid action. The data about the other patients indicate that their conditions are stable and do not require immediate assessment or changes in their care. Continuous PETCO monitoring is frequently used when patients are intubated. The rest mode should be used to allow patient recovery after a failed SBT, and an ScvO of 69% is within normal limits.

A patient who was admitted with a myocardial infarction experiences a 45-second episode of ventricular tachycardia, then converts to sinus rhythm with a heart rate of 98 beats/minute. Which of the following actions should the nurse take next? a. Immediately notify the health care provider. b. Document the rhythm and continue to monitor the patient. c. Perform synchronized cardioversion per agency dysrhythmia protocol. d. Prepare to give IV amiodarone (Cordarone) per agency dysrhythmia protocol.

d. Prepare to give IV amiodarone (Cordarone) per agency dysrhythmia protocol. The burst of sustained ventricular tachycardia indicates that the patient has significant ventricular irritability, and antidysrhythmic medication administration is needed to prevent further episodes. The nurse should notify the health care provider after the medication is started. Defibrillation is not indicated given that the patient is currently in a sinus rhythm. Documentation and continued monitoring are not adequate responses to this situation.

When analyzing the rhythm of a patient's electrocardiogram (ECG), the nurse will need to investigate further upon finding a(n) a. isoelectric ST segment. b. P-R interval of 0.18 second. c. Q-T interval of 0.38 second. d. QRS interval of 0.14 second.

d. QRS interval of 0.14 second. Because the normal QRS interval is 0.04 to 0.10 seconds, the patient's QRS interval of 0.14 seconds indicates that the conduction through the ventricular conduction system is prolonged. The P-R interval and Q-T interval are within normal range, and ST segment should be isoelectric (flat).

A patient had a non-ST-segment-elevation myocardial infarction (NSTEMI) 3 days ago. Which nursing intervention included in the plan of care is mostappropriate for the registered nurse (RN) to delegate to an experienced licensed practical/vocational nurse (LPN/LVN)? a. Evaluation of the patient's response to walking in the hallway b. Completion of the referral form for a home health nurse follow-up c. Education of the patient about the pathophysiology of heart disease d. Reinforcement of teaching about the purpose of prescribed medications

d. Reinforcement of teaching about the purpose of prescribed medications LPN/LVN education and scope of practice include reinforcing education that has previously been done by the RN. Evaluating the patient response to exercise after a NSTEMI requires more education and should be done by the RN. Teaching and discharge planning/ documentation are higher level skills that require RN education and scope of practice.

Which laboratory result for a patient with multifocal premature ventricular contractions (PVCs) is most important for the nurse to communicate to the health care provider? a. Blood glucose 243 mg/dL b. Serum chloride 92 mEq/L c. Serum sodium 134 mEq/L d. Serum potassium 2.9 mEq/L

d. Serum potassium 2.9 mEq/L Hypokalemia increases the risk for ventricular dysrhythmias such as PVCs, ventricular tachycardia, and ventricular fibrillation. The health care provider will need to prescribe a potassium infusion to correct this abnormality. Although the other laboratory values also are abnormal, they are not likely to be the etiology of the patient's PVCs and do not require immediate correction.

Which classification of dysrhythmia is most common with an inferior wall infarction in the first hour after STEMI? a. Sinus tachycardia b. Multifocal PVCs c. Atrial fibrillation d. Sinus bradycardia

d. Sinus bradycardia Sinus bradycardia (heart rate less than 60 beats/min) occurs in 30% to 40% of patients who sustain an acute myocardial infarction (MI). It is more prevalent with an inferior wall infarction in the first hour after ST segment elevation MI. Sinus tachycardia (heart rate more than 100 beats/min) most often occurs with an anterior wall MI. Premature atrial contractions (PACs) occur frequently in patients who sustain an acute MI. Atrial fibrillation is also common and may occur spontaneously or may be preceded by PACs. Premature ventricular contractions (PVCs) are seen in almost all patients within the first few hours after an MI.

Patient teaching regarding valvular heart disease should include which of the following? a. Increase fluid intake to increase cardiac output. b. Monitor and increase sodium intake to replace sodium lost with diuretics. c. Increase daily activity until shortness of breath occurs. d. Take prophylactic antibiotics before undergoing any invasive procedure.

d. Take prophylactic antibiotics before undergoing any invasive procedure. Education for the patient with acute or chronic heart failure secondary to valvular dysfunction includes (1) information related to diet, (2) fluid restrictions, (3) the actions and side effects of heart failure medications, (4) the need for prophylactic antibiotics before undergoing any invasive procedures such as dental work, and (5) when to call the health care provider to report a negative change in cardiac symptoms.

The nurse educator is evaluating the care that a new registered nurse (RN) provides to a patient receiving mechanical ventilation. Which action by the new RN indicates the need for more education? a. The RN increases the FIO to 100% before suctioning. b. The RN secures a bite block in place using adhesive tape. c. The RN asks for assistance to reposition the endotracheal tube. d. The RN positions the patient with the head of bed at 10 degrees.

d. The RN positions the patient with the head of bed at 10 degrees. The head of the patient's bed should be positioned at 30 to 45 degrees to prevent ventilator-associated pneumonia. The other actions by the new RN are appropriate.

When reviewing the 12-lead electrocardiograph (ECG) for a healthy 79-year-old patient who is having an annual physical examination, what will be of mostconcern to the nurse? a. The PR interval is 0.21 seconds. b. The QRS duration is 0.13 seconds. c. There is a right bundle-branch block. d. The heart rate (HR) is 42 beats/minute.

d. The heart rate (HR) is 42 beats/minute. The resting HR does not change with aging, so the decrease in HR requires further investigation. Bundle-branch block and slight increases in PR interval or QRS duration are common in older individuals because of increases in conduction time through the AV node, bundle of His, and bundle branches.

A nurse is obtaining the history of a patient who reveals that he had an MI 5 years ago. When the admission 12-lead ECG is reviewed, Q waves are noted in leads V3 and V4 only. Which of the following conclusions is most consistent with this situation? a. The patient may have had a posterior wall MI. b. The patient must have had a right ventricular MI. c. The admission 12-lead ECG was done incorrectly. d. The patient may have had an anterior MI.

d. The patient may have had an anterior MI. Not every acute myocardial infarction (MI) results in a pathologic Q wave on the 12-lead electrocardiogram (ECG). When the typical ECG changes are not present, the diagnosis depends on symptomatic clinical presentation, specific cardiac biomarkers (e.g., cTnI, cTnT, CK-MB), and non-ECG diagnostic tests such as cardiac catheterization. Anterior and posterior wall MIs have ST changes, not Q wave changes.

Which information given by a patient admitted with chronic stable angina will help the nurse confirm this diagnosis? a. The patient states that the pain "wakes me up at night." b. The patient rates the pain at a level 3 to 5 (0 to 10 scale). c. The patient states that the pain has increased in frequency over the last week. d. The patient states that the pain "goes away" with one sublingual nitroglycerin tablet.

d. The patient states that the pain "goes away" with one sublingual nitroglycerin tablet. Chronic stable angina is typically relieved by rest or nitroglycerin administration. The level of pain is not a consistent indicator of the type of angina. Pain occurring at rest or with increased frequency is typical of unstable angina.

The nurse is obtaining a health history from a 24-year-old patient with hypertrophic cardiomyopathy (HC). Which information obtained by the nurse is most important? a. The patient has a history of a recent upper respiratory infection. b. The patient has a family history of coronary artery disease (CAD). c. The patient reports using cocaine a "couple of times" as a teenager. d. The patient's 29-year-old brother died from a sudden cardiac arrest.

d. The patient's 29-year-old brother died from a sudden cardiac arrest. About half of all cases of HC have a genetic basis, and it is the most common cause of sudden cardiac death in otherwise healthy young people. The information about the patient's brother will be helpful in planning care (such as an automatic implantable cardioverter-defibrillator [AICD]) for the patient and in counseling other family members. The patient should be counseled against the use of stimulant drugs, but the limited past history indicates that the patient is not at current risk for cocaine use. Viral infections and CAD are risk factors for dilated cardiomyopathy, but not for HC.

Noninvasive emergency pacing is best achieved via the use of which type of temporary pacing? a. Transvenous (endocardial) b. Epicardial c. Transthoracic d. Transcutaneous

d. Transcutaneous Transcutaneous cardiac pacing involves the use of two large skin electrodes, one placed anteriorly and the other posteriorly on the chest, connected to an external pulse generator. It is a rapid, noninvasive procedure that nurses can perform in the emergency setting and is recommended for the treatment of symptomatic bradycardia.

When discussing risk factor modification for a 63-year-old patient who has a 5-cm abdominal aortic aneurysm, the nurse will focus discharge teaching on which patient risk factor? a. Male gender b. Turner syndrome c. Abdominal trauma history d. Uncontrolled hypertension

d. Uncontrolled hypertension All of the factors contribute to the patient's risk, but only hypertension can potentially be modified to decrease the patient's risk for further expansion of the aneurysm.

The nurse needs to quickly estimate the heart rate for a patient with a regular heart rhythm. Which method will be best to use? a. Count the number of large squares in the R-R interval and divide by 300. b. Print a 1-minute electrocardiogram (ECG) strip and count the number of QRS complexes. c. Calculate the number of small squares between one QRS complex and the next and divide into 1500. d. Use the 3-second markers to count the number of QRS complexes in 6 seconds and multiply by 10.

d. Use the 3-second markers to count the number of QRS complexes in 6 seconds and multiply by 10. This is the quickest way to determine the ventricular rate for a patient with a regular rhythm. All the other methods are accurate, but take longer.

A patient at the clinic says, "I have always taken a walk after dinner, but lately my leg cramps and hurts after just a few minutes of starting. The pain goes away after I stop walking, though." The nurse should a. check for the presence of tortuous veins bilaterally on the legs. b. ask about any skin color changes that occur in response to cold. c. assess for unilateral swelling, redness, and tenderness of either leg. d. assess for the presence of the dorsalis pedis and posterior tibial pulses.

d. assess for the presence of the dorsalis pedis and posterior tibial pulses. The nurse should assess for other clinical manifestations of peripheral arterial disease in a patient who describes intermittent claudication. Changes in skin color that occur in response to cold are consistent with Raynaud's phenomenon. Tortuous veins on the legs suggest venous insufficiency. Unilateral leg swelling, redness, and tenderness indicate venous thromboembolism (VTE).

During a physical examination of a 74-year-old patient, the nurse palpates the point of maximal impulse (PMI) in the sixth intercostal space lateral to the left midclavicular line. The most appropriate action for the nurse to take next will be to a. ask the patient about risk factors for atherosclerosis. b. document that the PMI is in the normal anatomic location. c. auscultate both the carotid arteries for the presence of a bruit. d. assess the patient for symptoms of left ventricular hypertrophy.

d. assess the patient for symptoms of left ventricular hypertrophy. The PMI should be felt at the intersection of the fifth intercostal space and the left midclavicular line. A PMI located outside these landmarks indicates possible cardiac enlargement, such as with left ventricular hypertrophy. Cardiac enlargement is not necessarily associated with atherosclerosis or carotid artery disease.

While caring for a 23-year-old patient with mitral valve prolapse (MVP) without valvular regurgitation, the nurse determines that discharge teaching has been effective when the patient states that it will be necessary to a. take antibiotics before any dental appointments. b. limit physical activity to avoid stressing the heart. c. take an aspirin a day to prevent clots from forming on the valve. d. avoid use of over-the-counter (OTC) medications that contain stimulant drugs.

d. avoid use of over-the-counter (OTC) medications that contain stimulant drugs. Use of stimulant medications should be avoided by patients with MVP because these may exacerbate symptoms. Daily aspirin and restricted physical activity are not needed by patients with mild MVP. Antibiotic prophylaxis is needed for patients with MVP with regurgitation but will not be necessary for this patient.

Four hours after mechanical ventilation is initiated for a patient with chronic obstructive pulmonary disease (COPD), the patient's arterial blood gas (ABG) results include a pH of 7.51, PaO2 of 82 mm Hg, PaCO2 of 26 mm Hg, and HCO3- of 23 mEq/L (23 mmol/L). The nurse will anticipate the need to a. increase the FIO2. b. increase the tidal volume. c. increase the respiratory rate. d. decrease the respiratory rate.

d. decrease the respiratory rate. The patient's PaCO and pH indicate respiratory alkalosis caused by too high a respiratory rate. The PaO is appropriate for a patient with COPD and increasing the respiratory rate and tidal volume would further lower the PaCO .

A patient has a normal cardiac rhythm and a heart rate of 72 beats/minute. The nurse determines that the P-R interval is 0.24 seconds. The most appropriate intervention by the nurse would be to a. notify the health care provider immediately. b. give atropine per agency dysrhythmia protocol. c. prepare the patient for temporary pacemaker insertion. d. document the finding and continue to monitor the patient.

d. document the finding and continue to monitor the patient. First-degree atrioventricular (AV) block is asymptomatic and requires ongoing monitoring because it may progress to more serious forms of heart block. The rate is normal, so there is no indication that atropine is needed. Immediate notification of the health care provider about an asymptomatic rhythm is not necessary.

After noting a pulse deficit when assessing a 74-year-old patient who has just arrived in the emergency department, the nurse will anticipate that the patient may require a. emergent cardioversion. b. a cardiac catheterization. c. hourly blood pressure (BP) checks. d. electrocardiographic (ECG) monitoring.

d. electrocardiographic (ECG) monitoring. Pulse deficit is a difference between simultaneously obtained apical and radial pulses. It indicates that there may be a cardiac dysrhythmia that would best be detected with ECG monitoring. Frequent BP monitoring, cardiac catheterization, and emergent cardioversion are used for diagnosis and/or treatment of cardiovascular disorders but would not be as helpful in determining the immediate reason for the pulse deficit.

When developing a teaching plan for a 61-year-old man with the following risk factors for coronary artery disease (CAD), the nurse should focus on the a. family history of coronary artery disease. b. increased risk associated with the patient's gender. c. increased risk of cardiovascular disease as people age. d. elevation of the patient's low-density lipoprotein (LDL) level.

d. elevation of the patient's low-density lipoprotein (LDL) level. Because family history, gender, and age are nonmodifiable risk factors, the nurse should focus on the patient's LDL level. Decreases in LDL will help reduce the patient's risk for developing CAD.

Abnormal heart sounds are labeled S and S and are referred to as __________ when auscultated during a tachycardic episode. a. Korotkoff sounds b. pulse pressure c. murmurs d. gallops

d. gallops Abnormal heart sounds are known as the third heart sound (S3) and the fourth heart sound (S4); they are referred to as gallops when auscultated during an episode of tachycardia. Murmurs are produced by turbulent flood flow through the chambers of the heart, from forward flow through narrowed or irregular valve openings, or backward regurgitate flow through an incompetent valve. Korotkoff sounds are the sounds created by turbulence of blood flow within a vessel caused by constriction of the blood pressure cuff. Pulse pressure describes the difference between systolic and diastolic values. A normal pulse pressure is 40 mm Hg.

A 73-year-old patient with chronic atrial fibrillation develops sudden severe pain, pulselessness, pallor, and coolness in the right leg. The nurse should notify the health care provider and immediately a. apply a compression stocking to the leg. b. elevate the leg above the level of the heart. c. assist the patient in gently exercising the leg. d. keep the patient in bed in the supine position.

d. keep the patient in bed in the supine position. The patient's history and clinical manifestations are consistent with acute arterial occlusion, and resting the leg will decrease the oxygen demand of the tissues and minimize ischemic damage until circulation can be restored. Elevating the leg or applying an elastic wrap will further compromise blood flow to the leg. Exercise will increase oxygen demand for the tissues of the leg.

While waiting for cardiac transplantation, a patient with severe cardiomyopathy has a ventricular assist device (VAD) implanted. When planning care for this patient, the nurse should anticipate a. giving immunosuppressive medications. b. preparing the patient for a permanent VAD. c. teaching the patient the reason for complete bed rest. d. monitoring the surgical incision for signs of infection.

d. monitoring the surgical incision for signs of infection. The insertion site for the VAD provides a source for transmission of infection to the circulatory system and requires frequent monitoring. Patient's with VADs are able to have some mobility and may not be on bed rest. The VAD is a bridge to transplantation, not a permanent device. Immunosuppression is not necessary for nonbiologic devices like the VAD.

A patient was admitted to the ICU 3 days ago with a diagnosis of myocardial infarction. The patient is complaining of increased chest pain when coughing, swallowing, and changing positions. The nurse hears a systolic scratching sound upon auscultation of the apical pulse. The nurse notifies the physician. Based on the symptoms, the physician suspects a(n) a. acute mitral regurgitation. b. aortic insufficiency. c. chronic mitral regurgitation. d. pericardial friction rub.

d. pericardial friction rub. A pericardial friction rub is a sound that can occur within 2 to 7 days after a myocardial infarction. The friction rub results from pericardial inflammation (pericarditis). Classically, a pericardial friction rub is a grating or scratching sound that is both systolic and diastolic, corresponding to cardiac motion within the pericardial sac. Acute mitral regurgitation occurs when the ventricle contracts during systole and a jet of blood is sent in a retrograde manner to the left atrium, causing a sudden increase in left atrial pressure, acute pulmonary edema, and low CO and leading to cardiogenic shock. Chronic mitral regurgitation is auscultated in the mitral area and occurs during systole. It is high pitched and blowing, although the pitch and intensity vary, depending on the degree of regurgitation. As mitral regurgitation progresses, the murmur radiates more widely. Aortic insufficiency is an incompetent aortic valve. If the valve cusps do not maintain this seal, the sound of blood flowing back into the left ventricle during diastole is heard as a decrescendo, high-pitched, blowing murmur.

The nurse has identified a nursing diagnosis of acute pain related to inflammatory process for a patient with acute pericarditis. The priority intervention by the nurse for this problem is to a. teach the patient to take deep, slow breaths to control the pain. b. force fluids to 3000 mL/day to decrease fever and inflammation. c. remind the patient to request opioid pain medication every 4 hours. d. place the patient in Fowler's position, leaning forward on the overbed table.

d. place the patient in Fowler's position, leaning forward on the overbed table. Sitting upright and leaning forward frequently will decrease the pain associated with pericarditis. Forcing fluids will not decrease the inflammation or pain. Taking deep breaths will tend to increase pericardial pain. Opioids are not very effective at controlling pain caused by acute inflammatory conditions and are usually ordered PRN. The patient would receive scheduled doses of a nonsteroidal antiinflammatory drug (NSAID).

When monitoring for the effectiveness of treatment for a patient with a large anterior wall myocardial infarction, the most important information for the nurse to obtain is a. central venous pressure (CVP). b. systemic vascular resistance (SVR). c. pulmonary vascular resistance (PVR). d. pulmonary artery wedge pressure (PAWP).

d. pulmonary artery wedge pressure (PAWP). PAWP reflects left ventricular end diastolic pressure (or left ventricular preload) and is a sensitive indicator of cardiac function. Because the patient is high risk for left ventricular failure, the PAWP must be monitored. An increase will indicate left ventricular failure. The other values would also provide useful information, but the most definitive measurement of changes in cardiac function is the PAWP.

A patient with ventricular fibrillation (VF) is a. hypertensive. b. bradypneic. c. diaphoretic. d. pulseless.

d. pulseless. In VF, the patient does not have a pulse, no blood is being pumped forward, and defibrillation is the only definitive therapy. No forward flow of blood or palpable pulse is present in VF.

Nadolol (Corgard) is prescribed for a patient with chronic stable angina and left ventricular dysfunction. To determine whether the drug is effective, the nurse will monitor for a. decreased blood pressure and heart rate. b. fewer complaints of having cold hands and feet. c. improvement in the strength of the distal pulses. d. the ability to do daily activities without chest pain.

d. the ability to do daily activities without chest pain. Because the medication is ordered to improve the patient's angina, effectiveness is indicated if the patient is able to accomplish daily activities without chest pain. Blood pressure and heart rate may decrease, but these data do not indicate that the goal of decreased angina has been met. The noncardioselective b-adrenergic blockers can cause peripheral vasoconstriction, so the nurse would not expect an improvement in distal pulse quality or skin temperature.

The nurse obtains a rhythm strip on a patient who has had a myocardial infarction and makes the following analysis: no visible P waves, P-R interval not measurable, ventricular rate 162, R-R interval regular, and QRS complex wide and distorted, QRS duration 0.18 second. The nurse interprets the patient's cardiac rhythm as a. atrial flutter. b. sinus tachycardia. c. ventricular fibrillation. d. ventricular tachycardia.

d. ventricular tachycardia. The absence of P waves, wide QRS, rate >150 beats/minute, and the regularity of the rhythm indicate ventricular tachycardia. Atrial flutter is usually regular, has a narrow QRS configuration, and has flutter waves present representing atrial activity. Sinus tachycardia has P waves. Ventricular fibrillation is irregular and does not have a consistent QRS duration.

When assisting with oral intubation of a patient who is having respiratory distress, in which order will the nurse take these actions? (Put a comma and a space between each answer choice [A, B, C, D, E].) a. Obtain a portable chest-x-ray. b. Position the patient in the supine position. c. Inflate the cuff of the endotracheal tube after insertion. d. Attach an end-tidal CO detector to the endotracheal tube. e. Oxygenate the patient with a bag-valve-mask device for several minutes.

e. Oxygenate the patient with a bag-valve-mask device for several minutes. b. Position the patient in the supine position. c. Inflate the cuff of the endotracheal tube after insertion. d. Attach an end-tidal CO detector to the endotracheal tube. a. Obtain a portable chest-x-ray. The patient is pre-oxygenated with a bag-valve-mask system for 3 to 5 minutes before intubation and then placed in a supine position. Following the intubation, the cuff on the endotracheal tube is inflated to occlude and protect the airway. Tube placement is assessed first with an end-tidal CO sensor, then with a chest x-ray.

The health care provider orders the following interventions for a 67-kg patient who has septic shock with a BP of 70/42 mm Hg and oxygen saturation of 90% on room air. In which order will the nurse implement the actions?(Put a comma and a space between each answer choice [A, B, C, D, E].) a. Obtain blood and urine cultures. b. Give vancomycin (Vancocin) 1 g IV. c. Start norepinephrine (Levophed) 0.5 mcg/min. d. Infuse normal saline 2000 mL over 30 minutes. e. Titrate oxygen administration to keep O saturation >95%.

e. Titrate oxygen administration to keep O saturation >95%. d. Infuse normal saline 2000 mL over 30 minutes. c. Start norepinephrine (Levophed) 0.5 mcg/min. a. Obtain blood and urine cultures. b. Give vancomycin (Vancocin) 1 g IV. The initial action for this hypotensive and hypoxemic patient should be to improve the oxygen saturation, followed by infusion of IV fluids and vasopressors to improve perfusion. Cultures should be obtained before administration of antibiotics.

A nurse is providing care to a patient on fibrinolytic therapy. Which of the following statements from the patient warrants further assessment and intervention by the critical care nurse? a. "My back is killing me!" b. "There is blood on my toothbrush!" c. "Look at the bruises on my arms!" d. "My arm is bleeding where my IV is!"

a. "My back is killing me!" The nurse must continually monitor for clinical manifestations of bleeding. Mild gingival bleeding and oozing around venipuncture sites are common and not causes for concern. However, severe lower back pain and ecchymoses are suggestive of retroperitoneal bleeding. If serious bleeding occurs, all fibrinolytic heparin therapies are discontinued, and volume expanders, coagulation factors, or both are administered.

A few days after experiencing a myocardial infarction (MI) and successful percutaneous coronary intervention, the patient states, "I just had a little chest pain. As soon as I get out of here, I'm going for my vacation as planned." Which reply would be most appropriate for the nurse to make? a. "What do you think caused your chest pain?" b. "Where are you planning to go for your vacation?" c. "Sometimes plans need to change after a heart attack." d. "Recovery from a heart attack takes at least a few weeks."

a. "What do you think caused your chest pain?" When the patient is experiencing denial, the nurse should assist the patient in testing reality until the patient has progressed beyond this step of the emotional adjustment to MI. Asking the patient about vacation plans reinforces the patient's plan, which is not appropriate in the immediate post-MI period. Reminding the patient in denial about the MI is likely to make the patient angry and lead to distrust of the nursing staff.

In the acute phase after STEMI, fibrinolytic therapy is used in combination with heparin to recanalize the coronary artery. The initial heparin bolus is a. 60 units/kg maximum 5000 units. b. 30 units/kg maximum 3000 units. c. 25 units/kg maximum of 2500 units. d. 12 units/kg maximum of 1000 units.

a. 60 units/kg maximum 5000 units. In the acute phase after ST segment elevation myocardial infarction, heparin is administered in combination with fibrinolytic therapy to recanalize (open) the coronary artery. For patients who will receive fibrinolytic therapy, an initial heparin bolus of 60 units/kg (maximum, 5000 units) is given intravenously followed by a continuous heparin drip at 12 units/kg/hr (maximum 1000 units/hr) to maintain an activated partial thromboplastin time between 50 and 70 seconds (1.5-2.0 times control).

A patient has been on the medical floor for 1 week after a vaginal hysterectomy. A urinary catheter was inserted. Complete blood cell count results have revealed escalating white blood cell counts. The patient is transferred to the critical care unit when her condition deteriorates. Septic shock is diagnosed. Which of the following is the pathophysiologic mechanism that results in septic shock? a. Bacterial toxins lead to vasodilation. b. Increased white blood cells are released to fight invading bacteria. c. Microorganisms invade organs such as the kidneys and heart. d. An increase of white blood cells leads to decreased red blood cell production and anemia

a. Bacterial toxins lead to vasodilation. The syndrome encompassing severe sepsis and septic shock is a complex systemic response that is initiated when a microorganism enters the body and stimulates the inflammatory or immune system. Shed protein fragments and the release of toxins and other substances from the microorganism activate the plasma enzyme cascades (complement, kinin and kallikrein, coagulation, and fibrinolytic factors), as well as platelets, neutrophils, monocytes, and macrophages.

The key to treatment of septic shock is finding the cause of the infection. Which of the following cultures are obtained before antibiotic therapy is initiated? (Select all that apply.) a. Blood cultures x 2 b. Wound cultures c. Urine cultures d. Sputum cultures e. CBC with differential

a. Blood cultures x 2 b. Wound cultures c. Urine cultures d. Sputum cultures A key measure in the treatment of septic shock is finding and eradicating the cause of the infection. At least two blood cultures plus urine, sputum, and wound cultures should be obtained to find the location of the infection before antibiotic therapy is initiated. Antibiotic therapy should be started within 1 hour of recognition of severe sepsis without delay for cultures.

A patient has been on the medical floor for 1 week after a vaginal hysterectomy. A urinary catheter was inserted. Complete blood cell count results have revealed escalating white blood cell counts. The patient is transferred to the critical care unit when her condition deteriorates. Septic shock is diagnosed. A pulmonary artery catheter is placed. Which of the following hemodynamic values would you expect to find? a. Cardiac output of 8 L/min b. Right atrial pressure of 17 mm Hg c. Pulmonary artery wedge pressure of 23 mm Hg d. Systemic vascular resistance of 1100

a. Cardiac output of 8 L/min Increased cardiac output and decreased systemic vascular resistance are classic signs of septic shock.

Which of the following conditions is usually associated with clubbing? a. Central cyanosis b. Peripheral cyanosis c. Carbon monoxide poisoning d. Acute hypoxemia

a. Central cyanosis Clubbing in the nail bed is a sign associated with longstanding central cyanotic heart disease or pulmonary disease withhypoxemia. Peripheral cyanosis, a bluish discoloration of the nail bed, is seen more commonly. Peripheral cyanosis results from a reduction in the quantity of oxygen in the peripheral extremities from arterial disease or decreased cardiac output. Central cyanosis is a bluish discoloration of the tongue and sublingual area. Multiracial studies indicate that the tongue is the most sensitive site for observation of central cyanosis.

A 46-year-old is diagnosed with thromboangiitis obliterans (Buerger's disease). When the nurse is developing a discharge teaching plan for the patient, which outcome has the highest priority for this patient? a. Cessation of all tobacco use b. Control of serum lipid levels c. Maintenance of appropriate weight d. Demonstration of meticulous foot care

a. Cessation of all tobacco use Absolute cessation of nicotine use is needed to reduce the risk for amputation in patients with Buerger's disease. Other therapies have limited success in treatment of this disease.

A patient's bedside ECG strips show the following changes: increased PR interval; increased QRS width; and tall, peaked T waves. Vital signs are T 98.2° F; HR 118 beats/min; BP 146/90 mm Hg; and RR 18 breaths/min. The patient is receiving the following medications: digoxin 0.125 mg PO every day; D5 1/2 normal saline with 40 mEq potassium chloride at 125 mL/hr; Cardizem at 30 mg PO q8h; and aldosterone at 300 mg PO q12h. The physician is notified of the ECG changes. What orders should the nurse expect to receive? a. Change IV fluid to D5 1/2 normal saline and draw blood chemistry. b. Give normal saline with 40 mEq of potassium chloride over a 6-hour period. c. Hold digoxin and draw serum digoxin level. d. Hold Cardizem and give 500 mL normal saline fluid challenge over a 2-hour period.

a. Change IV fluid to D5 1/2 normal saline and draw blood chemistry. The electrocardiographic (ECG) changes are most consistent with hyperkalemia. Removing the potassium from the intravenous line and drawing laboratory values to check the potassium level is the best choice with the least chance of further harm. Digoxin toxicity can be suspected related to the prolonged PR interval, but hyperkalemia explains all the ECG changes. The patient is not hypotensive or bradycardic, so holding the Cardizem is not indicated.

Which of the following are mechanisms responsible for a myocardial infarction (MI)? (Select all that apply.) a. Coronary artery thrombosis b. Plaque rupture c. Coronary artery spasm near the ruptured plaque d. Preinfarction angina e. Hyperlipidemia

a. Coronary artery thrombosis b. Plaque rupture c. Coronary artery spasm near the ruptured plaque The three mechanisms that block the coronary artery and are responsible for the acute reduction in oxygen delivery to the myocardium are (1) plaque rupture, (2) new coronary artery thrombosis, and (3) coronary artery spasm close to the ruptured plaque.

The nurse is caring for a patient who has an intraortic balloon pump (IABP) following a massive heart attack. When assessing the patient, the nurse notices blood backing up into the IABP catheter. In which order should the nurse take the following actions? (Put a comma and a space between each answer choice [A, B, C, D].) a. Ensure that the IABP console has turned off. b. Assess the patient's vital signs and orientation. c. Obtain supplies for insertion of a new IABP catheter. d. Notify the health care provider of the IABP malfunction.

a. Ensure that the IABP console has turned off. b. Assess the patient's vital signs and orientation. d. Notify the health care provider of the IABP malfunction. c. Obtain supplies for insertion of a new IABP catheter. Blood in the IABP catheter indicates a possible tear in the balloon. The console will shut off automatically to prevent complications such as air embolism. Next, the nurse will assess the patient and communicate with the health care provider about the patient's assessment and the IABP problem. Finally, supplies for insertion of a new IABP catheter may be needed, based on the patient assessment and the decision of the health care provider.

Which action by a new registered nurse (RN) who is orienting to the progressive care unit indicates a good understanding of the treatment of cardiac dysrhythmias? a. Injects IV adenosine (Adenocard) over 2 seconds to a patient with supraventricular tachycardia b. Obtains the defibrillator and quickly brings it to the bedside of a patient whose monitor shows asystole c. Turns the synchronizer switch to the "on" position before defibrillating a patient with ventricular fibrillation d. Gives the prescribed dose of diltiazem (Cardizem) to a patient with newonset type II second degree AV block

a. Injects IV adenosine (Adenocard) over 2 seconds to a patient with supraventricular tachycardia Adenosine must be given over 1 to 2 seconds to be effective. The other actions indicate a need for more education about treatment of cardiac dysrhythmias. The RN should hold the diltiazem until talking to the health care provider. The treatment for asystole is immediate CPR. The synchronizer switch should be "off" when defibrillating.

Which action will the nurse include in the plan of care for a patient who was admitted with syncopal episodes of unknown origin? a. Instruct the patient to call for assistance before getting out of bed. b. Explain the association between various dysrhythmias and syncope. c. Educate the patient about the need to avoid caffeine and other stimulants. d. Tell the patient about the benefits of implantable cardioverter-defibrillators.

a. Instruct the patient to call for assistance before getting out of bed. A patient with fainting episodes is at risk for falls. The nurse will plan to minimize the risk by having assistance whenever the patient up. The other actions may be needed if dysrhythmias are found to be the cause of the patient's syncope, but are not appropriate for syncope of unknown origin.

A patient's cardiac monitor shows a pattern of undulations of varying contours and amplitude with no measurable ECG pattern. The patient is unconscious and pulseless. Which action should the nurse take first? a. Perform immediate defibrillation. b. Give epinephrine (Adrenalin) IV. c. Prepare for endotracheal intubation. d. Give ventilations with a bag-valve-mask device.

a. Perform immediate defibrillation. The patient's rhythm and assessment indicate ventricular fibrillation and cardiac arrest; the initial action should be to defibrillate. If a defibrillator is not immediately available or is unsuccessful in converting the patient to a better rhythm, the other actions may be appropriate.

With anaphylactic shock, which mechanism results in a decreased cardiac output? a. Peripheral vasodilation b. Increased cardiac output c. Decreased alveolar ventilation d. Fluid retention resulting in congestive heart failure

a. Peripheral vasodilation Peripheral vasodilation results in decreased venous return. This decreases intravascular volume and the development of relative hypovolemia. Decreased venous return results in decreased stroke volume and a fall in cardiac output.

A patient who is being admitted to the emergency department with intermittent chest pain gives the following list of medications to the nurse. Which medication has the most immediate implications for the patient's care? a. Sildenafil (Viagra) b. Furosemide (Lasix) c. Captopril (Capoten) d. Warfarin (Coumadin)

a. Sildenafil (Viagra) The nurse will need to avoid giving nitrates to the patient because nitrate administration is contraindicated in patients who are using sildenafil because of the risk of severe hypotension caused by vasodilation. The other home medications also should be documented and reported to the health care provider but do not have as immediate an impact on decisions about the patient's treatment.

A patient in the outpatient clinic has a new diagnosis of peripheral artery disease (PAD). Which group of medications will the nurse plan to include when providing patient teaching about PAD management? a. Statins b. Antibiotics c. Thrombolytics d. Anticoagulants

a. Statins Current research indicates that statin use by patients with PAD improves multiple outcomes. There is no research that supports the use of the other medication categories in PAD.

A physician orders removal of the central venous catheter (CVC) line. The patient has a diagnosis of heart failure with chronic obstructive pulmonary disease. The nurse would place the patient in what position for this procedure? a. Supine in bed b. Supine in a chair c. Flat in bed d. Reverse Trendelenburg position

a. Supine in bed Recommended techniques to avoid air embolus during CVC removal include removing the catheter when the patient is supine in bed (not in a chair) and placing the patient flat or in the reverse Trendelenburg position if the patient's clinical condition permits this maneuver. Patients with heart failure, pulmonary disease, and neurologic conditions with raised intracranial pressure should not be placed flat.

Which of the following clinical manifestations is not suggestive of systemic inflammatory response syndrome (SIRS)? a. Temperature of 37.5° C b. Heart rate of 95 beats/min c. Respiratory rate of 24 breath/min d. White blood cell (WBC) count of 15,000 cells/mm3

a. Temperature of 37.5° C SIRS occurs when two or more of four clinical manifestations are present in the patient at high risk. These manifestations are temperature less than 36° C or greater than 38° C, heart rate greater than 90 beats/min, respiratory rate greater than 20 breaths/min or PaCO less than 32 mm Hg, or WBC greater than 12,000 cells/mm or less than 4000 cells/mm or greater than 10% immature (band forms).

After teaching a patient with newly diagnosed Raynaud's phenomenon about how to manage the condition, which action by the patient demonstrates that the teaching has been effective? a. The patient exercises indoors during the winter months. b. The patient places the hands in hot water when they turn pale. c. The patient takes pseudoephedrine (Sudafed) for cold symptoms. d. The patient avoids taking nonsteroidal antiinflammatory drugs (NSAIDs).

a. The patient exercises indoors during the winter months. Patients should avoid temperature extremes by exercising indoors when it is cold. To avoid burn injuries, the patient should use warm, rather than hot, water to warm the hands. Pseudoephedrine is a vasoconstrictor, and should be avoided. There is no reason to avoid taking NSAIDs with Raynaud's phenomenon.

Norepinephrine (Levophed) has been prescribed for a patient who was admitted with dehydration and hypotension. Which patient data indicate that the nurse should consult with the health care provider before starting the norepinephrine? a. The patient's central venous pressure is 3 mm Hg. b. The patient is in sinus tachycardia at 120 beats/min. c. The patient is receiving low dose dopamine (Intropin). d. The patient has had no urine output since being admitted.

a. The patient's central venous pressure is 3 mm Hg. Adequate fluid administration is essential before administration of vasopressors to patients with hypovolemic shock. The patient's low central venous pressure indicates a need for more volume replacement. The other patient data are not contraindications to norepinephrine administration.

Which data collected by the nurse caring for a patient who has cardiogenic shock indicate that the patient may be developing multiple organ dysfunction syndrome (MODS)? a. The patient's serum creatinine level is elevated. b. The patient complains of intermittent chest pressure. c. The patient's extremities are cool and pulses are weak. d. The patient has bilateral crackles throughout lung fields.

a. The patient's serum creatinine level is elevated. The elevated serum creatinine level indicates that the patient has renal failure as well as heart failure. The crackles, chest pressure, and cool extremities are all consistent with the patient's diagnosis of cardiogenic shock.

When preparing to defibrillate a patient. In which order will the nurse perform the following steps? (Put a comma and a space between each answer choice [A, B, C, D, E].) a. Turn the defibrillator on. b. Deliver the electrical charge. c. Select the appropriate energy level. d. Place the paddles on the patient's chest. e. Check the location of other staff and call out "all clear."

a. Turn the defibrillator on. c. Select the appropriate energy level. d. Place the paddles on the patient's chest. e. Check the location of other staff and call out "all clear." b. Deliver the electrical charge. This order will result in rapid defibrillation without endangering hospital staff,

A transvenous pacemaker is inserted through the right subclavian vein and threaded into the right ventricle. The pacemaker is placed on demand at a rate of 70. The pacing mode for this pacemaker is a. VVI. b. AOO. c. DDD. d. VAT

a. VVI. The original code is based on three categories, each represented by a letter. The first letter refers to the cardiac chamber that is paced. The second letter designates which chamber is sensed, and the third letter indicates the pacemaker's response to the sensed event. A VVI pacemaker paces the ventricle when the pacemaker fails to sense an intrinsic ventricular depolarization.

A patient who is orally intubated and receiving mechanical ventilation is anxious and is "fighting" the ventilator. Which action should the nurse take next? a. Verbally coach the patient to breathe with the ventilator. b. Sedate the patient with the ordered PRN lorazepam (Ativan). c. Manually ventilate the patient with a bag-valve-mask device. d. Increase the rate for the ordered propofol (Diprivan) infusion.

a. Verbally coach the patient to breathe with the ventilator. The initial response by the nurse should be to try to decrease the patient's anxiety by coaching the patient about how to coordinate respirations with the ventilator. The other actions may also be helpful if the verbal coaching is ineffective in reducing the patient's anxiety.

A patient's vital signs are pulse 87, respirations 24, and BP of 128/64 mm Hg and cardiac output is 4.7 L/min. The patient's stroke volume is _____ mL.(Round to the nearest whole number.)

54 Stroke volume = cardiac output/heart rate

A 67-year-old patient is admitted to the hospital with a diagnosis of venous insufficiency. Which patient statement is most supportive of the diagnosis? a. "I can't get my shoes on at the end of the day." b. "I can't seem to ever get my feet warm enough." c. "I have burning leg pains after I walk two blocks." d. "I wake up during the night because my legs hurt."

a. "I can't get my shoes on at the end of the day." Because the edema associated with venous insufficiency increases when the patient has been standing, shoes will feel tighter at the end of the day. The other patient statements are characteristic of peripheral artery disease (PAD).

Zeroing the pressure transducer on hemodynamic monitoring equipment occurs when the displays reads a. 0. b. 250. c. 600. d. 760.

a. 0 The monitor is adjusted so that "0" is displayed, which equals atmospheric pressure. Atmospheric pressure is not zero; it is 760 mm Hg at sea level. Using zero to represent current atmospheric pressure provides a convenient baseline for hemodynamic measurement purposes.

Which of the following historical findings would indicate a high risk for latex allergy? (Select all that apply.) a. Allergic reaction to anesthetics b. Eczema of the hands c. Congenital urologic disorder d. Asthma e. Health care worker

a. Allergic reaction to anesthetics b. Eczema of the hands c. Congenital urologic disorder e. Health care worker Prevention of anaphylactic shock is one of the primary responsibilities of nurses in critical care areas. Preventive measures include the identification of patients at risk and cautious assessment of each patient's response to the administration of medications, blood, and blood products. A complete and accurate history of each patient's allergies is an essential component of preventive nursing care. In addition to a list of the allergies, a detailed description of the type of response for each one should be obtained.

Which of the following clinical manifestations are indicative of left ventricular failure? (Select all that apply.) a. Cool, pale extremities b. Jugular venous distention c. Liver tenderness d. Weak peripheral pulses e. Rales

a. Cool, pale extremities d. Weak peripheral pulses e. Rales Patients presenting with left ventricular failure have one of the following: (1) decreased exercise tolerance, (2) fluid retention, or (3) discovery during examination of noncardiac problems. Clinical manifestations of left ventricular failure include decreased peripheral perfusion with weak or diminished pulses; cool, pale extremities; and, in later stages, peripheral cyanosis.

Which of the following has become the first-line hemodynamic assessment tool in the critical care unit? a. Echocardiogram b. ECG c. Exercise stress test d. 24-hour Holter monitor

a. Echocardiogram Echocardiography is quickly becoming a first-line hemodynamic assessment tool in critical care units. Echocardiography is used to detect structural heart abnormalities such as mitral valve stenosis and regurgitation, prolapse of mitral valve leaflets, aortic stenosis and insufficiency, hypertrophic cardiomyopathy, atrial septal defect, thoracic aortic dissection, cardiac tamponade, and pericardial effusion.

Evidence-based guidelines for the treatment of septic shock include which of the following? (Select all that apply.) a. Fluid resuscitation to maintain central venous pressure at 8 mm Hg or greater b. Low-dose dopamine for renal protection c. High-dose corticosteroids d. Administration of activated protein C e. Achieve central venous oxygen saturation of 70% or more

a. Fluid resuscitation to maintain central venous pressure at 8 mm Hg or greater d. Administration of activated protein C e. Achieve central venous oxygen saturation of 70% or more There is no evidence to support the use of dopamine; low-dose steroids are part of the sepsis management bundle.

A patient who has recently started taking pravastatin (Pravachol) and niacin (Nicobid) reports the following symptoms to the nurse. Which is most important to communicate to the health care provider? a. Generalized muscle aches and pains b. Dizziness when changing positions quickly c. Nausea when taking the drugs before eating d. Flushing and pruritus after taking the medications

a. Generalized muscle aches and pains Muscle aches and pains may indicate myopathy and rhabdomyolysis, which have caused acute kidney injury and death in some patients who have taken the statin medications. These symptoms indicate that the pravastatin may need to be discontinued. The other symptoms are common side effects when taking niacin, and although the nurse should follow-up with the health care provider, they do not indicate that a change in medication is needed.

A 78-kg patient with septic shock has a urine output of 30 mL/hr for the past 3 hours. The pulse rate is 120/minute and the central venous pressure and pulmonary artery wedge pressure are low. Which order by the health care provider will the nurse question? a. Give PRN furosemide (Lasix) 40 mg IV. b. Increase normal saline infusion to 250 mL/hr. c. Administer hydrocortisone (Solu-Cortef) 100 mg IV. d. Titrate norepinephrine (Levophed) to keep systolic BP >90 mm Hg.

a. Give PRN furosemide (Lasix) 40 mg IV. Furosemide will lower the filling pressures and renal perfusion further for the patient with septic shock. The other orders are appropriate.

A 66-year-old patient is admitted to the critical care unit with a diagnosis of acute inferior MI. A 12-lead electrocardiogram (ECG) is done to validate the area of infarction. For the above patient, which leads on the ECG would correlate with an inferior wall MI? a. II, III, aVF b. V5 to V6, I, aVL c. V2 to V4 d. V1 to V2

a. II, II, aVF Inferior infarctions are manifested by electrocardiographic (ECG) changes in leads II, III, and aVF. Lateral wall infarctions are manifested by ECG changes in leads V5 to V6 , I, and aVL. Anterior wall infarctions are manifested by ECG changes in leads V2 to V4. Posterior wall infarctions are manifested by ECG changes in leads V1 to V2 .I, aVF

After the nurse gives IV atropine to a patient with symptomatic type 1, second-degree atrioventricular (AV) block, which finding indicates that the medication has been effective? a. Increase in the patient's heart rate b. Increase in strength of peripheral pulses c. Decrease in premature atrial contractions d. Decrease in premature ventricular contractions

a. Increase in the patient's heart rate Atropine will increase the heart rate and conduction through the AV node. Because the medication increases electrical conduction, not cardiac contractility, the quality of the peripheral pulses is not used to evaluate the drug effectiveness. The patient does not have premature atrial or ventricular contractions.

Which of the following is most indicative of decreased left ventricular preload? a. Increased PAOP/PAWP b. Decreased PAOP/PAWP c. Increased CVP d. Decreased CVP

a. Increased PAOP/PAWP Pulmonary artery occlusion pressure (PAOP) and pulmonary artery wedge pressure (PAWP) normally reflect the pressure in the left ventricle at the end of diastole. Left ventricular end-diastolic pressure is preload, and so an increase in preload will first increase the PAOP and PCWP measurements. Although central venous pressure increases in severe cases of increased preload, it can increase for other reasons. The other changes are not associated with increased preload.

Three days after experiencing a myocardial infarction (MI), a patient who is scheduled for discharge asks for assistance with hygiene activities, saying, "I am too nervous to take care of myself." Based on this information, which nursing diagnosis is appropriate? a. Ineffective coping related to anxiety b. Activity intolerance related to weakness c. Denial related to lack of acceptance of the MI d. Disturbed personal identity related to understanding of illness

a. Ineffective coping related to anxiety The patient data indicate that ineffective coping after the MI caused by anxiety about the impact of the MI is a concern. The other nursing diagnoses may be appropriate for some patients after an MI, but the data for this patient do not support denial, activity intolerance, or disturbed personal identity.

A nurse is caring for a patient with shock of unknown etiology whose hemodynamic monitoring indicates BP 92/54, pulse 64, and an elevated pulmonary artery wedge pressure. Which collaborative intervention ordered by the health care provider should the nurse question? a. Infuse normal saline at 250 mL/hr. b. Keep head of bed elevated to 30 degrees. c. Hold nitroprusside (Nipride) if systolic BP <90 mm Hg. d. Titrate dobutamine (Dobutrex) to keep systolic BP >90 mm Hg.

a. Infuse normal saline at 250 mL/hr. The patient's elevated pulmonary artery wedge pressure indicates volume excess. A saline infusion at 250 mL/hr will exacerbate the volume excess. The other actions are appropriate for the patient.

An 82-year-old patient is admitted into the critical care unit with a diagnosis of left-sided heart failure related to mitral stenosis. Physical assessment findings reveal tachycardia with an S and a 3/6 systolic murmur. The grading of a murmur as a 3/6 refers to which of the following characteristics of murmurs? a. Intensity b. Quality c. Timing d. Pitch

a. Intensity Intensity, or the "loudness," is graded on a scale of 1 to 6; the higher the number, the louder the murmur.

Heart murmurs are characterized by which of the following criteria? (Select all that apply.) a. Intensity b. Location c. Quality d. Pitch e. Pathologic cause

a. Intensity b. Location c. Quality d. Pitch Murmurs are characterized by specific criteria: Timing is the place in the cardiac cycle (systole/diastole). Location is where it is auscultated on the chest wall (mitral or aortic area). Radiation is how far the sound spreads across chest wall. Quality is whether the murmur is blowing, grating, or harsh. Pitch is whether the tone is high or low. Intensity is the loudness is graded on a scale of 1 through 6; the higher the number, the louder the murmur.

_____ are created by the turbulence of blood flow through a vessel caused by constriction of the blood pressure cuff. a. Korotkoff sounds b. Pulse pressures c. Murmurs d. Gallops

a. Korotkoff sounds Korotkoff sounds are the sounds created by turbulence of blood flow within a vessel caused by constriction of the blood pressure cuff. Pulse pressure describes the difference between systolic and diastolic values. The normal pulse pressure is 40 mm Hg. Abnormal heart sounds are known as the third heart sound (S3) and the fourth heart sound (S4); they are referred to as gallops when auscultated during an episode of tachycardia. Murmurs are produced by turbulent blood flow through the chambers of the heart, from forward flow through narrowed or irregular valve openings, or backward regurgitate flow through an incompetent valve.

The patient's admitting 12-lead ECG shows wide, M-shaped P waves. Which of the following admitting diagnoses could be responsible for this finding? a. Mitral stenosis b. Chronic pulmonary disease c. Hypotension d. Pericarditis

a. Mitral stenosis Wide, M-shaped P waves are seen in left atrial hypertrophy and are called P mitrale because left atrial hypertrophy is often caused by mitral stenosis. On an electrocardiogram, ventricular fibrillation appears as a continuous, undulating pattern without clear P, QRS, or T waves.

Which information about a patient who has been receiving thrombolytic therapy for an acute myocardial infarction (AMI) is most important for the nurse to communicate to the health care provider? a. No change in the patient's chest pain b. An increase in troponin levels from baseline c. A large bruise at the patient's IV insertion site d. A decrease in ST-segment elevation on the electrocardiogram

a. No change in the patient's chest pain Continued chest pain suggests that the thrombolytic therapy is not effective and that other interventions such as percutaneous coronary intervention (PCI) may be needed. Bruising is a possible side effect of thrombolytic therapy, but it is not an indication that therapy should be discontinued. The decrease of the ST-segment elevation indicates that thrombolysis is occurring and perfusion is returning to the injured myocardium. An increase in troponin levels is expected with reperfusion and is related to the washout of cardiac markers into the circulation as the blocked vessel is opened.

A patient's arterial line waveform has become damped. The nurse should a. check for kinks, blood, and air bubbles in the tubing. b. prepare for a normal saline fluid challenge for hypotension. c. discontinue the arterial line. d. check the patient's lung sounds.

a. check for kinks, blood, and air bubbles in the tubing. A damped waveform occurs when communication from the artery to the transducer is interrupted and produces false values on the monitor and oscilloscope. Damping is caused by a fibrin "sleeve" that partially occludes the tip of the catheter, by kinks in the catheter or tubing, or by air bubbles in the system.

When assessing a patient with PVCs, the nurse knows that the ectopic beat is multifocal because it appears a. in various shapes in the same lead. b. with increasing frequency. c. to widen the QRS width. d. in a specific pattern in the same lead.

a. in various shapes in the same lead. If the ventricular ectopic beats are of various shapes in the same lead, they are multifocal. Multifocal ventricular ectopics are more serious than unifocal ventricular ectopics because they indicate a greater area of irritable myocardial tissue and are more likely to deteriorate into ventricular tachycardia or fibrillation.

The standard policy on the cardiac unit states, "Notify the health care provider for mean arterial pressure (MAP) less than 70 mm Hg." The nurse will need to call the health care provider about the a. postoperative patient with a BP of 116/42. b. newly admitted patient with a BP of 150/87. c. patient with left ventricular failure who has a BP of 110/70. d. patient with a myocardial infarction who has a BP of 140/86.

a. postoperative patient with a BP of 116/42. The mean arterial pressure (MAP) is calculated using the formula MAP = (systolic BP + 2 diastolic BP)/3. The MAP for the postoperative patient in answer 3 is 67. The MAP in the other three patients is higher than 70 mm Hg.

The mechanism of dilation with percutaneous transluminal coronary angioplasty (PTCA) is a. stretching of the vessel wall, resulting in fracture of the plaque. b. anticoagulation after the completion of the procedure, enhancing dilation. c. plaque removal after balloon inflation. d. compression of plaque against the vessel wall.

a. stretching of the vessel wall, resulting in fracture of the plaque. Percutaneous transluminal coronary angioplasty involves the use of a balloon-tipped catheter that, when advanced through an atherosclerotic lesion (atheroma), can be inflated intermittently for the purpose of dilating the stenotic area and improving blood flow through it. The high balloon-inflation pressure stretches the vessel wall, fractures the plaque, and enlarges the vessel lumen.

The most common site for sepsis and septic shock is a. the respiratory system. b. the gastrointestinal system. c. the genitourinary system. d. the circulatory system.

a. the respiratory system. The respiratory system is the most common site of infection producing severe sepsis and septic shock followed by the genitourinary and gastrointestinal systems.

One of the reasons that contributes to higher mortality rates from acute MI in women is: a. waiting longer to seek medical care. b. being younger when symptoms occur. c. risk factors associated with MI are more stronger in women than men. d. women have smaller hearts than men.

a. waiting longer to seek medical care. Risk factors associated with acute myocardial infarction (MI) more strongly in women than in men include hypertension, diabetes mellitus, alcohol intake, and physical inactivity. Many reasons contribute to higher mortality rates from acute MI in women, and these include waiting longer to seek medical care, having smaller coronary arteries, being older when symptoms occur, and experiencing very different symptoms from those of men of the same age.

The nurse is admitting a patient with possible rheumatic fever. Which question on the admission health history will be most pertinent to ask? a. "Do you use any illegal IV drugs?" b. "Have you had a recent sore throat?" c. "Have you injured your chest in the last few weeks?" d. "Do you have a family history of congenital heart disease?"

b. "Have you had a recent sore throat?" Rheumatic fever occurs as a result of an abnormal immune response to a streptococcal infection. Although illicit IV drug use should be discussed with the patient before discharge, it is not a risk factor for rheumatic fever, and would not be as pertinent when admitting the patient. Family history is not a risk factor for rheumatic fever. Chest injury would cause musculoskeletal chest pain rather than rheumatic fever.

On returning from the cardiac catheterization laboratory, the patient asks if he can get up in the chair. The most appropriate response would be a. "You cannot get up because you may pass out." b. "You cannot get up because you may start bleeding." c. "You cannot get up because you may fall." d. "You cannot get up until you urinate."

b. "You cannot get up because you may start bleeding." After catheterization, the patient remains flat for up to 6 hours (varies by institutional protocol and catheter size) to allow the femoral arterial puncture site to form a stable clot. Most bleeding occurs within the first 2 to 3 hours after the procedure.

A patient with rheumatic fever has subcutaneous nodules, erythema marginatum, and polyarthritis. Based on these findings, which nursing diagnosis would be most appropriate? a. Pain related to permanent joint fixation b. Activity intolerance related to arthralgia c. Risk for infection related to open skin lesions d. Risk for impaired skin integrity related to pruritus

b. Activity intolerance related to arthralgia The patient's joint pain will lead to difficulty with activity. The skin lesions seen in rheumatic fever are not open or pruritic. Although acute joint pain will be a problem for this patient, joint inflammation is a temporary clinical manifestation of rheumatic fever and is not associated with permanent joint changes.

The P wave represents which of the following? a. Atrial contraction b. Atrial depolarization c. Sinus node discharge d. Ventricular contraction

b. Atrial depolatization The P wave is an electrical event and represents atrial depolarization. Atrial contraction should accompany the P wave but does not always. The sinus node discharge is too faint to be recorded on the surface electrocardiogram. Ventricular contraction usually accompanies the QRS complex.

When admitting a patient with a non-ST-segment-elevation myocardial infarction (NSTEMI) to the intensive care unit, which action should the nurse perform first? a. Obtain the blood pressure. b. Attach the cardiac monitor. c. Assess the peripheral pulses. d. Auscultate the breath sounds.

b. Attach the cardiac monitor. Because dysrhythmias are the most common complication of myocardial infarction (MI), the first action should be to place the patient on a cardiac monitor. The other actions also are important and should be accomplished as quickly as possible.

When evaluating a patient with a central venous catheter, the nurse observes that the insertion site is red and tender to touch and the patient's temperature is 101.8° F. What should the nurse plan to do next? a. Give analgesics and antibiotics as ordered. b. Discontinue the catheter and culture the tip. c. Change the flush system and monitor the site. d. Check the site more frequently for any swelling.

b. Discontinue the catheter and culture the tip. The information indicates that the patient has a local and systemic infection caused by the catheter, and the catheter should be discontinued. Changing the flush system, giving analgesics, and continued monitoring will not help prevent or treat the infection. Administration of antibiotics is appropriate, but the line should still be discontinued to avoid further complications such as endocarditis.

A patient with cardiogenic shock has the following vital signs: BP 102/50, pulse 128, respirations 28. The pulmonary artery wedge pressure (PAWP) is increased and cardiac output is low. The nurse will anticipate an order for which medication? a. 5% human albumin b. Furosemide (Lasix) IV c. Epinephrine (Adrenalin) drip d. Hydrocortisone (Solu-Cortef)

b. Furosemide (Lasix) IV The PAWP indicates that the patient's preload is elevated, and furosemide is indicated to reduce the preload and improve cardiac output. Epinephrine would further increase heart rate and myocardial oxygen demand. 5% human albumin would also increase the PAWP. Hydrocortisone might be considered for septic or anaphylactic shock.

Which of the following would indicate successful reperfusion after administration of a fibrinolytic agent? (Select all that apply.) a. Gradual decrease in chest pain b. Intermittent, multifocal premature ventricular contractions c. Rapid resolution of ST elevation d. Rapid rise in creatine kinase MB fraction

b. Intermittent, multifocal premature ventricular contractions c. Rapid resolution of ST elevation d. Rapid rise in creatine kinase MB fraction A reliable indicator of reperfusion is the appearance of various reperfusion dysrhythmias such as premature ventricular contractions, bradycardia, heart block, and ventricular tachycardia. Rapid resolution of the previously elevated ST segment should occur. The serum concentration of creatine kinase rises rapidly and markedly, a phenomenon termed "washout".

When assessing the pulmonary arterial waveform, the nurse notices dampening. After tightening the stopcocks and flushing the line, the nurse decides to calibrate the transducer. What are two essential components included in calibration? a. Obtaining a baseline blood pressure and closing the transducer to air b. Leveling the air-fluid interface to the phlebostatic axis and opening the transducer to air c. Having the patient lay flat and closing the transducer to air d. Obtaining blood return on line and closing all stopcocks

b. Leveling the air-fluid interface to the phlebostatic axis and opening the transducer to air Ensuring accuracy of waveform calibration of the system includes opening the transducer to air and leveling the air-fluid interface of the transducer to the phlebostatic axis.

Which intervention will the nurse include in the plan of care for a patient who has cardiogenic shock? a. Check temperature every 2 hours. b. Monitor breath sounds frequently. c. Maintain patient in supine position. d. Assess skin for flushing and itching.

b. Monitor breath sounds frequently. Since pulmonary congestion and dyspnea are characteristics of cardiogenic shock, the nurse should assess the breath sounds frequently. The head of the bed is usually elevated to decrease dyspnea in patients with cardiogenic shock. Elevated temperature and flushing or itching of the skin are not typical of cardiogenic shock.

Which nursing action should be included in the plan of care after endovascular repair of an abdominal aortic aneurysm? a. Record hourly chest tube drainage. b. Monitor fluid intake and urine output. c. Check the abdominal incision for any redness. d. Teach the reason for a prolonged recovery period.

b. Monitor fluid intake and urine output. Because renal artery occlusion can occur after endovascular repair, the nurse should monitor parameters of renal function such as intake and output. Chest tubes will not be needed for endovascular surgery, the recovery period will be short, and there will not be an abdominal wound.

The nurse who works in the vascular clinic has several patients with venous insufficiency scheduled today. Which patient should the nurse assign to an experienced licensed practical/vocational nurse (LPN/LVN)? a. Patient who has been complaining of increased edema and skin changes in the legs b. Patient who needs wound care for a chronic venous stasis ulcer on the right lower leg c. Patient who has a history of venous thromboembolism and is complaining of some dyspnea d. Patient who needs teaching about the use of elastic compression stockings for venous insufficiency

b. Patient who needs wound care for a chronic venous stasis ulcer on the right lower leg LPN education and scope of practice includes wound care. The other patients, which require more complex assessments or education, should be managed by the RN.

A heart murmur is described as blowing, grating, or harsh. This description would fall under which criteria? a. Intensity b. Quality c. Timing d. Pitch

b. Quality Quality is whether the murmur is blowing, grating, or harsh. Intensity is the loudness graded on a scale of 1 through 6; the higher the number, the louder is the murmur. Timing is the place in the cardiac cycle (systole/diastole). Pitch iswhether the tone is high or low.

Which assessment finding in a patient who is admitted with infective endocarditis (IE) is most important to communicate to the health care provider? a. Generalized muscle aching b. Sudden onset right flank pain c. Janeway's lesions on the palms d. Temperature 100.7° F (38.1° C)

b. Sudden onset right flank pain Sudden onset of flank pain indicates possible embolization to the kidney and may require diagnostic testing such as a renal arteriogram and interventions to improve renal perfusion. The other findings are typically found in IE, but do not require any new interventions.

Which assessment data collected by the nurse who is admitting a patient with chest pain suggest that the pain is caused by an acute myocardial infarction (AMI)? a. The pain increases with deep breathing. b. The pain has lasted longer than 30 minutes. c. The pain is relieved after the patient takes nitroglycerin. d. The pain is reproducible when the patient raises the arms.

b. The pain has lasted longer than 30 minutes. Chest pain that lasts for 20 minutes or more is characteristic of AMI. Changes in pain that occur with raising the arms or with deep breathing are more typical of musculoskeletal pain or pericarditis. Stable angina is usually relieved when the patient takes nitroglycerin.

Signs of hypovolemia in the trauma patient include a. distended neck veins. b. a decreased level of consciousness. c. bounding radial and pedal pulses. d. a widening pulse pressure.

b. a decreased level of consciousness. Signs of underperfusion include flattened neck veins, a decreased level of consciousness, weak and thready peripheral pulses, and a narrowed pulse pressure.

Profound weight loss in patients with SIRS or MODS is the result of a. hypometabolism. b. hypermetabolism. c. hyperglycemia. d. intolerance to enteral feedings

b. hypermetabolism. Hypermetabolism in SIRS or MODS results in profound weight loss, cachexia, and loss of organ function. The goal of nutritional support is the preservation of organ structure and function. Although nutritional support may not definitely alter the course of organ dysfunction, it prevents generalized nutritional deficiencies and preserves gut integrity. Enteral nutrition may exert a physiologic effect that downregulates the systemic immune response and reduces oxidate stress.

A patient with a serum potassium level of 6.8 mEq/L may exhibit electrocardiographic changes of a. a prominent U wave. b. tall, peaked T waves. c. a narrowed QRS. d. sudden ventricular dysrhythmias.

b. tall, peaked T waves. Normal serum potassium levels are 3.5 to 4.5 mEq/L. Tall, narrow peaked T waves are usually, although not uniquely, associated with early hyperkalemia and are followed by prolongation of the PR interval, loss of the P wave, widening of the QRS complex, heart block, and asystole. Severely elevated serum potassium (greater than 8 mEq/L) causes a wide QRS tachycardia.

A patient with ST-segment elevation in three contiguous electrocardiographic (ECG) leads is admitted to the emergency department (ED) and diagnosed as having an ST-segment-elevation myocardial infarction (STEMI). Which question should the nurse ask to determine whether the patient is a candidate for thrombolytic therapy? a. "Do you have any allergies?" b. "Do you take aspirin on a daily basis?" c. "What time did your chest pain begin?" d. "Can you rate your chest pain using a 0 to 10 scale?"

c. "What time did your chest pain begin?" Thrombolytic therapy should be started within 6 hours of the onset of the myocardial infarction (MI), so the time at which the chest pain started is a major determinant of the appropriateness of this treatment. The other information will also be needed, but it will not be a factor in the decision about thrombolytic therapy.

An intraaortic balloon pump (IABP) is being used for a patient who is in cardiogenic shock. Which assessment data indicate to the nurse that the goals of treatment with the IABP are being met? a. Urine output of 25 mL/hr b. Heart rate of 110 beats/minute c. Cardiac output (CO) of 5 L/min d. Stroke volume (SV) of 40 mL/beat

c. Cardiac output (CO) of 5 L/min A CO of 5 L/min is normal and indicates that the IABP has been successful in treating the shock. The low SV signifies continued cardiogenic shock. The tachycardia and low urine output also suggest continued cardiogenic shock.

The family members of a patient who has just been admitted to the intensive care unit (ICU) with multiple traumatic injuries have just arrived in the ICU waiting room. Which action should the nurse take next? a. Explain ICU visitation policies and encourage family visits. b. Immediately take the family members to the patient's bedside. c. Describe the patient's injuries and the care that is being provided. d. Invite the family to participate in a multidisciplinary care conference.

c. Describe the patient's injuries and the care that is being provided. Lack of information is a major source of anxiety for family members and should be addressed first. Family members should be prepared for the patient's appearance and the ICU environment before visiting the patient for the first time. ICU visiting should be individualized to each patient and family rather than being dictated by rigid visitation policies. Inviting the family to participate in a multidisciplinary conference is appropriate but should not be the initial action by the nurse.

The nurse notes that a patient's endotracheal tube (ET), which was at the 22-cm mark, is now at the 25-cm mark and the patient is anxious and restless. Which action should the nurse take next? a. Offer reassurance to the patient. b. Bag the patient at an FIO2 of 100%. c. Listen to the patient's breath sounds. d. Notify the patient's health care provider.

c. Listen to the patient's breath sounds. The nurse should first determine whether the ET tube has been displaced into the right mainstem bronchus by listening for unilateral breath sounds. If so, assistance will be needed to reposition the tube immediately. The other actions are also appropriate, but detection and correction of tube malposition are the most critical actions.

The nurse is caring for a patient who has an intraaortic balloon pump in place. Which action should be included in the plan of care? a. Position the patient supine at all times. b. Avoid the use of anticoagulant medications. c. Measure the patient's urinary output every hour. d. Provide passive range of motion for all extremities.

c. Measure the patient's urinary output every hour. Monitoring urine output will help determine whether the patient's cardiac output has improved and also help monitor for balloon displacement. The head of the bed can be elevated up to 30 degrees. Heparin is used to prevent thrombus formation. Limited movement is allowed for the extremity with the balloon insertion site to prevent displacement of the balloon.

Which assessment finding for a patient who has been admitted with a right calf venous thromboembolism (VTE) requires immediate action by the nurse? a. Erythema of right lower leg b. Complaint of right calf pain c. New onset shortness of breath d. Temperature of 100.4° F (38° C)

c. New onset shortness of breath New onset dyspnea suggests a pulmonary embolus, which will require rapid actions such as oxygen administration and notification of the health care provider. The other findings are typical of VTE.

The nurse is reviewing the laboratory results for newly admitted patients on the cardiovascular unit. Which patient laboratory result is most important to communicate as soon as possible to the health care provider? a. Patient whose triglyceride level is high b. Patient who has very low homocysteine level c. Patient with increase in troponin T and troponin I level d. Patient with elevated high-sensitivity C-reactive protein level

c. Patient with increase in troponin T and troponin I level The elevation in troponin T and I indicates that the patient has had an acute myocardial infarction. Further assessment and interventions are indicated. The other laboratory results are indicative of increased risk for coronary artery disease but are not associated with acute cardiac problems that need immediate intervention

Which electrocardiographic (ECG) change is most important for the nurse to report to the health care provider when caring for a patient with chest pain? a. Inverted P wave b. Sinus tachycardia c. ST-segment elevation d. First-degree atrioventricular block

c. ST-segment elevation The patient is likely to be experiencing an ST-segment-elevation myocardial infarction (STEMI). Immediate therapy with percutaneous coronary intervention (PCI) or thrombolytic medication is indicated to minimize myocardial damage. The other ECG changes may also suggest a need for therapy, but not as rapidly.

The nurse is caring for a patient receiving a continuous norepinephrine (Levophed) IV infusion. Which patient assessment finding indicates that the infusion rate may need to be adjusted? a. Heart rate is 58 beats/minute. b. Mean arterial pressure (MAP) is 56 mm Hg. c. Systemic vascular resistance (SVR) is elevated. d. Pulmonary artery wedge pressure (PAWP) is low.

c. Systemic vascular resistance (SVR) is elevated. Vasoconstrictors such as norepinephrine (Levophed) will increase SVR, and this will increase the work of the heart and decrease peripheral perfusion. The infusion rate may need to be decreased. Bradycardia, hypotension (MAP of 56 mm Hg), and low PAWP are not associated with norepinephrine infusion.

When SIRS is the result of infection, it is called a. inflammation. b. anaphylaxis. c. sepsis. d. pneumonia.

c. sepsis. When SIRS is the result of infection, the term sepsis is used.

The patient has an HR of 84 beats/min and an SV of 65 mL. Calculate the CO. a. 149 mL b. 500 mL c. 4650 mL d. 5460 mL

d. 5460 mL Cardiac output (CO) is the product of heart rate (HR) multiplied by stroke volume (SV). SV is the volume of blood ejected by the heart during each beat (reported in milliliters). 84 x 65 = 5460 mL.

After receiving change-of-shift report about the following four patients, which patient should the nurse assess first? a. 39-year-old with pericarditis who is complaining of sharp, stabbing chest pain b. 56-year-old with variant angina who is to receive a dose of nifedipine (Procardia) c. 65-year-old who had a myocardial infarction (MI) 4 days ago and is anxious about the planned discharge d. 59-year-old with unstable angina who has just returned to the unit after having a percutaneous coronary intervention (PCI)

d. 59-year-old with unstable angina who has just returned to the unit after having a percutaneous coronary intervention (PCI) This patient is at risk for bleeding from the arterial access site for the PCI, so the nurse should assess the patient's blood pressure, pulse, and the access site immediately. The other patients should also be assessed as quickly as possible, but assessment of this patient has the highest priority.

An 82-year-old patient is admitted into the critical care unit with a diagnosis of left-sided heart failure related to mitral stenosis. Physical assessment findings reveal tachycardia with an S and a 3/6 systolic murmur. Which of the following descriptions best describes the murmur heard with mitral stenosis? a. High-pitched systolic sound b. Medium-pitched systolic sound c. High-pitched diastolic sound d. Low-pitched diastolic sound

d. Low-pitched diastolic sound Mitral stenosis describes a narrowing of the mitral valve orifice. This produces a low-pitched murmur, which varies in intensity and harshness depending on the degree of valvular stenosis. It occurs during diastole, is auscultated at the mitral area (fifth ICS, midclavicular line), and does not radiate.

Which of the following conditions can cause an artificial increase in the PAOP/PCWP? a. Aortic regurgitation b. Aortic stenosis c. Mitral stenosis d. Mitral regurgitation

d. Mitral regurgitation If mitral regurgitation is present, the mean pulmonary artery occlusion pressure reading is artificially elevated because of abnormal backflow of blood from the left ventricle to the left atrium during systole.

While assessing a patient who was admitted with heart failure, the nurse notes that the patient has jugular venous distention (JVD) when lying flat in bed. Which action should the nurse take next? a. Document this finding in the patient's record. b. Obtain vital signs, including oxygen saturation. c. Have the patient perform the Valsalva maneuver. d. Observe for JVD with the patient upright at 45 degrees.

d. Observe for JVD with the patient upright at 45 degrees. When the patient is lying flat, the jugular veins are at the level of the right atrium, so JVD is a common (but not a clinically significant) finding. Obtaining vital signs and oxygen saturation is not warranted at this point. JVD is an expected finding when a patient performs the Valsalva maneuver because right atrial pressure increases. JVD that persists when the patient is sitting at a 30- to 45-degree angle or greater is significant. The nurse will document the JVD in the medical record if it persists when the head is elevated.

A new-onset MI can be recognized by which of the following ECG changes? a. Q waves b. Smaller R waves c. Widened QRS d. ST segment elevation

d. ST segment elevation Any change from baseline is expressed in millimeters and may indicate myocardial ischemia (one small box equals 1 mm). ST segment elevation of 1 to 2 mm is associated with acute myocardial injury, preinfarction, and pericarditis. ST segment depression (decrease from baseline more of 1 to 2 mm) is associated with myocardial ischemia. Widened QRS complexes are indicative of ventricular depolarization abnormalities such as bundle branch blocks and ventricular dysrhythmias. Q waves and smaller R waves are indications usually present 24 hours to 1 week after the myocardial infarction is completely evolved; they represent necrosis.

Dyspnea with wheezing, a nonproductive cough, and pulmonary crackles that progress to the gurgling sounds of pulmonary edema is described as a. dyspnea. b. orthopnea. c. paroxysmal nocturnal dyspnea. d. cardiac asthma.

d. cardiac asthma. Dyspnea with wheezing, a nonproductive cough, and pulmonary crackles that progress to the gurgling sounds of pulmonary edema are symptoms of cardiac asthma. With dyspnea, the patient feels shortness of breath from pulmonary vascular congestion and decreased lung compliance. In orthopnea, the patient has difficulty breathing when lying flat because of an increase in venous return that occurs in the supine position. Paroxysmal nocturnal dyspnea is a severe form of orthopnea in which the patient awakens from sleep gasping for air.

Contractility of the left side of the heart is measured by a. pulmonary artery wedge pressure. b. left atrial pressure. c. systemic vascular resistance. d. left ventricular stroke work index.

d. left ventricular stroke work index. Contractility of the left side of the heart is measured by the left ventricular stroke work index.

The rationale for administration of a fibrinolytic agent is a. dilation of the blocked coronary artery. b. anticoagulation to prevent formation of new emboli. c. dissolution of atherosclerotic plaque at the site of blockage. d. restoration of blood flow to the obstructed coronary artery via lysis of the thrombus.

d. restoration of blood flow to the obstructed coronary artery via lysis of the thrombus. The administration of a fibrinolytic agent results in the lysis of the acute thrombus, thus recanalizing, or opening, the obstructed coronary artery and restoring blood flow to the affected tissue. After perfusion is restored, adjunctive measures are taken to prevent further clot formation and reocclusion.

To verify the correct placement of an oral endotracheal tube (ET) after insertion, the best initial action by the nurse is to a. auscultate for the presence of bilateral breath sounds. b. obtain a portable chest x-ray to check tube placement. c. observe the chest for symmetric chest movement with ventilation. d. use an end-tidal CO monitor to check for placement in the trachea.

d. use an end-tidal CO monitor to check for placement in the trachea. End-tidal CO monitors are currently recommended for rapid verification of ET placement. Auscultation for bilateral breath sounds and checking chest expansion are also used, but they are not as accurate as end-tidal CO monitoring. A chest x-ray confirms the placement but is done after the tube is secured.

When assessing a patient with possible peripheral artery disease (PAD), the nurse obtains a brachial BP of 147/82 and an ankle pressure of 112/74. The nurse calculates the patient's ankle-brachial index (ABI) as ________ (round up to the nearest hundredth).

0.76 The ABI is calculated by dividing the ankle systolic BP by the brachialsystolic BP.

During a history examination, a patient tells the nurse, "The cardiologist says I have a leaking valve." The nurse documents that the patient has a history of a. acute mitral regurgitation. b. aortic insufficiency. c. chronic mitral regurgitation. d. pericardial friction rub.

b. aortic insufficiency. Aortic insufficiency is an incompetent aortic valve. If the valve cusps do not maintain this seal, the sound of blood flowing back into the left ventricle during diastole is heard as a decrescendo, high-pitched, blowing murmur. A pericardial friction rub is a sound that can occur within 2 to 7 days after a myocardial infarction. The friction rub results from pericardial inflammation (pericarditis). Classically, a pericardial friction rub is a grating or scratching sound that is both systolic and diastolic, corresponding to cardiac motion within the pericardial sac. Acute mitral regurgitation occurs when the ventricle contracts during systole and a jet of blood is sent in a retrograde manner to the left atrium, causing a sudden increase in left atrial pressure, acute pulmonary edema, and low CO and leading to cardiogenic shock. Chronic mitral regurgitation is auscultated in the mitral area and occurs during systole. It is high pitched and blowing, although the pitch and intensity vary, depending on the degree of regurgitation. As mitral regurgitation progresses, the murmur radiates more widely.

The presence of a carotid or femoral bruit may be evidence of a. left-sided heart failure. b. blood flow through a partially occluded vessel. c. the early onset of pulmonary embolism. d. myocardial rupture.

b. blood flow through a partially occluded vessel. A bruit is an extracardiac vascular sound that results from blood flow through a tortuous or partially occluded vessel.

A patient who is on the progressive care unit develops atrial flutter, rate 150, with associated dyspnea and chest pain. Which action that is included in the hospital dysrhythmia protocol should the nurse do first? a. Obtain a 12-lead electrocardiogram (ECG). b. Notify the health care provider of the change in rhythm. c. Give supplemental O at 2 to 3 L/min via nasal cannula. d. Assess the patient's vital signs including oxygen saturation.

c. Give supplemental O at 2 to 3 L/min via nasal cannula. Because this patient has dyspnea and chest pain in association with the new rhythm, the nurse's initial actions should be to address the patient's airway, breathing, and circulation (ABC) by starting with oxygen administration. The other actions also are important and should be implemented rapidly.

The most common complication of fibrinolytic therapy is a. reperfusion chest pain. b. lethargy. c. bleeding. d. heart blocks.

c. bleeding. The most common complication related to thrombolysis is bleeding.

The target INR range is a. 1.0 to 2.0. b. 1.5 to 3.0. c. 1.5 to 2.5. d. 2.0 to 3.0.

d. 2.0 to 3.0. A target international normalized ratio of 2.5 (range, 2.0-3.0) is desirable.

When analyzing an electrocardiographic (ECG) rhythm strip of a patient with a regular heart rhythm, the nurse counts 30 small blocks from one R wave to the next. The nurse calculates the patient's heart rate as ____.

50 There are 1500 small blocks in a minute, and the nurse will divide 1500 by 30.

Which of the following statements made by a patient would indicate the need for further education before an electrophysiology procedure? a. "I need to take all my heart medications the morning of the procedure." b. "The doctor is going to make my heart beat wrong on purpose." c. "I will be awake but relaxed during the procedure." d. "I will be x-rayed during the procedure."

a. "I need to take all my heart medications the morning of the procedure." All antidysrhythmic medications are discontinued several days before the study so that any ventricular dysrhythmias may be readily induced during the electrophysiology procedure (EPS). Anticoagulants, especially warfarin, are also stopped before EPS. Premedication is administered before the study to induce a relaxed state, and during the procedure, the patient is conscious but receives sedative agents (midazolam) at regular intervals.

A 20-year-old has a mandatory electrocardiogram (ECG) before participating on a college soccer team and is found to have sinus bradycardia, rate 52. Blood pressure (BP) is 114/54, and the student denies any health problems. What action by the nurse is most appropriate? a. Allow the student to participate on the soccer team. b. Refer the student to a cardiologist for further diagnostic testing. c. Tell the student to stop playing immediately if any dyspnea occurs. d. Obtain more detailed information about the student's family health history.

a. Allow the student to participate on the soccer team. In an aerobically trained individual, sinus bradycardia is normal. The student's normal BP and negative health history indicate that there is no need for a cardiology referral or for more detailed information about the family's health history. Dyspnea during an aerobic activity such as soccer is normal.

Two days after an acute myocardial infarction (MI), a patient complains of stabbing chest pain that increases with a deep breath. Which action will the nurse take first? a. Auscultate the heart sounds. b. Check the patient's temperature. c. Notify the patient's health care provider. d. Give the PRN acetaminophen (Tylenol).

a. Auscultate the heart sounds. The patient's clinical manifestations and history are consistent with pericarditis, and the first action by the nurse should be to listen for a pericardial friction rub. Checking the temperature and notifying the health care provider are also appropriate actions but would not be done before listening for a rub. It is not stated for what symptom (e.g., headache) or finding (e.g., increased temperature) the PRN acetaminophen (Tylenol) is ordered.

Which of the following effects can be associated with physical exercise? (Select all that apply.) a. Decreased LDL cholesterol b. Increased HDL cholesterol c. Decreased triglycerides d. Increased insulin resistance e. Decreased incidence of depression

a. Decreased LDL cholesterol b. Increased HDL cholesterol c. Decreased triglycerides e. Decreased incidence of depression Many research trials have demonstrated the positive effects of physical activity on the other major cardiac risk factors. Exercise alters the lipid profile by decreasing low-density lipoprotein cholesterol and triglyceride levels and increasing high-density lipoprotein cholesterol levels. Exercise reduces insulin resistance at the cellular level, lowering the risk for developing type 2 diabetes, especially if combined with a weight loss program.

A patient with septic shock has a BP of 70/46 mm Hg, pulse 136, respirations 32, temperature 104° F, and blood glucose 246 mg/dL. Which intervention ordered by the health care provider should the nurse implement first? a. Give normal saline IV at 500 mL/hr. b. Give acetaminophen (Tylenol) 650 mg rectally. c. Start insulin drip to maintain blood glucose at 110 to 150 mg/dL. d. Start norepinephrine (Levophed) to keep systolic blood pressure >90 mm Hg.

a. Give normal saline IV at 500 mL/hr. Because of the low systemic vascular resistance (SVR) associated with septic shock, fluid resuscitation is the initial therapy. The other actions also are appropriate, and should be initiated quickly as well.

When caring for a patient who has just arrived on the medical-surgical unit after having cardiac catheterization, which nursing intervention should the nurse delegate to a licensed practical/vocational nurse (LPN/LVN)? a. Give the scheduled aspirin and lipid-lowering medication. b. Perform the initial assessment of the catheter insertion site. c. Teach the patient about the usual postprocedure plan of care. d. Titrate the heparin infusion according to the agency protocol.

a. Give the scheduled aspirin and lipid-lowering medication. Administration of oral medications is within the scope of practice for LPNs/LVNs. The initial assessment of the patient, patient teaching, and titration of IV anticoagulant medications should be done by the registered nurse (RN).

A patient with suspected neurogenic shock after a diving accident has arrived in the emergency department. A cervical collar is in place. Which actions should the nurse take (select all that apply)? a. Prepare to administer atropine IV. b. Obtain baseline body temperature. c. Infuse large volumes of lactated Ringer's solution. d. Provide high-flow oxygen (100%) by non-rebreather mask. e. Prepare for emergent intubation and mechanical ventilation.

a. Prepare to administer atropine IV. b. Obtain baseline body temperature. d. Provide high-flow oxygen (100%) by non-rebreather mask. e. Prepare for emergent intubation and mechanical ventilation. All of the actions are appropriate except to give large volumes of lactated Ringer's solution. The patient with neurogenic shock usually has a normal blood volume, and it is important not to volume overload the patient. In addition, lactated Ringer's solution is used cautiously in all shock situations because the failing liver cannot convert lactate to bicarbonate.

A patient is admitted to the emergency department (ED) for shock of unknown etiology. The first action by the nurse should be to a. administer oxygen. b. obtain a 12-lead electrocardiogram (ECG). c. obtain the blood pressure. d. check the level of consciousness.

a. administer oxygen. The initial actions of the nurse are focused on the ABCs—airway, breathing, and circulation—and administration of oxygen should be done first. The other actions should be accomplished as rapidly as possible after oxygen administration.

A patient with dilated cardiomyopathy has new onset atrial fibrillation that has been unresponsive to drug therapy for several days. The priority teaching needed for this patient would include information about a. anticoagulant therapy. b. permanent pacemakers. c. electrical cardioversion. d. IV adenosine (Adenocard).

a. anticoagulant therapy. Atrial fibrillation therapy that has persisted for more than 48 hours requires anticoagulant treatment for 3 weeks before attempting cardioversion. This is done to prevent embolization of clots from the atria. Cardioversion may be done after several weeks of anticoagulation therapy. Adenosine is not used to treat atrial fibrillation. Pacemakers are routinely used for patients with bradydysrhythmias. Information does not indicate that the patient has a slow heart rate.

The purpose of the Allen test is to a. assess adequate blood flow through the ulnar artery. b. occlude the brachial artery and evaluate hypoxemia to the hand. c. test the patency of an internal graft. d. determine the size of needle to be used for puncture.

a. assess adequate blood flow through the ulnar artery. The Allen test assesses the adequacy of blood flow to the hand through the ulnar artery.

Adenosine (Adenocard) is an antidysrhythmic agent given primarily to a. convert supraventricular tachycardias. b. act as a second-line drug for premature ventricular contractions (PVCs). c. treat AV blocks. d. coarsen ventricular fibrillation so that defibrillation is effective.

a. convert supraventricular tachycardias. Adenosine occurs endogenously in the body as a building block of adenosine triphosphate (ATP). Given in intravenous boluses, adenosine slows conduction through the atrioventricular (AV) node, causing transient AV block. It is used clinically to convert supraventricular tachycardias and to facilitate the differential diagnosis of rapid dysrhythmias.

A reliable indicator of reperfusion after fibrinolytic therapy is a. dysrhythmias. b. Q waves. c. elevated ST segments. d. a rapid decrease in serum CK levels.

a. dysrhythmias. Initially, when there is reperfusion, ischemic chest pain ceases abruptly as blood flow is restored. Another reliable indicator of reperfusion is the appearance of various "reperfusion" dysrhythmias. Premature ventricular contractions, bradycardias, heart block, ventricular tachycardia, and (rarely) ventricular fibrillation may occur.

A patient is admitted to the hospital with possible acute pericarditis. The nurse should plan to teach the patient about the purpose of a. echocardiography. b. daily blood cultures. c. cardiac catheterization. d. 24-hour Holter monitor.

a. echocardiography. Echocardiograms are useful in detecting the presence of the pericardial effusions associated with pericarditis. Blood cultures are not indicated unless the patient has evidence of sepsis. Cardiac catheterization and 24-hour Holter monitor is not a diagnostic procedure for pericarditis.

Pulsus paradoxus may be seen on intra-arterial blood pressure monitoring when a. there is a decrease of more than 10 mm Hg in the arterial waveform before inhalation. b. there is a single, nonperfused beat. c. the waveform demonstrates tall, tented T waves. d. the pulse pressure is greater than 20 mm Hg on exhalation.

a. there is a decrease of more than 10 mm Hg in the arterial waveform before inhalation. Pulsus paradoxus is a decrease of more than 10 mm Hg in the arterial waveform that occurs during inhalation. It is caused by a fall in CO as a result of increased negative intrathoracic pressure during inhalation.

The nurse has started discharge teaching for a patient who is to continue warfarin (Coumadin) following hospitalization for venous thromboembolism (VTE). The nurse determines that additional teaching is needed when the patient says which of the following? a. "I should get a Medic Alert device stating that I take Coumadin." b. "I should reduce the amount of green, leafy vegetables that I eat." c. "I will need routine blood tests to monitor the effects of the Coumadin." d. "I will check with my health care provider before I begin any new medications."

b. "I should reduce the amount of green, leafy vegetables that I eat." Patients taking warfarin are taught to follow a consistent diet with regard to foods that are high in vitamin K, such as green, leafy vegetables. The other patient statements are accurate.

Which actions could the nurse delegate to unlicensed assistive personnel (UAP) who are providing care for a patient who is at risk for venous thromboembolism? a. Monitor for any bleeding after anticoagulation therapy is started. b. Apply sequential compression device whenever the patient is in bed. c. Ask the patient about use of herbal medicines or dietary supplements. d. Instruct the patient to call immediately if any shortness of breath occurs.

b. Apply sequential compression device whenever the patient is in bed. UAP training includes the use of equipment that requires minimal nursing judgment, such as sequential compression devices. Patient assessment and teaching require more education and critical thinking and should be done by the registered nurse (RN).

Ventricular tachycardia has which of the following hemodynamic effects? a. Decreased cardiac output from increased ventricular filling time b. Decreased cardiac output from decreased stroke volume c. Decreased cardiac output from increased preload d. Decreased cardiac output from decreased afterload

b. Decreased cardiac output from decreased stroke volume Tachycardia is detrimental to anyone with ischemic heart disease because it decreases the time for ventricular filling, decreases stroke volume, and compromises cardiac output. Tachycardia increases heart work and myocardial oxygen demand while decreasing oxygen supply by decreasing coronary artery filling time.

Which anticoagulant(s) enhances activity of antithrombin III and does not require aPTT or ACT monitoring? a. Heparin b. Enoxaparin (Lovenox) c. Bivalirudin (Angiomax) d. Argatroban (Argatroban)

b. Enoxaparin (Lovenox) Enoxaparin (Lovenox) enhances activity of antithrombin III, a more predictable response than heparin, because enoxaparin is not largely bound to protein. There is no need for activated partial thromboplastin time (aPTT) or ACT monitoring, and there is a lower risk of heparin-induced thrombocytopenia (HIT) than with unfractionated heparin (UFH). Heparin sodium enhances activity of antithrombin III, a natural anticoagulant, to prevent clot formation. The effectiveness of treatment may be monitored by aPTT or ACT. Response is variable because of binding with plasma proteins effects may be reversed with protamine sulfate.Bivalirudin (Angiomax) directly inhibits thrombin. It may be administered alone or in combination with glycoprotein IIb/IIIa inhibitors and produces a dose-dependent increase in aPTT and ACT. It may be used instead of UFH for patients with HIT.Argatroban (Argatroban) directly inhibits thrombin. It may be used instead of UFH for patients with HIT. Whereas ACT is monitored duringpercutaneous coronary intervention, aPTT is used during prolonged infusion.

When caring for a patient on the first postoperative day after an abdominal aortic aneurysm repair, which assessment finding is most important for the nurse to communicate to the health care provider? a. Presence of flatus b. Loose, bloody stools c. Hypoactive bowel sounds d. Abdominal pain with palpation

b. Loose, bloody stools Loose, bloody stools at this time may indicate intestinal ischemia or infarction, and should be reported immediately because the patient may need an emergency bowel resection. The other findings are normal on the first postoperative day after abdominal surgery.

Which serum lipid value is a significant predictor of future acute MI in persons with established coronary artery atherosclerosis? a. High-density lipoprotein (HDL) b. Low-density lipoprotein (LDL) c. Triglycerides d. Very-low-density lipoprotein

b. Low-density lipoprotein (LDL) Both the LDL-C and total serum cholesterol levels are directly correlated with risk for coronary artery disease, and high levels of each are significant predictors of future acute myocardial infarction in persons with established coronary artery atherosclerosis. LDL-C is the major atherogenic lipoprotein and thus is the primary target for cholesterol-lowering efforts.

The nurse is caring for a patient with critical limb ischemia who has just arrived on the nursing unit after having percutaneous transluminal balloon angioplasty. Which action should the nurse perform first? a. Begin oral intake. b. Obtain vital signs. c. Assess pedal pulses. d. Start discharge teaching.

b. Obtain vital signs. Bleeding is a possible complication after catheterization of the femoral artery, so the nurse's first action should be to assess for changes in vital signs that might indicate hemorrhage. The other actions are also appropriate but can be done after determining that bleeding is not occurring.

Which portion of the ECG is most valuable in diagnosing atrioventricular (AV) conduction disturbances? a. P wave b. PR interval c. QRS complex d. QT interval

b. PR interval he PR interval is an indicator of atrioventricular nodal function. The P wave represents atrial depolarization. The QRS complex represents ventricular depolarization, corresponding to phase 0 of the ventricular action potential. The QT interval is measured from the beginning of the QRS complex to the end of the T wave and indicates the total time interval from the onset of depolarization to the completion of repolarization.

A patient presents with atrial flutter with an atrial rate of 280 beats/min and a ventricular rate of 70 beats/min. Which of the following best explains this discrepancy in rates? a. The ventricles are too tired to respond to all the atrial signals. b. The AV node does not conduct all the atrial signals to the ventricles. c. Some of the atrial beats are blocked before reaching the AV node. d. The ventricles are responding to a ventricular ectopic pacemaker.

b. The AV node does not conduct all the atrial signals to the ventricles. The atrioventricular (AV) node does not allow conduction of all these impulses to the ventricles. In this case, the rhythm would be described as atrial flutter with a 4:1 AV block, indicating that only one of every four atrial signals is conducted to the ventricles.

When caring for a patient who has an arterial catheter in the left radial artery for arterial pressure-based cardiac output (APCO) monitoring, which information obtained by the nurse is most important to report to the health care provider? a. The patient has a positive Allen test. b. There is redness at the catheter insertion site. c. The mean arterial pressure (MAP) is 86 mm Hg. d. The dicrotic notch is visible in the arterial waveform.

b. There is redness at the catheter insertion site. Redness at the catheter insertion site indicates possible infection. The Allen test is performed before arterial line insertion, and a positive test indicates normal ulnar artery perfusion. A MAP of 86 is normal and the dicrotic notch is normally present on the arterial waveform.

A patient is scheduled for a cardiac catheterization with coronary angiography. Before the test, the nurse informs the patient that a. it will be important to lie completely still during the procedure. b. a flushed feeling may be noted when the contrast dye is injected. c. monitored anesthesia care will be provided during the procedure. d. arterial pressure monitoring will be required for 24 hours after the test.

b. a flushed feeling may be noted when the contrast dye is injected. A sensation of warmth or flushing is common when the contrast material is injected, which can be anxiety-producing unless it has been discussed with the patient. The patient may receive a sedative drug before the procedure, but monitored anesthesia care is not used. Arterial pressure monitoring is not routinely used after the procedure to monitor blood pressure. The patient is not immobile during cardiac catheterization and may be asked to cough or take deep breaths.

To assess the patient with pericarditis for evidence of a pericardial friction rub, the nurse should a. listen for a rumbling, low-pitched, systolic murmur over the left anterior chest. b. auscultate by placing the diaphragm of the stethoscope on the lower left sternal border. c. ask the patient to cough during auscultation to distinguish the sound from a pleural friction rub. d. feel the precordial area with the palm of the hand to detect vibrations with cardiac contraction.

b. auscultate by placing the diaphragm of the stethoscope on the lower left sternal border. Pericardial friction rubs are heard best with the diaphragm at the lower left sternal border. The nurse should ask the patient to hold his or her breath during auscultation to distinguish the sounds from a pleural friction rub. Friction rubs are not typically low pitched or rumbling and are not confined to systole. Rubs are not assessed by palpation.

The difference between primary and secondary multiple organ dysfunction syndrome (MODS) is that primary MODS is the result of a. widespread systemic inflammation that results in dysfunction of organs not involved in the initial insult. b. direct organ injury. c. disorganization of the immune system response. d. widespread disruption of the coagulation system.

b. direct organ injury. Organ dysfunction may be the direct consequence of an initial insult (primary MODS) or can manifest latently and involve organs not directly affected in the initial insult (secondary MODS). Patients can experience both primary and secondary MODS. Primary MODS results from a well-defined insult in which organ dysfunction occurs early and is directly attributed to the insult itself.

The most common complication of a central venous catheter (CVC) is a. air embolus. b. infection. c. thrombus formation. d. pneumothorax.

b. infection. Infection related to the use of CVCs is a major problem. The incidence of infection strongly correlates with the length of time the CVC has been inserted, with longer insertion times leading to a higher infection rate. The risk of air embolus, although uncommon, is always present for a patient with a central venous line in place. Air can enter during insertion through a disconnected or broken catheter by means of an open stopcock, or air can enter along the path of a removed CVC. Unfortunately, clot formation (thrombus) at the CVC site is common. Thrombus formation is not uniform; it may involve development of afibrin sleeve around the catheter, or the thrombus may be attached directly to the vessel wall. Pneumothorax has a higher occurrence during placement of a CVC than during removal.

An 81-year-old patient who has been in the intensive care unit (ICU) for a week is now stable and transfer to the progressive care unit is planned. On rounds, the nurse notices that the patient has new onset confusion. The nurse will plan to a. give PRN lorazepam (Ativan) and cancel the transfer. b. inform the receiving nurse and then transfer the patient. c. notify the health care provider and postpone the transfer. d. obtain an order for restraints as needed and transfer the patient.

b. inform the receiving nurse and then transfer the patient. The patient's history and symptoms most likely indicate delirium associated with the sleep deprivation and sensory overload in the ICU environment. Informing the receiving nurse and transferring the patient is appropriate. Postponing the transfer is likely to prolong the delirium. Benzodiazepines and restraints contribute to delirium and agitation.

After receiving 2 L of normal saline, the central venous pressure for a patient who has septic shock is 10 mm Hg, but the blood pressure is still 82/40 mm Hg. The nurse will anticipate an order for a. nitroglycerine (Tridil). b. norepinephrine (Levophed). c. sodium nitroprusside (Nipride). d. methylprednisolone (Solu-Medrol).

b. norepinephrine (Levophed). When fluid resuscitation is unsuccessful, vasopressor drugs are administered to increase the systemic vascular resistance (SVR) and blood pressure, and improve tissue perfusion. Nitroglycerin would decrease the preload and further drop cardiac output and BP. Methylprednisolone (Solu-Medrol) is considered if blood pressure does not respond first to fluids and vasopressors. Nitroprusside is an arterial vasodilator and would further decrease SVR.

In analyzing the ECG strip, the nurse notices a spike before each QRS complex. The patient's heart rate is 70 beats/min. This phenomenon is reflective of a. 60-cycle electrical interference; check equipment. b. pacing artifact; the pacemaker is sensing and capturing. c. electrical artifact; the pacemaker is not sensing. d. patient movement; check electrodes.

b. pacing artifact; the pacemaker is sensing and capturing. The pacing artifact is the spike that is seen on the electrocardiographic tracing as the pacing stimulus is delivered to the heart. A P wave is visible after the pacing artifact if the atrium is being paced. Similarly, a QRS complex follows a ventricular pacing artifact. With dual-chamber pacing, a pacing artifact precedes both the P wave and the QRS complex.

The predominant form of percutaneous coronary intervention (PCI) is a. Rotablator. b. stents. c. directional coronary atherectomy (DCA). d. balloon angioplasty (PTCA).

b. stents. Stents are currently the predominant form of percutaneous coronary intervention and are used in more than 90% of all interventional procedures. TheRotablator device has a high-speed, rotating, diamond-coated bur that drills through the plaque, creating tiny particles. Because directional coronary atherectomy extracts pieces of atheroma that can be studied microscopically (similar to a biopsy specimen), it has contributed significantly to our understanding of atherosclerosis and restenosis. Percutaneous transluminal coronary angioplasty (PTCA), frequently abbreviated to balloon angioplasty or simplyangioplasty, was introduced in 1977 as an alternative to coronary surgical revascularization. PTCA avoided many of the risks associated with cardiac surgery (general anesthesia, sternotomy, extracorporeal circulation, and mechanical ventilation), but its success was hampered by complications related to the procedure (acute closure) and restenosis or renarrowing of the vessel after the procedure.

The possibility of microshock when handling a temporary pacemaker can be minimized by a. decreasing the milliamperes. b. wearing gloves. c. positioning the patient on the left side. d. wearing rubber-soled shoes.

b. wearing gloves. The possibility of "microshock" can be minimized by wearing gloves when handling the pacing wires and by proper insulation of terminal pins of pacing wires when they are not in use. The latter can be accomplished either by using caps provided by the manufacturer or by improvising with a plastic syringe or section of disposable rubber glove. The wires are to be taped securely to the patient's chest to prevent accidental electrode displacement.

Which statement by a patient with restrictive cardiomyopathy indicates that the nurse's discharge teaching about selfmanagement has been most effective? a. "I will avoid taking aspirin or other antiinflammatory drugs." b. "I will need to limit my intake of salt and fluids even in hot weather." c. "I will take antibiotics when my teeth are cleaned at the dental office." d. "I should begin an exercise program that includes things like biking or swimming."

c. "I will take antibiotics when my teeth are cleaned at the dental office." Patients with restrictive cardiomyopathy are at risk for infective endocarditis and should use prophylactic antibiotics for any procedure that may cause bacteremia. The other statements indicate a need for more teaching by the nurse. Dehydration and vigorous exercise impair ventricular filling in patients with restrictive cardiomyopathy. There is no need to avoid salt (unless ordered), aspirin, or NSAIDs.

When performing a 12-lead ECG, how many wires are connected to the patient? a. 3 b. 5 c. 10 d. 12

c. 10 The standard 12-lead electrocardiogram provides a picture of electrical activity in the heart using 10 different electrode positions to create 12 unique views of electrical activity occurring within the heart. Fours wires are applied to the extremities to produce leads I, II, III, aVR, aVL, and aVF. Six wires are attached to the V to V chest lead positions.

A patient's blood pressure is 90/72 mm Hg. What is the patient's pulse pressure? a. 40 mm Hg b. 25 mm Hg c. 18 mm Hg d. 12 mm Hg

c. 18 mm Hg Pulse pressure describes the difference between systolic and diastolic values. The normal pulse pressure is 40 mm Hg (i.e., the difference between an SBP of 120 mm Hg and a DBP of 80 mm Hg). A patient with a blood pressure of 90/72mm Hg has a pulse pressure of 18 mm Hg.

Why is the measurement of the QT interval important? a. It represents ventricular depolarization. b. It represents ventricular contraction. c. An increasing QT interval increases the risk of torsades de pointes. d. A decreasing QT interval increases the risk of torsades de pointes.

c. An increasing QT interval increases the risk of torsades de pointes. A prolonged QT interval is significant because it can predispose the patient to the development of polymorphic ventricular tachycardia, known also as torsades de pointes. A long QT interval can be congenital, as a result of genetic inheritance, or it can be acquired from an electrolyte imbalance or medications.

The nurse is caring for a patient with a descending aortic dissection. Which assessment finding is most important to report to the health care provider? a. Weak pedal pulses b. Absent bowel sounds c. Blood pressure 137/88 mm Hg d. 25 mL urine output over last hour

c. Blood pressure 137/88 mm Hg The blood pressure is typically kept at less than 120 mm Hg systolic to minimize extension of the dissection. The nurse will need to notify the health care provider so that b-blockers or other antihypertensive medications can be prescribed. The other findings are typical with aortic dissection and should also be reported but do not require immediate action.

A 46-year-old service-counter worker undergoes sclerotherapy for treatment of superficial varicose veins at an outpatient center. Which instructions should the nurse provide to the patient before discharge? a. Sitting at the work counter, rather than standing, is recommended. b. Exercise, such as walking or jogging, can cause recurrence of varicosities. c. Elastic compression stockings should be applied before getting out of bed. d. Taking an aspirin daily will help prevent clots from forming around venous valves.

c. Elastic compression stockings should be applied before getting out of bed. Elastic compression stockings are applied with the legs elevated to reduce pressure in the lower legs. Walking is recommended to prevent recurrent varicosities. Sitting and standing are both risk factors for varicose veins and venous insufficiency. An aspirin a day is not adequate to prevent venous thrombosis and would not be recommended for the patient who had just had sclerotherapy.

A patient who has chest pain is admitted to the emergency department (ED) and all of the following are ordered. Which one should the nurse arrange to be completed first? a. Chest x-ray b. Troponin level c. Electrocardiogram (ECG) d. Insertion of a peripheral IV

c. Electrocardiogram (ECG) The priority for the patient is to determine whether an acute myocardial infarction (AMI) is occurring so that reperfusion therapy can begin as quickly as possible. ECG changes occur very rapidly after coronary artery occlusion, and an ECG should be obtained as soon as possible. Troponin levels will increase after about 3 hours. Data from the chest x-ray may impact the patient's care but are not helpful in determining whether the patient is experiencing a myocardial infarction (MI). Peripheral access will be needed but not before the ECG.

Heparin is ordered for a patient with a non-ST-segment-elevation myocardial infarction (NSTEMI). What is the purpose of the heparin? a. Heparin enhances platelet aggregation. b. Heparin decreases coronary artery plaque size. c. Heparin prevents the development of new clots in the coronary arteries. d. Heparin dissolves clots that are blocking blood flow in the coronary arteries.

c. Heparin prevents the development of new clots in the coronary arteries. Heparin helps prevent the conversion of fibrinogen to fibrin and decreases coronary artery thrombosis. It does not change coronary artery plaque, dissolve already formed clots, or enhance platelet aggregation.

A patient recovering from heart surgery develops pericarditis and complains of level 6 (0 to 10 scale) chest pain with deep breathing. Which ordered PRN medication will be the most appropriate for the nurse to give? a. Fentanyl 1 mg IV b. IV morphine sulfate 4 mg c. Oral ibuprofen (Motrin) 600 mg d. Oral acetaminophen (Tylenol) 650 mg

c. Oral ibuprofen (Motrin) 600 mg The pain associated with pericarditis is caused by inflammation, so nonsteroidal antiinflammatory drugs (NSAIDs) (e.g., ibuprofen) are most effective. Opioid analgesics are usually not used for the pain associated with pericarditis.

Three days after an anterior wall MI, a patient is in the critical care unit. The patient is receiving oxygen at 4 L/min by nasal cannula; nitroglycerin paste, 1-inch q6h; and Lopressor 25 mg PO q12h. The monitor shows that the patient is beginning to have premature ventricular contractions (PVCs). Over the course of the next several hours, the PVCs increase in frequency to more than 15 per minute, with occasional runs of multifocal bigeminal PVCs. The patient's vital signs follow: heart rate, 84 beats/min; sinus rhythm with described PVCs; blood pressure, 124/68 mm Hg; respirations, 20 breaths/min; and SpO2, 92%. Laboratory values are blood pH, 7.44; potassium, 4.4mEq/L; and magnesium, 1.0mEq/L. Which of the following, if ordered, would be inappropriate for this patient? a. Lidocaine 100 mg IV push b. Increase oxygen to 6 L/min by nasal cannula c. Potassium chloride 40 mEq in 250 mL 0.9% saline IV piggyback over a 4-hour period d. Magnesium sulfate 2 g IV piggyback over a 2-hour period

c. Potassium chloride 40 mEq in 250 mL 0.9% saline IV piggyback over a 4-hour period PVCs are initially controlled by administering oxygen to reduce myocardial hypoxia and by correcting acid-base or electrolyte imbalances. In the setting of an acute myocardial infarction, premature ventricular contractions (PVCs) are pharmacologically treated if they have the following characteristics: frequent (>6/min), closely coupled (R-on-T phenomenon), multiform shapes, and occurrence in bursts of three or more, increasing the risk of sustained ventricular tachycardia. The potassium level is within normal limits and replacement is not warranted. All other interventions listed are appropriate for this patient.

The nurse notes premature ventricular contractions (PVCs) while suctioning a patient's endotracheal tube. Which action by the nurse is a priority? a. Decrease the suction pressure to 80 mm Hg. b. Document the dysrhythmia in the patient's chart. c. Stop and ventilate the patient with 100% oxygen. d. Give antidysrhythmic medications per protocol.

c. Stop and ventilate the patient with 100% oxygen. Dysrhythmias during suctioning may indicate hypoxemia or sympathetic nervous system stimulation. The nurse should stop suctioning and ventilate the patient with 100% oxygen. Lowering the suction pressure will decrease the effectiveness of suctioning without improving the hypoxemia. Because the PVCs occurred during suctioning, there is no need for antidysrhythmic medications (which may have adverse effects) unless they recur when the suctioning is stopped and patient is well oxygenated.

Which action will the nurse implement for a patient who arrives for a calcium-scoring CT scan? a. Insert an IV catheter. b. Administer oral sedative medications. c. Teach the patient about the procedure. d. Confirm that the patient has been fasting.

c. Teach the patient about the procedure. The nurse will need to teach the patient that the procedure is rapid and involves little risk. None of the other actions are necessary.

The mean arterial pressure (MAP) is calculated by a. averaging three of the patient's blood pressure readings over a 6-hour period. b. dividing the systolic pressure by the diastolic pressure. c. adding the systolic pressure and two diastolic pressures and then dividing by 3. d. dividing the diastolic pressure by the pulse pressure.

c. adding the systolic pressure and two diastolic pressures and then dividing by 3. The mean arterial pressure is one-third systole and two-thirds diastole.

A patient is admitted to the ICU with diagnosis of fever of unknown origin. The patient is complaining of fatigue, malaise, joint pain, and shivering. VS include: T 103F, HR 90 beats/min, RR 22 breaths/min, BP 132/78, and O sat 94% on 2L/NC. Blood cultures are ordered. The patient has a history of MI 3 months ago. The nurse suspects that the patient has developed a. CAD. b. heart failure. c. endocarditis. d. pulmonary embolus.

c. endocarditis. Initial symptoms include fever, sometimes accompanied by rigor (shivering), fatigue, and malaise, with up to 50% of patients complaining of myalgias and joint pain. Blood cultures are drawn during periods of elevated temperature.

A 68-year-old patient is admitted to the critical care unit with reports of midchest pressure radiating into the jaw and shortness of breath when walking up stairs. The patient is admitted with a diagnosis of "rule out myocardial infarction." When inspecting the patient, the nurse notes that the patient needs to sit in a high Fowler position to breathe. This may indicate a. pericarditis. b. anxiety. c. heart failure. d. angina.

c. heart failure. Sitting upright to breathe may be necessary for the patient with acute heart failure, and leaning forward may be the least painful position for a patient with pericarditis.

Vasopressors are used cautiously with critical care patients because they can cause a. vasoconstriction of the smooth muscles. b. vasodilation of the smooth muscles. c. increased afterload of the heart. d. decreased preload of the heart

c. increased afterload of the heart. Vasopressors are not widely used in the treatment of critically ill cardiac patients because the dramatic increase in afterload is taxing to a damaged heart. Vasopressin, also known as antidiuretic hormone, has become popular in the critical care setting for its vasoconstrictive effects. At higher doses, vasopressin directly stimulates V1 receptors in vascular smooth muscle, resulting in vasoconstriction of capillaries and small arterioles.

During discharge teaching with a 68-year-old patient who had a mitral valve replacement with a mechanical valve, the nurse instructs the patient on the a. use of daily aspirin for anticoagulation. b. correct method for taking the radial pulse. c. need for frequent laboratory blood testing. d. need to avoid any physical activity for 1 month.

c. need for frequent laboratory blood testing. Anticoagulation with warfarin (Coumadin) is needed for a patient with mechanical valves to prevent clotting on the valve. This will require frequent international normalized ratio (INR) testing. Daily aspirin use will not be effective in reducing the risk for clots on the valve. Monitoring of the radial pulse is not necessary after valve replacement. Patients should resume activities of daily living as tolerated.

A patient with diabetes mellitus and chronic stable angina has a new order for captopril (Capoten). The nurse should teach the patient that the primary purpose of captopril is to a. lower heart rate. b. control blood glucose levels. c. prevent changes in heart muscle. d. reduce the frequency of chest pain.

c. prevent changes in heart muscle. The purpose for angiotensin-converting enzyme (ACE) inhibitors in patients with chronic stable angina who are at high risk for a cardiac event is to decrease ventricular remodeling. ACE inhibitors do not directly impact angina frequency, blood glucose, or heart rate.

When caring for a patient with mitral valve stenosis, it is most important that the nurse assess for a. diastolic murmur. b. peripheral edema. c. shortness of breath on exertion. d. right upper quadrant tenderness.

c. shortness of breath on exertion. The pressure gradient changes in mitral stenosis lead to fluid backup into the lungs, resulting in hypoxemia and dyspnea. The other findings also may be associated with mitral valve disease but are not indicators of possible hypoxemia.

ST segment monitoring for ischemia has gained increasing importance with the advent of thrombolytic therapy. The most accurate method for monitoring the existence of true ischemic changes is a. T-wave inversion in leads overlying the ischemia. b. ST segment depression in leads overlying the ischemia. c. adjusting the gain control on bedside monitoring for best visualization. d. 12-lead ECG for confirmation.

d. 12-lead ECG for confirmation. Cardiac biomarkers are proteins that are released from damaged myocardial cells. The initial elevation of cTnI, cTnT, and CK-MB occurs 3 to 6 hours after the acute myocardial damage. This means that if an individual comes to the emergency department as soon as chest pain is experienced, the biomarkers will not have risen. For this reason, it is clinical practice to diagnose an acute myocardial infarction by 12-lead electrocardiography and clinical symptoms without waiting for elevation of cardiac biomarkers.

Which action will the nurse need to do when preparing to assist with the insertion of a pulmonary artery catheter? a. Determine if the cardiac troponin level is elevated. b. Auscultate heart and breath sounds during insertion. c. Place the patient on NPO status before the procedure. d. Attach cardiac monitoring leads before the procedure.

d. Attach cardiac monitoring leads before the procedure. Dysrhythmias can occur as the catheter is floated through the right atrium and ventricle, and it is important for the nurse to monitor for these during insertion. Pulmonary artery catheter insertion does not require anesthesia, and the patient will not need to be NPO. Changes in cardiac troponin or heart and breath sounds are not expected during pulmonary artery catheter insertion.

A patient is admitted to the ICU after a positive exercise treadmill test with a diagnosis of CAD and stable angina. Radiographic test show that the patient has blockage in the left main coronary artery and four other vessels. The nurse anticipates that the patient will be scheduled for a. medical therapy antianginal medications. b. PCI. c. TAVR. d. CABG.

d. CABG. Early studies demonstrated coronary artery bypass graft surgery was more effective than medical therapy for improving survival in patients with left main or three-vessel coronary artery disease and at relieving anginal symptoms. Medical therapy is recommended if the ischemia is prevented by antianginal medications that are well tolerated by the patient. Surgical revascularization has been shown to be more efficacious than percutaneous coronary intervention in patients with multivessel or left main coronary disease. Transcatheter aortic valve replacement is a transformational therapy for patients who have severe aortic stenosis but who are extremely high-risk surgical candidates or who are inoperable by virtue of associated co-morbidities.

A patient who has had chest pain for several hours is admitted with a diagnosis of rule out acute myocardial infarction (AMI). Which laboratory test should the nurse monitor to help determine whether the patient has had an AMI? a. Myoglobin b. Homocysteine c. C-reactive protein d. Cardiac-specific troponin

d. Cardiac-specific troponin Troponin levels increase about 4 to 6 hours after the onset of myocardial infarction (MI) and are highly specific indicators for MI. Myoglobin is released within 2 hours of MI, but it lacks specificity and its use is limited. The other laboratory data are useful in determining the patient's risk for developing coronary artery disease (CAD) but are not helpful in determining whether an acute MI is in progress.

Which of the following values, when elevated, places the patient at lowest risk for CAD? a. Very-low-density lipoproteins (VLDLs) b. Triglycerides c. Low-density lipoproteins (LDLs) d. High-density lipoproteins (HDLs)

d. High-density lipoproteins (HDLs) All of the reasons are not completely understood, but one recognized physiologic effect is the ability of HDL to promote the efflux of cholesterol from cells. This process may minimize the accumulation of foam cells in the artery wall and thus decrease the risk of developing atherosclerosis. High HDL levels confer both anti-inflammatory and antioxidant benefits on the arterial wall. In contrast, a low HDL level is an independent risk factor for the development of CAD and other atherosclerotic conditions.

Which of the following cholesterol values indicates a heightened risk for the development of CAD? a. Total cholesterol level of 170 mg/dL b. HDL cholesterol level of 30 mg/dL c. Triglyceride level of 120 mg/dL d. LDL cholesterol level >190 mg/dL

d. LDL cholesterol level >190 mg/dL Low-density lipoprotein (LDL) cholesterol is usually described as the "bad cholesterol" because high levels are associated with an increased risk of acute coronary syndrome (ACS), stroke, and peripheral arterial disease (PAD). High LDL levels initiate the atherosclerotic process by infiltrating the vessel wall and binding to the matrix of cells beneath the endothelium. Total cholesterol levels below 200 are considered normal. High-density lipoprotein (HDL) cholesterol levels below 40 are at low risk of coronary artery disease. Triglyceride levels below 150 are considered normal.

Which assessment information is most important for the nurse to obtain to evaluate whether treatment of a patient with anaphylactic shock has been effective? a. Heart rate b. Orientation c. Blood pressure d. Oxygen saturation

d. Oxygen saturation Because the airway edema that is associated with anaphylaxis can affect airway and breathing, the oxygen saturation is the most critical assessment. Improvements in the other assessments will also be expected with effective treatment of anaphylactic shock.

A nurse from the ICU receives report from the ED nurse on a patient that includes a diagnosis of syncope unknown etiology. Orthostatic VS lying: 110/80 mm/Hg; sitting: 100/74 mm/Hg; standing: 92/40 mm/Hg. Based on this information, the nurse should monitor the patient's a. breathing. b. dietary intake. c. peripheral pulses. d. activity.

d. activity. Postural (orthostatic) hypotension occurs when the systolic blood pressure drops by 10 to 20 mm Hg or the diastolic blood pressure drops by 5 mm Hg after a change from the supine posture to the upright posture. This is usually accompanied by dizziness, lightheadedness, or syncope. If a patient experiences these symptoms, it is important to complete a full set of postural vital signs before increasing the patient's activity level.

Several hours after an open surgical repair of an abdominal aortic aneurysm, the UAP reports to the nurse that urinary output for the past 2 hours has been 40 mL. The nurse notifies the health care provider and anticipates an order for a(n) a. hemoglobin count. b. additional antibiotic. c. decrease in IV infusion rate. d. blood urea nitrogen (BUN) level.

d. blood urea nitrogen (BUN) level. The decreased urine output suggests decreased renal perfusion, and monitoring of renal function is needed. There is no indication that infection is a concern, so antibiotic therapy and a WBC count are not needed. The IV rate may be increased because hypovolemia may be contributing to the patient's decreased urinary output.

The nurse is caring for a patient with a subarachnoid hemorrhage who is intubated and placed on a mechanical ventilator with 10 cm H2O of peak end-expiratory pressure (PEEP). When monitoring the patient, the nurse will need to notify the health care provider immediately if the patient develops a. oxygen saturation of 93%. b. respirations of 20 breaths/minute. c. green nasogastric tube drainage. d. increased jugular venous distention.

d. increased jugular venous distention. Increases in jugular venous distention in a patient with a subarachnoid hemorrhage may indicate an increase in intracranial pressure (ICP) and that the PEEP setting is too high for this patient. A respiratory rate of 20, O saturation of 93%, and green nasogastric tube drainage are within normal limits.

To maintain proper cuff pressure of an endotracheal tube (ET) when the patient is on mechanical ventilation, the nurse should a. inflate the cuff with a minimum of 10 mL of air. b. inflate the cuff until the pilot balloon is firm on palpation. c. inject air into the cuff until a manometer shows 15 mm Hg pressure. d. inject air into the cuff until a slight leak is heard only at peak inflation.

d. inject air into the cuff until a slight leak is heard only at peak inflation. The minimal occluding volume technique involves injecting air into the cuff until an air leak is present only at peak inflation. The volume to inflate the cuff varies with the ET and the patient's size. Cuff pressure should be maintained at 20 to 25 mm Hg. An accurate assessment of cuff pressure cannot be obtained by palpating the pilot balloon.

The nurse teaches the patient being evaluated for rhythm disturbances with a Holter monitor to a. connect the recorder to a computer once daily. b. exercise more than usual while the monitor is in place. c. remove the electrodes when taking a shower or tub bath. d. keep a diary of daily activities while the monitor is worn.

d. keep a diary of daily activities while the monitor is worn. The patient is instructed to keep a diary describing daily activities while Holter monitoring is being accomplished to help correlate any rhythm disturbances with patient activities. Patients are taught that they should not take a shower or bath during Holter monitoring and that they should continue with their usual daily activities. The recorder stores the information about the patient's rhythm until the end of the testing, when it is removed and the data are analyzed.

When assisting with the placement of a pulmonary artery (PA) catheter, the nurse notes that the catheter is correctly placed when the monitor shows a a. typical PA pressure waveform. b. tracing of the systemic arterial pressure. c. tracing of the systemic vascular resistance. d. typical PA wedge pressure (PAWP) tracing.

d. typical PA wedge pressure (PAWP) tracing. The purpose of a PA line is to measure PAWP, so the catheter is floated through the pulmonary artery until the dilated balloon wedges in a distal branch of the pulmonary artery, and the PAWP readings are available. After insertion, the balloon is deflated and the PA waveform will be observed. Systemic arterial pressures are obtained using an arterial line and the systemic vascular resistance is a calculated value, not a waveform.


Kaugnay na mga set ng pag-aaral

Econ Final (Chapter Tests Questions)

View Set

MAR3023 Brady Exam 3 (Ch. 11-16) Tb Questions

View Set

A&P Final (Autonomic Nervous System)

View Set

Lab Simulation 3-1: Working with Steganography

View Set